「N Engl J Med」の記事一覧

SARS-CoV-2に対するmRNAワクチン 中間報告
SARS-CoV-2に対するmRNAワクチン 中間報告
An mRNA Vaccine against SARS-CoV-2 - Preliminary Report N Engl J Med. 2020 Nov 12;383(20):1920-1931. doi: 10.1056/NEJMoa2022483. Epub 2020 Jul 14. 原文をBibgraph(ビブグラフ)で読む 上記論文の日本語要約 【背景】重症急性呼吸器症候群コロナウイルス2(SARS-CoV-2)は、2019年半に出現した後、世界的に広がり、ワクチン開発を急ぐ国際的な取り組みが促されている。ワクチン候補のmRNA-1273は、融合前の安定化したSARS-CoV-2スパイクタンパクをコードする。 【方法】18-55歳の健康成人45例を対象に第1相用量漸増非盲検試験を実施し、mRNA-1273ワクチン25μg、100μg、250μgいずれかの用量を28日間隔を空けて2回接種した。各用量群に15例が参加した。 【結果】ワクチン初回接種後、高用量群ほど接種後の抗体反応が高かった[酵素結合免疫吸着測定法で測定した29日目の抗S-2P抗体の幾何平均抗体価(GMT):25μg群40,227、100μg群109,209、250μg群213,526]。2回目の接種後、抗体価は上昇した(57日目のGMT:順に299,751、782,719、1,192,154)。2回目の接種後、評価した全例から2通りの方法で血清中和活性が検出され、その値は対照の回復期血清検体パネルの分布の上位半分とほぼ同じだった。参加者の半数以上に疲労、悪寒、頭痛、筋肉痛、注射部位疼痛などの非自発的な有害事象報告があった。2回目の接種の方が全身性有害事象が多く認められ、最高用量群では特に多く、250μg投与群の3例(21%)に重篤な有害事象が1つ以上報告された。 【結論】mRNA-1273ワクチンは全例で抗SARS-CoV-2免疫反応を誘導し、試験に支障を来す安全性の懸念は認められなかった。この結果は、このワクチンの開発継続を支持するものである。 第一人者の医師による解説 第3相試験では 100 μg、28日間隔の投与で94.1%の有効性 田中 栄 東京大学医学部整形外科教授 MMJ. April 2021;17(2):40 新型コロナウイルス感染症パンデミック終息に向けての切り札として期待されているワクチンの開発は驚異的なスピードで進んでおり、2020年12月には世界に先駆けて英国で承認されたアデノウイルスベクターワクチン(Oxford-AstraZeneca COVID-19ワクチン[AZD1222])の投与が開始された。今回のワクチン開発を特徴づけているのが、これまで臨床で使用されてこなかったタイプのワクチン-mRNAワクチン-の登場である。本論文は米国Moderna社が開発したmRNAワクチン mRNA-1273の第1相試験に関する報告で、オンラインでは2020年7月14日に掲載された。 SARS-CoV-2のスパイク(S)蛋白は突起様の構造を形成し、ウイルスが宿主細胞に感染する際、細胞膜上の受容体ACE(アンジオテンシン変換酵素)2と結合する重要な役割を担っている。mRNA1273ワクチンは、この一部(S-2P)を抗原として用いている。このワクチンの設計では、mRNAに人為的に変異(S2サブユニット中心ヘリックスの連続するアミノ酸2個をプロリンに置換)を導入することでウイルスが受容体に結合する前の構造(prefusion conformation)を安定的にとるように工夫している。このような構造をとることで抗体誘導能が10倍増加するという。mRNA-1273ワクチンは変異型S-2PをコードするmRNAを脂質ナノ粒子内に封入した製剤であり、動物実験では変異型S-2P蛋白そのものをアジュバントと一緒に投与した場合に比べ、mRNA型ワクチンは、抗体誘導能は同程度、T細胞免疫誘導は優れていることが報告されている。 本試験では各群15人の健常成人に対して、それぞれ25、100、250μgのmRNA-1273ワクチンを28日の間隔をあけて2回筋注した。抗S-2P抗体は用量依存性に誘導され、2回目投与後は全例で中和活性のある抗体が回復期患者血清に匹敵する程度に誘導された。試験中止が必要な重篤な(serious)有害事象はみられなかったが、ワクチン投与に伴い倦怠感、冷感、頭痛、筋痛、投与部位痛などは半数以上の被験者に生じ、特に250μg投与群では2回目投与後3人に発熱などの全身性の重症(severe)有害事象がみられた。 mRNA-1273ワクチンについてはその後の第3相無作為化プラセボ対照試験において100μg、28日間隔の投与で94.1%の有効性が示された(1)。各国で一般市民への投与が開始されており、本号が出るころにはある程度実臨床での評価が進んでいるものと思われる。 1. Baden LR, et al. N Engl J Med. 2020:NEJMoa2035389.
COVID-19肺炎入院患者に用いるトシリズマブ
COVID-19肺炎入院患者に用いるトシリズマブ
Tocilizumab in Patients Hospitalized with Covid-19 Pneumonia N Engl J Med. 2021 Jan 7;384(1):20-30. doi: 10.1056/NEJMoa2030340. Epub 2020 Dec 17. 原文をBibgraph(ビブグラフ)で読む 上記論文の日本語要約 【背景】新型コロナウイルス感染症(COVID-19)肺炎は過剰な炎症を伴うことが多い。十分な医療サービスを受けていない集団・人種・少数民族の間でCOVID-19発症率が過度に高いが、COVID-19肺炎で入院したこのような患者に対する抗インターロイキン-6受容体抗体トシリズマブの安全性と有効性は明らかになっていない。 【方法】COVID-19肺炎で入院した人工呼吸管理下にない患者を標準治療と併用してトシリズマブ(体重1kgあたり8mgを静脈内投与)またはプラセボを1~2回投与するグループに2対1の割合で無作為に割り付けた。施設の選択で、高リスク患者や少数集団を組み入れる施設を含めることに重点を置いた。主要転帰は28日目までの人工呼吸管理または死亡とした。 【結果】計389例を無作為化し、修正intention-to-treat集団にはトシリズマブ群249例、プラセボ群128例を含め、56.0%がヒスパニックまたはラテンアメリカ系、14.9%が黒人、12.7%がアメリカインディアンまたはアラスカ原住民、12.7%が非ヒスパニック系白人、3.7%がその他または不明であった。28日目までに人工呼吸管理を実施したか死亡した患者の累積率は、トシリズマブ群12.0%(95%信頼区間[CI]8.5~16.9)、プラセボ群19.3%(95%CI 13.3~27.4)であった(人工呼吸管理または死亡のハザード比0.56、95%CI 0.33~0.97、ログランク検定のP=0.04)。生存時間解析で評価した臨床的失敗までの時間は、トシリズマブ群の方がプラセボ群よりも良好であった(ハザード比0.55、95%CI 0.33~0.93)。28日目までに、トシリズマブ群の10.4%、プラセボ群の8.6%にあらゆる原因による死亡が発生した(加重平均差2.0パーセントポイント、95%CI -5.2~7.8)。安全性解析対象集団では、トシリズマブ群250例中38例(15.2%)、プラセボ群127例中25例(19.7%)に重篤な有害事象が発現した。 【結論】人工呼吸管理下にないCOVID-19肺炎入院患者で、トシリズマブによって人工呼吸管理または死亡の複合転帰を辿る確率が低下したが、生存率は改善しなかった。安全性に関する新たな懸念事項は認められなかった。 第一人者の医師による解説 ステロイドや抗ウイルス薬を含む標準治療への トシリズマブの上乗せ効果を確認 金子 祐子 慶應義塾大学医学部リウマチ・膠原病内科准教授 MMJ. April 2021;17(2):43 2019年中国に端を発した新型コロナウイルス(SARS-CoV-2)による感染症(COVID-19)は瞬く間に世界規模の大流行となり、1年を経過した現在も全世界に深刻な影響を及ぼしている。COVID-19患者の血清IL-6濃度は重症度と相関することや、IL-6上昇は人工呼吸器装着の予測となることが報告され、IL-6受容体阻害薬であるトシリズマブはCOVID-19治療薬として期待されていた。 本論文は、入院中のCOVID-19肺炎患者においてトシリズマブの有効性を検証した海外第3相無作為化二重盲検プラセボ対照臨床試験(EMPACTA)結果の報告である。米国と南米、アフリカの6カ国において、主として援助が不十分な少数民族を対象に、人工呼吸器は装着されていないが入院中で酸素投与を必要とする患者を中心としてintention-to-treat(ITT)集団ではトシリズマブ群に249人、プラセボ群に128人が組み入れられた。標準治療として80%以上が全身性ステロイド投与を、80%弱が抗ウイルス薬投与を受けていた。主要評価項目である28日以内の死亡または人工呼吸器装着率は、トシリズマブ群で12.0%、プラセボ群で19.3%とトシリズマブ群で有意に低く、死亡、人工呼吸器装着、集中治療室入室、試験からの脱落を包括した臨床的増悪は、ハザード比0.55でトシリズマブ群の成績が優れていた。28日以内の死亡率(トシリズマブ群10.4%、プラセボ群8.6%)、退院までの日数中央値(トシリズマブ群6.0日、プラセボ群7.5日)、重篤な有害事象の発生率(トシリズマブ群15.2%、プラセボ群19.7%)に関しては両群間で差を認めなかった。 トシリズマブのCOVID-19に対する有効性はこれまで複数の臨床試験で検証されたが、対象とする患者集団によって異なる結果が示されてきた。人工呼吸器装着中を約4割含む重症患者および重症度が低めの患者を対象とした試験では、トシリズマブ治療の有効性は示されず(1),(2)、人工呼吸器装着は10%程度で本研究と同様の患者を対象とした試験では死亡または人工呼吸器装着率や退院までの日数に関して標準治療へのトシリズマブ追加投与が優れていた(3)。総合すると、トシリズマブは中等症から人工呼吸器を装着していない段階の重症患者に、標準治療であるステロイドと抗ウイルス薬への上乗せ効果があると考えられるが、現在も複数の試験が進行中であり、今後の研究結果を注視する必要がある。 1. Ivan O. R, et al. medRxiv 2020.08.27.20183442;doi: https://doi.org/10.1101/2020.08.27.20183442(preprint) 2. Stone JH, et al. N Engl J Med. 2020;383(24):2333-2344. 3. Horby PW et al. medRxiv 2021.02.11.21249258; doi: https://doi.org/10.1101/2021.02.11.21249258(preprint)
インスリン治療歴のない2型糖尿病に用いる週1回のインスリン投与
インスリン治療歴のない2型糖尿病に用いる週1回のインスリン投与
Once-Weekly Insulin for Type 2 Diabetes without Previous Insulin Treatment N Engl J Med. 2020 Nov 26;383(22):2107-2116. doi: 10.1056/NEJMoa2022474. Epub 2020 Sep 22. 原文をBibgraph(ビブグラフ)で読む 上記論文の日本語要約 【背景】2型糖尿病患者の基礎インスリン注射の頻度を減らすことで、患者が治療を受け入れ、アドヒアランスが高まると考えられている。insulin icodecは、糖尿病治療用に週1回投与で設計された開発中の基礎インスリンアナログである。 【方法】ジぺプチジルペプチダーゼ4阻害薬併用の有無を問わず、メトホルミン服用下で血糖制御不良(糖化ヘモグロビン[HbA1c]値7.0-9.5%)で長期インスリン療法歴がない2型糖尿病患者を対象に、insulin icodec週1回投与の有効性と安全性をインスリングラルギンU100の1日1回投与と比較する26週間の第II相無作為化二重盲検ダブルダミー試験を実施した。主要評価項目は、HbA1c値のベースラインから26週時までの変化量とした。このほか、低血糖発作、インスリンによる有害事象などの安全性評価項目を評価した。 【結果】計247例をicodecとグラルギンに(1対1の割合で)無作為に割り付けた。両群のベースラインの患者背景はほぼ同じであり、平均HbA1c値はicodec群8.09%、グラルギン群7.96%であった。HbA1c値ベースラインからの推定平均変化量は、icodec群-1.33%ポイント、グラルギン群-1.15%ポイントであった。26週時の推定平均値がそれぞれ6.69%、6.87%であり、ベースラインからの変化量の推定群間差は-0.18%ポイント(95%CI -0.38~0.02、P=0.08)であった。重症度レベル2(血糖値54mg/dL未満)またはレベル3(重度の認知機能低下)の低血糖発現率は低かった(icodec群1人年当たり0.53件、グラルギン群0.46件、推定率比1.09、95%CI 0.45~2.65)。インスリンによる重要な有害事象に群間差はなく、過敏症率および注射部位反応率が低かった。ほとんどの有害事象が軽度で、重試験薬によると思われる重篤なイベントはなかった。 【結論】2型糖尿病患者に用いるinsulin icodec週1回投与は、インスリングラルギンU100の1日1回投与とほぼ同等の有効性および安全性が示された。 第一人者の医師による解説 患者の治療負担軽減を期待 遷延性低血糖を生じないかなど今後の研究結果の注視必要 林 哲範 北里大学医学部臨床検査診断学・診療講師 MMJ. April 2021;17(2):50 基礎インスリンの注射頻度が減ることによって2型糖尿病患者の治療の受け入れやアドヒアランスが改善し、さらに良好な血糖管理も得られる可能性がある。今回報告された試験は、長期インスリン治療歴がなく、ジペプチジルペプチダーゼ-4(DPP-4)阻害薬の併用下・非併用でメトホルミンを服用中だが血糖コントロール不良(HbA1c7.0~9.5%)の2型糖尿病患者を対象に、週1回投与型の新規インスリンとして開発中のインスリンアイコデク(insulin icodec)の有効性と安全性を1日1回のインスリングラルギンU100を対照として比較検討することを目的に、26週間の無作為化二重盲検ダブルダミー第2相試験として実施された。主要評価項目は、ベースラインから26週間後のHbA1c値の変化とした。安全性の評価項目は低血糖エピソード、インスリン関連有害事象などであった。 適格患者247人をアイコデク群(125人)またはグラルギン群(122人)に無作為に割り付けた。両群の患者背景に有意差はなかった。投与後は血糖自己測定の結果により、週1回、インスリン用量が調整された。ベースラインのHbA1cの平均はアイコデク群8.09%、グラルギン群7.96%であった。26週後のHbA1cの平均変化量はアイコデク群-1.33%、グラルギン群-1.15%で、26週時点のHbA1cの平均はそれぞれ6.69%、6.87%であった。ベースラインからのHbA1c平均変化量の群間差は-0.18%(95%信頼区間 , -0.38 ~ 0.02;P=0.08)で有意差はなかった。副作用の低血糖に関して、血糖値<54mg/dLの低血糖または重度の認知機能障害を伴う低血糖の発生率は両群で同程度であった(1患者・年あたりアイコデク群0.53件、グラルギン群0.46件)。インスリン投与に関連する重要な有害事象の発現率について2群間の差はなかった。有害事象の多くは軽度で、試験薬に関連すると判断された重篤な有害事象はなかった。 結論として、2型糖尿病患者において、週1回のインスリンアイコデクによる治療は、血糖降下作用と安全性プロファイルが1日1回のインスリングラルギンU100と同等であった。 今回の第2相試験で、週1回のインスリンアイコデクはインスリングラルギンと同程度の血糖低下作用、安全性を有することが示唆された。患者の治療への負担軽減が期待される一方で、高齢者糖尿病などで遷延性低血糖を生じないか、低血糖の際にどのような対処がよいかなど、今後の研究結果も注視する必要があると考えられる。
スウェーデンの肥満者研究の肥満手術後の平均余命
スウェーデンの肥満者研究の肥満手術後の平均余命
Life Expectancy after Bariatric Surgery in the Swedish Obese Subjects Study N Engl J Med. 2020 Oct 15;383(16):1535-1543. 原文をBibgraph(ビブグラフ)で読む 上記論文の日本語要約 【背景】肥満があると平均余命が短くなる。肥満手術によって死亡の長期的相対リスクが低下することが知られているが、平均余命にもたらす効果が明らかになっていない。 【方法】Gompertz比例ハザード回帰モデルを用いて、前向き対照スウェーデン肥満者(SOS)研究で肥満手術を施行した患者(手術群)および通常の肥満治療を実施した患者(対照群)、一般集団から抽出した無作為標本となるSOS参照研究の参加者(参照コホート)で死亡率と平均余命を比較した。 【結果】2007例を手術群、2040例を対照群に組み入れ、1135例を参照コホートに組み入れた。解析時点(2018年12月31日)で、死亡率の追跡期間中央値は、手術群24(四分位範囲22~27)年、対照群22(21~27)年であり、試験参加者の99.9%から死亡に関するデータが入手できた。SOS参照コホートでは、追跡期間中央値は20(四分位範囲10~21)年であり、参加者の100%から死亡に関するデータが入手できた。手術群の457例(22.8%)および対照群の539例(26.4%)が死亡した(ハザード比0.77、95%信頼区間[CI]0.68~0.87、P<0.001)。対応するハザード比は、心血管疾患による死亡で0.70(95%CI 0.57~0.85)、がんによる死亡で0.77(同0.61~0.96)であった。手術群の調整後平均余命中央値は、3.0年(95%CI~4.2)であり、対照群より長かったが、一般集団より5.5年短かった。術後90日以内の死亡率は0.2%であり、手術群の2.9%に再手術を施行した。 【結論】肥満手術を施行した肥満患者で、通常の肥満治療より平均余命が長くなった。一般集団と比べると、両群ともに死亡率がなお高かった。 第一人者の医師による解説 遺伝マーカーや手術反応性マーカーの特定で 減量手術実施判断への活用を期待 門脇 孝 国家公務員共済組合連合会 虎の門病院院長 MMJ. April 2021;17(2):51 Swedish Obese Subjects(SOS)研究は、高度肥満症に対する減量手術の前向き長期追跡成績を報告している世界で代表的な減量手術研究の1つである。2007年には、減量手術後平均10.9年の追跡データの解析により死亡率が29%低下したことを発表している。しかし、最近の後ろ向き研究の成績では、減量手術を受けた人の死亡率は一般人口に比べ依然として高率であることが指摘されている。 本研究ではSOS研究の肥満患者で減量手術を受けた群と通常治療を受けた対照群の20年以上の前向き追跡で得られた死亡率をほぼ同年代の一般人口の死亡率と比較した。その結果、減量手術群の死亡率は、通常治療群に対しハザード比0.77と低下し、心血管死ではハザード比0.70、がん死ではハザード比0.77であった。減量手術群では、通常治療群に比べ3.0年の余命延長が認められたが、一般人口との比較では5.5年短命であった。また、90日の周術期死亡率は0.2%であったが、再手術を受けた患者の死亡率は2.9%であった。 本研究は、20年以上の前向き追跡調査の結果、減量手術が心血管死とがん死を減少させることを示した。一方、高度肥満者に通常治療が行われた場合の平均余命が一般人口に比べ約8年短いこと、減量手術によって延長する平均余命は現在のところ約3年であることが明らかとなった。この平均余命の延長幅は、SOS研究のように高度肥満に加え重篤な併発症を有する患者を含む高リスク群における結果で、他の患者集団にそのまま当てはまるものではないことに注意する必要がある。 本研究の結果は、最近発表された北欧5カ国の減量手術後の平均余命が一般人口に比べ依然として短いという結果(1)と矛盾しない。平均余命が依然として短い理由として、減量手術後も一般人口よりはBMI高値であること、代謝異常がすでに術前に大血管や細小血管の不可逆的障害を起こしている場合があること、手術に伴う合併症、減量手術群で増加するアルコール依存症や自殺、転倒・外傷などの要因が関与していると考えられる。 本研究では、減量手術が平均余命延長の観点から特に有益なサブグループを特定する試みを行ったが、特定することはできなかった。最近では、肥満そのものやエネルギーバランスの変化に対する体重や体組織の変化が遺伝的に規定されていることが明らかとなっている。今後、これらの研究により、遺伝マーカーや手術反応性マーカーが特定され、減量手術を行うか否かの決定に活用されることが期待される。 1. Kauppila JH, et al. Gastroenterology. 2019;157(1):119-127.e1.
進行ALK陽性肺がんの1次治療に用いるロルラチニブとクリゾチニブの比較
進行ALK陽性肺がんの1次治療に用いるロルラチニブとクリゾチニブの比較
First-Line Lorlatinib or Crizotinib in Advanced ALK-Positive Lung Cancer N Engl J Med. 2020 Nov 19;383(21):2018-2029. doi: 10.1056/NEJMoa2027187. 原文をBibgraph(ビブグラフ)で読む 上記論文の日本語要約 【背景】第3世代の未分化リンパ腫キナーゼ(ALK)阻害薬ロルラチニブは、治療歴のあるALK陽性非小細胞肺がん(NSCLC)に対する抗腫瘍活性がある。進行ALK陽性NSCLCの1次治療に用いるロルラチニブのクリゾチニブと比較した有効性は明らかになっていない。 【方法】進行または転移性ALK陽性NSCLCがあり、転移性NSCLCに対する全身治療歴のない患者296例を対象に、ロルラチニブをクリゾチニブと比較する第III相国際共同無作為化試験を実施した。主要評価項目は、盲検下の独立中央判定で評価した無増悪生存期間とした。独立に評価した客観的奏効率、頭蓋内奏効率を副次的評価項目とした。病勢進行または死亡の期待数177件中約133件(75%)発生後に有効性の中間解析を実施するよう計画した。 【結果】12カ月時の無増悪生存率はロルラチニブ群78%(95%信頼区間[CI]70~84)、クリゾチニブ群39%(95%CI 30~48)であった(病勢進行または死亡のハザード比0.28、95%CI 0.19~0.41、P<0.001)。ロルラチニブ群の76%(95%CI 68~83)とクリゾチニブ群の58%(95%CI 49~66)に客観的奏効が認められ、測定可能な脳転移があった患者ではそれぞれ82%(95%CI 57~96)と23%(95%CI 5~54)が頭蓋内奏効を得、ロルラチニブを投与した患者の71%が頭蓋内完全奏効を得た。ロルラチニブ群で頻度が高かった有害事象は、高脂血症、浮腫、体重増加、末梢性ニューロパチー、認知障害であった。ロルラチニブは、クリゾチニブと比較すると、グレード3または4の有害事象(主に脂質値異常)が多かった(72% vs. 56%)。それぞれ7%と9%が有害事象のため治療を中止した。 【結論】治療歴のない進行ALK陽性NSCLC患者を対象とした結果の中間解析から、ロルラチニブの投与を投与した患者は、クリゾチニブを投与した患者と比べて無増悪生存期間が有意に長く、頭蓋内奏効の確率が高かった。ロルラチニブで脂質値異常の発現頻度が高かったため、グレード3または4の有害事象発現率はロルラチニブの方がクリゾチニブよりも高かった。 第一人者の医師による解説 ロルラチニブは頭蓋内病変に対して奏効 アレクチニブとの使い分けが臨床上の課題 大谷 咲子 北里大学医学部呼吸器内科診療講師/佐々木 治一郎 北里大学医学部附属新世紀医療開発センター横断的医療領域開発部門臨床腫瘍学教授 MMJ. April 2021;17(2):37 未分化リンパ腫キナーゼ(ALK)融合遺伝子は、非小細胞肺がんの約3~5%に認めるドライバー遺伝子異常である。進行・再発ALK融合遺伝子陽性肺がんに対するALKチロシンキナーゼ阻害薬(ALK-TKI)治療は、プラチナ製剤併用療法との比較試験で無増悪生存期間(PFS)の有意な延長を示したクリゾチニブで確立した(1)。その後、クリゾチニブと第2世代 ALK-TKIアレクチニブの第3相比較試験(ALEX試験)の結果、アレクチニブがPFSの有意な延長を示した(2)。このような背景から日本肺癌学会の「肺癌診療ガイドライン 2020年版」では、ALK融合遺伝子陽性肺がんの1次治療としてアレクチニブを推奨している(3)。 本論文は、未治療ALK融合遺伝子陽性肺がんを対象に第3世代ALK-TKIロルラチニブをクリゾチニブと比較する国際共同無作為化第3相試験(CROWN試験)の中間報告である。本試験には日本を含む23カ国104施設が参加し、対象は未治療の進行 ALK融合遺伝子陽性肺がん患者で、ロルラチニブ群149人、クリゾチニブ群147人に割り付けられた。主要評価項目はPFS、副次評価項目は客観的奏効割合と頭蓋内病変への奏効割合とした。中間解析のデータカットオフ時の12カ月PFS率は、ロルラチニブ群78%、クリゾチニブ群39%、ハザード比(HR)0.28(P<0.001)とロルラチニブ群が有意に優れていた。客観的奏効割合(76% 対 58%)および測定可能脳転移があった患者での奏効割合(82% 対 23%)ともにロルラチニブ群の方がクリゾチニブ群に比べ高かった。さらに頭蓋内病変を有するロルラチニブ群の71%で完全奏効を認めた。ロルラチニブ群で頻度の高い有害事象は高脂血症、浮腫、体重増加、末梢神経障害、認知機能低下であった。また、ロルラチニブ群はクリゾチニブ群よりもグレード3以上の有害事象(主に高脂血症)の発生が多かった(72% 対 56%)。 ロルラチニブはこれまで既存のALK-TKI耐性後の2次治療薬として承認されていたが、CROWN試験の結果より米食品医薬品局(FDA)は1次治療薬として承認した。日本でも2021年3月現在、1次治療薬として承認申請中である。ロルラチニブは他のALK-TKIに比べ特に脳移行性が高く、頭蓋内病変を有する患者だけでなく、頭蓋内病変の発生も抑制し高い病勢制御を期待できる。一方、グレード3以上の有害事象の頻度がやや高いことから、日本ではアレクチニブとの使い分けが臨床上の課題となる。今後、脳転移の有無や患者の状態、合併症に応じて複数のALK-TKIの中から最適な薬剤を選択することが重要となる。 1. Solomon BJ, et al. N Engl J Med. 2014;371(23):2167-2177. 2. Peters S, et al. N Engl J Med. 2017;377(9):829-838. 3. 肺癌診療ガイドライン 2020 年版:183-188.
収縮期心不全に用いるomecamtiv mecarbilによる心筋ミオシン活性化
収縮期心不全に用いるomecamtiv mecarbilによる心筋ミオシン活性化
Cardiac Myosin Activation with Omecamtiv Mecarbil in Systolic Heart Failure N Engl J Med. 2021 Jan 14;384(2):105-116. doi: 10.1056/NEJMoa2025797. Epub 2020 Nov 13. 原文をBibgraph(ビブグラフ)で読む 上記論文の日本語要約 【背景】選択的心筋ミオシン活性化薬omecamtiv mecarbilは、左室駆出率が低下した心不全の心機能を改善することが示されている。心血管転帰にもたらす効果は明らかになっていない。 【方法】左室駆出率が35%未満の収縮期心不全(入院および外来)患者8256例を標準心不全治療に加えてomecamtiv mecarbil群(薬物動態学を基に決定した用量25mg、37.5mg、50mgのいずれかを1日2回)またはプラセボ群に無作為に割り付けた。主要評価項目は、心不全イベント(入院または心不全による救急受診)の初回発生または心血管死の複合とした。 【結果】中央値21.8カ月の間に、omecamtiv mecarbil群4120例中1523例(37.0%)とプラセボ群4112例中1607例(39.1%)に主要評価項目が発生した(ハザード比0.92、95%CI 0.86~0.99、P=0.03)。それぞれ808例(19.6%)、798例(19.4%)が心血管の原因で死亡した(同1.01、0.92~1.11)。カンザスシティ心筋症質問票の総合症状スコア変化量に群間差はなかった。24週時、N末端プロB型ナトリウム利尿ペプチド(NT-proBNP)中央値の試験開始時からの変化量は、omecamtiv mecarbil群の方がプラセボ群よりも10%低く、心臓トロポニンI値中央値は4ng/L高かった。心虚血と心室性不整脈イベントの発現頻度は両群同等だった。 【結論】左室駆出率が低下した心不全にomecamtiv mecarbilを投与すると、心不全イベントと心血管死の複合転帰の発生率がプラセボ投与よりも低かった。 第一人者の医師による解説 作用機序を踏まえると従来の強心薬に比べ安全性は高い さらなる臨床試験の結果に注視 佐野 元昭 慶應義塾大学医学部循環器内科准教授 MMJ. June 2021;17(3):80 左室収縮機能が低下した心不全の治療には、利尿薬、強心薬、神経内分泌因子修飾薬(レニン・アンジオテンシン・アルドステロン系抑制薬、β遮断薬、ネプリライシン阻害薬)、心拍数を低下させるイバブラジン、SGLT2阻害薬などが用いられている。心臓のポンプ機能の低下による心拍出量の減少は、うっ血、浮腫や呼吸困難の原因となるだけでなく、神経内分泌因子を活性化させて心不全の病態を悪化させるため、強心薬を用いて、安全にポンプ機能を立ち上げることができれば、それに越したことはない。現在、日本でよく用いられている強心薬ピモベンダンは、心筋のCa2+感受性を増強する作用やプロテインキナーゼ A(PKA)活性化作用を介して、心筋の収縮力を高めるとともに、心筋拡張機能を改善する。ピモベンダンを心不全患者に投与すると確かに運動耐用能は改善するが、死亡率が上昇する傾向が示されたため、他の薬剤で症状が改善しない場合、不整脈の増悪に注意しながら一時的に使用する薬剤として位置づけられている。カテコラミン類似薬の強心薬デノパミンに関しても同様である。 オメカムチブメカルビルは、ミオシンに結合して心筋収縮力を増強させる新規作用機序による強心薬である(1)。β遮断薬を使用していても強心作用を発揮する。細胞内Ca2+動態に影響を与えないため不整脈による突然死を増加させるリスクは低いと考えられる。また、酸素需要を増加させずに心筋 収縮力を増強できる点も魅力的である。 今回のGALACTIC-HF試験では、症候性慢性心不全で駆出率が35%以下の患者を対象に、標準的な心不全治療に加えてオメカムチブメカルビルを投与することの安全性と有効性が評価された。その結果、プラセボ群と比較し、心血管死および全死亡を増やすことなく、初回の心不全イベント(心不全による入院または緊急受診)または心血管死の複合エンドポイントをわずかではあるが有意に低下させた(ハザード比,0.92;P=0.03)。しかし、最も期待された心不全に伴う症状、身体的制限、生活の質(QOL)の改善は認められなかった。患者の3分の1に植込み型除細動器(ICD)が装着されており、ICDで不整脈死がある程度抑制されていた集団が対象であった点も考慮する必要がある。 作用機序を踏まえると、従来の強心薬に比べ安全性がより高いと考えられるオメカムチブメカルビルに関しては、2020年末、開発・商業化権がアムジェン社からサイトキネティクス社へ移管されることが発表された。今後、国内外での承認申請の動向やさらなる臨床試験の結果を注視したい。 1. Malik FI, et al. Science. 2011;331(6023):1439-1443.
心房細動の初期治療に用いる冷凍アブレーションと薬物療法
心房細動の初期治療に用いる冷凍アブレーションと薬物療法
Cryoablation or Drug Therapy for Initial Treatment of Atrial Fibrillation N Engl J Med. 2021 Jan 28;384(4):305-315. doi: 10.1056/NEJMoa2029980. Epub 2020 Nov 16. 原文をBibgraph(ビブグラフ)で読む 上記論文の日本語要約 【背景】ガイドラインでは、心房細動患者にカテーテルアブレーションを検討する前に1種類以上の抗不整脈薬を試すことが推奨されている。しかし、1次治療にアブレーションを用いた方が洞調律の維持に有効であると思われる。 【方法】未治療の症候性発作性心房細動患者303例を、冷凍バルーンを用いたカテーテルアブレーション実施群と、初期の洞調律回復を目的とした抗不整脈薬投与群に無作為化した。心房頻脈性不整脈を検出するため、全例に植込み型心臓モニタリング機器を留置した。追跡調査期間は12カ月であった。主要評価項目は、カテーテルアブレーション実施後または抗不整脈薬投与開始91~365日後のあらゆる心房頻脈性不整脈(心房細動、心房粗動または心房頻拍)再発の初回記録とした。副次評価項目は、症候性不整脈がないこと、心房細動の負荷、QOLとした。 【結果】1年時、アブレーション群154例中66例(42.9%)、抗不整脈薬群149例中101例(67.8%)に心房頻脈性不整脈の再発が認められた(ハザード比0.48、95%CI 0.35~0.66、P<0.001)。アブレーション群の11.0%、抗不整脈薬群の26.2%に症候性の心房頻脈性不整脈の再発が認められた(ハザード比0.39、95%CI 0.22~0.68)。心房細動が発生していた時間の割合の中央値は、アブレーション群0%(四分位範囲0~0.08)、抗不整脈薬群0.13%(四分位範囲0~1.60)であった。アブレーション群の5例(3.2%)と抗不整脈薬群の6例(4.0%)に重篤な有害事象が発生した。 【結論】症候性発作性心房細動の初期治療を受けた患者を継続的な心調律モニタリングで評価した結果、カテーテルによる冷凍バルーンアブレーションの心房細動再発率が抗不整脈薬による薬物療法よりも有意に低かった。 第一人者の医師による解説 第1選択とするには安全性が非常に重要 侵襲的な手技のリスクは常に念頭に置く必要あり 五十嵐 都 筑波大学医学医療系循環器先進治療研究部門准教授/家田 真樹 筑波大学医学医療系循環器内科教授 MMJ. August 2021;17(4):114 発作性心房細動の初回治療として、ガイドラインでは抗不整脈薬をまず投与し無効な場合にカテーテルアブレーションを行うべきと記載されている(1)。しかしながら、薬物療法の心房細動抑制効果は十分とはいえず、副作用も懸念される。一方、カテーテルアブレーションを薬物療法の無効例に対して行った場合、洞調律維持に有効であったとの報告がある(2)。 今回報告されたEARLY-AF試験では、未治療の発作性心房細動患者を対象に初回治療として冷凍焼灼術(クライオバルーンアブレーション)による心房不整脈の再発抑制効果を抗不整脈薬と比較した。全患者に植込み型心電図記録計(ICM)を植え込み、不整脈を正確に検出できるようにした。その結果、1年の時点で主要評価項目である心房不整脈の再発率はアブレーション群の方が有意に低かった。副次評価項目である心房細動発症の累積時間率(burden)もアブレーション群の方が低かった。有害事象に関して両群間に有意差はなかった(アブレーション群5人:横隔神経麻痺3人、徐脈2人、抗不整脈薬群6人:wide QRS頻拍2人、失神1人、心不全1人、徐脈2人)。 先行研究では、薬物療法でコントロールが不良であった患者への後治療としてアブレーションを行っており、アブレーションの有効性が過大評価されていた可能性がある。本研究の特徴は初回治療としてアブレーションと薬物療法を比較している点と、ICMにより長期間の正確なモニターを行った点である。 最近の研究ではリズムコントロールを早期に行うことは、脳卒中を含む心血管イベントを抑制すると報告されている(3)。また心房細動は進行性の疾患であるため、早期にアブレーションを行うことで心房の線維化など組織的な変化を抑制し長期的な予後を改善させるかもしれない。しかしながら、アブレーションを第1選択とするには安全性が非常に重要な点である。本研究では有害事象発生率は2群間で差がなくアブレーションに関連した死亡、血栓塞栓イベントはなかったが、侵襲的な手技のリスクは常に念頭に置く必要がある。また、心房細動の累積時間率に関して2群間の差はそれほど大きくなくアブレーションを強く勧める根拠にはならないかもしれない。本研究は追跡期間が短いため、アブレーションの長期的な効果を含め、今後のさらなる検討が期待される。 1. Hindricks G, Eur Heart J. 2021;42(5):373-498. 2. Wilber DJ, et al. JAMA. 2010;303(4):333-340. 3. Kirchhof P, et al. N Engl J Med. 2020;383(14):1305-1316.
ANCA関連血管炎治療に用いるavacopan
ANCA関連血管炎治療に用いるavacopan
Avacopan for the Treatment of ANCA-Associated Vasculitis N Engl J Med. 2021 Feb 18;384(7):599-609. doi: 10.1056/NEJMoa2023386. 原文をBibgraph(ビブグラフ)で読む 上記論文の日本語要約 【背景】C5a受容体阻害薬avacopanは、抗好中球細胞質抗体(ANCA)関連血管炎の治療薬として研究中である。 【方法】この無作為化比較試験では、ANCA関連血管炎患者をavacopan 30mg 1日2回投与とprednisoneの用量漸減法による経口投与群に1対1の割合で割り付けた。全例にシクロホスファミド(その後アザチオプリン)またはリツキシマブを併用した。1つ目の主要評価項目は寛解とし、26週時のバーミンガム血管炎活動性スコア(BVAS)が0点(範囲0-63点、スコアが高いほど疾患活動性が高い)および直前4週間のグルココルチコイド不使用と定義した。2つ目の主要評価項目は寛解維持とし、26週時および52週時の寛解と定義した。両評価項目で非劣性(マージン20%ポイント)および優越性を評価した。 【結果】計331例を無作為化し、166例をavacopan群、165例をprednisone群に割り付けた。試験開始時のBVAS平均スコアは両群とも16点であった。avacopan群166例中120例(72.3%)、prednisone群164例中115例(70.1%)が26週時に寛解(1つ目の主要評価項目)を得た(推定公差3.4%ポイント、95%CI -6.0-12.8、非劣性のP<0.001、優越性のP=0.24)。avacopan群166例中109例(65.7%)、prednisone群164例中90例(54.9%)が52週時に寛解を維持していた(2つ目の主要評価項目、推定公差12.5%ポイント、95%CI 2.6-22.3、非劣性のP<0.001、優越性のP=0.007)。avacopan群の37.3%、prednisone群の39.0%に重篤な有害事象(血管炎悪化を除く)が発生した。 【結論】ANCA関連血管炎患者を対象とした本試験で、avacopanは26週時の寛解でprednisone漸減投与に対して非劣性が示されたが優越性は示されず、52週時の寛解維持では優越性が示された。全例がシクロホスファミドまたはリツキシマブを併用していた。52週以降のavacopanの安全性および臨床効果は、本試験では評価しなかった。 第一人者の医師による解説 グルココルチコイドの副作用を低減 ANCA関連血管炎の新治療法に期待 三森 経世 医療法人医仁会武田総合病院院長 MMJ. August 2021;17(4):121 ANCA関連血管炎(AAV)は小動脈が侵され抗好中球細胞質抗体(ANCA)が陽性となる自己免疫疾患で、多発血管炎性肉芽腫症(GPA)、顕微鏡的多発血管炎(MPA)および好酸球性多発血管炎性肉芽腫症(EGPA)が含まれ、急速に進行する糸球体腎炎、間質性肺炎、末梢神経炎などの多彩な臓器病変を呈する重篤な疾患である。従来、AAVの治療は大量グルココルチコイド(GC)とシクロホスファミドまたはリツキシマブなどの免疫抑制薬の併用が主体であった。しかし、再燃が多く、長期にわたるGCの副作用が問題となっている。 AAVの病態にはANCAと補体が関与し、ANCAが好中球表面に発現した自己抗原に結合するとともに、C5aがC5a受容体に結合して好中球のケモタキシスと活性化を引き起こすと考えられている。アバコパンは低分子経口 C5a受容体アンタゴニストであり、C5a受容体に選択的に結合して、C5aとANCAによる好中球の活性化を抑制すると考えられる。 本論文は、世界の143施設が参加し、AAVに対するアバコパンの有効性と安全性を検討した第3相試験の報告である。アバコパン 30mgの1日2回経口投与(A群)166人とプレドニゾン漸減療法(P群:1日60mgで 開始し21週までに中止)164人が二重プラセボ二重盲検試験で比較された。GPA181人とMPA149人がエントリーされ、PR3-ANCAが43%、MPO-ANCAが57%を占めたが、解析では両者は区別されていない。全例で免疫抑制薬(シクロホスファミドまたはリツキシマブ)が併用され、途中増悪時のGC救済療法は許容されている。 26週目の寛解達成率(Birmingham Vasculitis Activity Score[BVAS]=0および4週間前までのGC中止)はA群72.3%、P群70.1%であり、A群のP群に対する非劣性が証明された。52週目の寛解維持率はA群65.7%、P群54.9%で、A群の非劣性のみならず優越性も認められた。また、A群はP群より52週目までの再燃率が有意に低く、推算糸球体濾過量(eGFR)、蛋白尿、生活の質(QOL)の改善でも上回っていた。GCによる副作用の発現率は当然ながら、P群でA群よりも高かった。死亡例はA群2例、P群4例で、肝機能障害がA群で9例にみられたが、安全性に関して両群間で有意差はみられなかった。 本試験で、AAVにおいて補体阻害薬であるアバコパンのプレドニゾン漸減療法に対する非劣性と、52週での優越性が証明されたことは、将来の治療戦略に大きな変革をもたらす可能性があり、GCの使用を減らし副作用を低減できることにも大きな利点がある。長期成績と長期安全性、寛解導入後の薬剤減量・中止の可能性などが今後の課題である。
脊髄性筋萎縮症I型に用いるrisdiplam
脊髄性筋萎縮症I型に用いるrisdiplam
Risdiplam in Type 1 Spinal Muscular Atrophy N Engl J Med. 2021 Mar 11;384(10):915-923. doi: 10.1056/NEJMoa2009965. Epub 2021 Feb 24. 原文をBibgraph(ビブグラフ)で読む 上記論文の日本語要約 【背景】脊髄性筋萎縮症I型は、機能性生存運動ニューロン(SMN)タンパク低値によって生じるまれな進行性神経筋疾患である。risdiplamは、SMN2のRNA前駆体スプライシングを修飾し、機能性SMNタンパク値を上昇させる経口投与可能な小分子である。 【方法】支えなしで座位が保持できない1-7カ月齢の脊髄性筋萎縮症I型乳児を対象にrisdiplamを検討した、2段階から成る第II/III相非盲検試験のパート1の結果を報告する。主要評価項目は、安全性、薬物動態、薬力学(血中SMNをタンパク濃度など)およびパート2で用いるrisdiplamの用量決定とした。5秒間以上の支えなしでの座位保持能を探索的評価項目とした。 【結果】乳児計21例を組み入れた。4例を低用量群とし、12カ月時の最終用量を1日当たり0.08mg/kgとした。17例を高用量群とし、12カ月時の最終用量を1日当たり0.2mg/kgとした。ベースラインの血中SMNタンパク濃度中央値は低用量群1.31ng/mL、高用量群2.54ng/mLであり、12カ月時に中央値はそれぞれ3.05ng/mL、5.66ng/mLまで増加し、ベースラインの中央値のそれぞれ3.0倍、1.9倍となった。重篤な有害事象に肺炎、気道感染、急性呼吸不全があった。本稿発表時点では、4例が呼吸器合併症のため死亡している。高用量群の7例が支えなしで5秒間以上座位が保持できたが、低用量群では1例も認められなかった。試験のパート2に用いる用量には高用量(1日当たり0.2mg/kg)を用いることが決定した。 【結論】脊髄性筋萎縮症I型乳児で、経口risdiplamを用いた治療によって血中機能性SMNタンパク発現量が増加した。 第一人者の医師による解説 リスジプラムは全身に作用 有効性と安全性の追加検証を期待 佐橋 健太郎 名古屋大学医学部附属病院脳神経内科講師/勝野 雅央 名古屋大学大学院医学研究科神経内科学教授 MMJ. August 2021;17(4):111 脊髄性筋萎縮症(SMA)は主にSMN1遺伝子欠失変異によるSMN蛋白欠乏により、脳幹や脊髄の下位運動ニューロン変性に伴う、進行性筋力低下、筋萎縮をきたす予後不良の遺伝性疾患である。SMA最多の重症の1型は6カ月齢までに発症し、座位保持能を獲得できず、呼吸筋麻痺により寿命は中央値10.5カ月(1)とされる。ヒトはSMN1重複遺伝子であるSMN2を有するが、mRNA前駆体のエクソン 7の選択的スプライシングによりSMN2からは機能性 SMN蛋白が十分に産生されない。治療薬としては、核酸医薬ヌシネルセンや低分子化合物リスジプラムによるSMN2スプライシング制御治療や、組換えアデノウイルスベクター製剤オナセムノゲン アベパルボベクによるSMN遺伝子補充療法が開発されており、リスジプラムは全身に作用する特色がある。 本論文は、リスジプラム開発元 F. Hoffmann-La Roche社による研究支援のもと、1型 SMN乳児21人(中央値6.7カ月齢:他試験より経過が長い例(2),(3))を対象に実施されたリスジプラム第2/3相非盲検単一群試験(FIREFISH試験)のパート 1の報告である。主要評価項目は安全性、薬物動態、薬力学と、パート 2のための投与量選択とし、また事後分析による探索的評価項目として、永続的な呼吸補助の必要のない無イベント生存、支持なしで5秒以上の座位保持能 (BSID-Ⅲの第22項)、CHOPINTENDとHINE-2運動機能スコアなどが設定された。その結果、12カ月の観察期間で低用量、高用量コホートともに血漿 SMN蛋白上昇が示されたが(それぞれベースライン値の3.0、1.9倍)、個人内の測定値のばらつきが問題として挙げられた。全体21人中19人で無イベント生存、高用量コホート 7人で座位保持能獲得が確認され、また自然歴ではほぼ観察されない運動機能スコアの改善が特に高用量コホートでみられている。一方、重篤な有害事象として肺炎、気道感染がみられた。死亡例の原因は呼吸器合併症であり、SMAに伴う呼吸不全と分類されているが、多くが高用量コホートであり、薬剤関連性の可能性除外も必要と考えられる。最終的にパート 2では高用量のリスジプラム使用が支持されており、さらにリアルワールド設定に近い2?25歳の、重症度の下がる2/3型対象のプラセ ボ 対照二重盲検第2/3相試験(SUNFISH試験:NCT02908685)も進行中であり、リスジプラムの有効性と安全性についての追加検証が待たれる。 略 号:BSID- Ⅲ(Bayley Scales of Infant and Toddler Development, third edition)、CHOP-INTEND (Children's Hospital of Philadelphia Infant Test of Neuromuscular Disorders)、HINE-2(Hammersmith Infant Neuromuscular Examination) Finkel RS, et al. Neurology. 2014;83(9):810-817. Finkel RS, et al. N Engl J Med. 2017;377(18):1723-1732. Mendell JR, et al. N Engl J Med. 2017;377(18):1713-1722.
非アルコール性脂肪肝炎に用いるセマグルチド皮下投与のプラセボ対照試験
非アルコール性脂肪肝炎に用いるセマグルチド皮下投与のプラセボ対照試験
A Placebo-Controlled Trial of Subcutaneous Semaglutide in Nonalcoholic Steatohepatitis N Engl J Med. 2021 Mar 25;384(12):1113-1124. doi: 10.1056/NEJMoa2028395. Epub 2020 Nov 13. 原文をBibgraph(ビブグラフ)で読む 上記論文の日本語要約 【背景】非アルコール性脂肪肝炎(NASH)はよく見られる疾患であり、合併症率と死亡率が上昇するが、治療選択肢が少ない。NASHに用いるグルカゴン様ペプチド1(GLP-1)受容体作動薬セマグルチドの有効性と安全性は不明である。 【方法】生検でNASHが確定した肝線維化分類F1、F2またはF3の患者を対象に、72週間の第II相二重盲検試験を実施した。患者をセマグルチド0.1mg、0.2mg、0.4mgを1日1回皮下投与するグループと対応するプラセボを投与するグループに3対3対3対1対1対1の割合で割り付けた。主要評価項目は、肝線維化の増悪がないNASHの消失とした。検証的副次的評価項目は、NASHの増悪がない1段階以上の肝線維化分類改善とした。この評価項目の解析は肝線維化分類がF2またはF3の患者のみを対象とし、その他の解析は全例を対象に実施した。 【結果】計320例(このうち230例が肝線維化分類F2またはF3)をセマグルチド0.1 mg群(80例)、同0.2mg群(78例)、同0.4mg群(82例)、プラセボ群(80例)に割り付けた。肝線維化の増悪を伴わずNASHが消失した患者の割合は、0.1mg群40%、0.2mg群36%、0.4mg群59%、プラセボ群17%であった(プラセボと比較したセマグルチド0.4mgのP<0.001)。0.4mg群の43%とプラセボ群の33%に肝線維化分類の改善が認められた(P=0.48)。平均体重減少率は、0.4mg群で13%、プラセボ群1%であった。悪心、便秘、嘔吐の発現率は、0.4mg群の方がプラセボ群よりも高かった(悪心42% vs. 11%、便秘22% vs. 12%、嘔吐15% vs. 2%)。セマグルチドを投与した患者3例(1%)に悪性新生物が報告されたが、プラセボを投与した患者では1例も報告されなかった。全体で、セマグルチド群の15%とプラセボ群の8%に新生物(良性、悪性または不明)が報告されたが、特定の臓器に発現するパターンは認められなかった。 【結論】NASH患者を対象とした第II相試験では、セマグルチド群で、プラセボ群と比較してNASHが消失した患者の割合が有意に高かった。しかし、線維化分類が改善した患者の割合に群間差は認められなかった。 第一人者の医師による解説 全身疾患を踏まえたNAFLD治療 ─木も見て森も見る─ 芥田 憲夫 虎の門病院肝臓内科医長 MMJ. August 2021;17(4):118 非アルコール性脂肪性肝疾患(NAFLD)はメタボリックシンドローム関連因子とともに脂肪肝を認めた病態である。その中でも進行性で肝硬変や肝がんの発症母地ともなる非アルコール性脂肪肝炎(NASH)の日本における患者数は400万人前後とされる。3大死因は、心血管疾患(CVD)、肝がん以外の悪性新生物、次いで肝不全や肝がんを含む肝関連事象であり、これらは肝臓の線維化進行に伴いリスクが上昇するとされる(1)。これまでNASHの肝線維化改善を目指した臨床試験が多数行われてきたが、現時点で既承認薬はない。 本論文は、病理所見に基くNASHの消失を指標としてグルカゴン様ペプチド(GLP)-1受容体作動薬のセマグルチド1日1回皮下投与の効果をプラセボと比較した第2相試験の報告である。糖尿病のない患者も30%台で含まれ、セマグルチドは0.1、0.2、0.4mg/日の3群とし72週時点の肝組織改善を評価している。主要評価項目は線維化ステージ2か3の進行例における肝線維化の悪化を伴わないNASHの消失(炎症改善)、副次評価項目はNASH悪化を伴わない肝線維化の改善としている。その結果、主要評価項目はセマグルチド 0.4mg群が59%で、プラセボ群の17%と比較して有意に高かった。一方、副次評価項目はセマグルチド0.4mg群が43%で、プラセボ群の33%と比較し有意差はなかった。有害事象はセマグルチド群において胃腸障害が多かった。今回の結果に基づき、第3相試験に進んでいる。ここで留意すべき点は、NASHは肝臓だけの疾患ではなく、主な死因はCVDということである。さらに、NASHの肝線維化は生命予後に影響する重要な要因であるが、線維化の改善を主要評価項目に据えてきた多数の臨床試験が成功しなかった経緯を考えると、主要評価項目を肝線維化を惹起する炎症の改善へとシフトするような柔軟な対応も必要となる。以上の問題点を解決することが期待されるのがセマグルチドであろう。実際、今回の試験の主要評価項目は炎症改善に焦点を当てている。また、GLP-1受容体作動薬はすでに大規模臨床試験でCVDを抑制する高いエビデンスが示されているため肝関連事象のみならずCVD抑制も期待される(2)。さらに、糖尿病に限定せず開発が行われていることも重要である。最後に、NAFLDは「木(肝臓)を見て森(全身)を見ず」の診療を行っていては本質的な生命予後改善にはつながらない。これからは「木も見て森も見る」、まさに全身臓器をターゲットとすべき疾患であることを踏まえた新薬開発を行う必要がある。 1. Angulo P, et al. Gastroenterology. 2015;149(2):389-97.e10. 2. Marso SP, et al. N Engl J Med. 2016;375(4):311-322.
原発性高シュウ酸尿症1型に用いるRNAi治療薬lumasiran
原発性高シュウ酸尿症1型に用いるRNAi治療薬lumasiran
Lumasiran, an RNAi Therapeutic for Primary Hyperoxaluria Type 1 N Engl J Med. 2021 Apr 1;384(13):1216-1226. doi: 10.1056/NEJMoa2021712. 原文をBibgraph(ビブグラフ)で読む 上記論文の日本語要約 【背景】原発性高シュウ酸尿症1型(PH1)は、肝臓でシュウ酸が過剰に産生されることによって生じるまれな遺伝性疾患であり、腎結石や腎石灰化症、腎不全、全身性シュウ酸症を引き起こす。開発中のRNA干渉(RNAi)治療薬、lumasiranは、グリコール酸オキシダーゼを標的として肝臓でのシュウ酸の産生を抑制する。 【方法】この第III相二重盲検試験では、6歳以上のPH1患者をlumasiran群とプラセボ群に(2対1の割合で)割り付け、6カ月間皮下投与した(ベースラインと1、2、3、6カ月時に投与)。主要評価項目は、ベースラインから6カ月時までの24時間尿中シュウ酸排泄量の変化率(3~6カ月時までの平均変化率)とした。ベースラインから6カ月時までの血漿中シュウ酸値の変化率(3~6カ月時までの平均変化率)と6カ月時に24時間尿中シュウ酸排泄量が正常範囲上限の1.5倍以下であった患者の割合を副次評価項目とした。 【結果】計39例を無作為化し、26例をlumasiran群、13例をプラセボ群に割り付けた。24時間尿中シュウ酸排泄量の変化率の最小二乗平均差(lumasiran-プラセボ)は-53.5%ポイントであり(P<0.001)、lumasiran群では65.4%低下し、1カ月時に効果が認められた。階層的に検討した全副次評価項目の群間差は有意であった。血漿中シュウ酸値の変化率の差(lumasiran-プラセボ)は-39.5%ポイントであった(P<0.001)。6カ月時の24時間尿中シュウ酸排泄量が正常範囲上限の1.5倍以下であった患者の割合は、lumasiran群84%、プラセボ群0%であった(P<0.001)。lumasiran群の38%に軽度かつ一過性の注射部位反応が報告された。 【結論】lumasiranは、PH1の進行性腎不全の原因となる尿中シュウ酸排泄を抑制した。lumasiranを投与した患者の大多数は、6カ月間の治療後に正常値または正常値に近い値を示した。 第一人者の医師による解説 臓器移植に代わるPH1患者の革新的根治治療薬 他の希少疾患でのRNAi治療薬の開発を期待 笠原 群生 国立成育医療研究センター臓器移植センター長・副院長 MMJ. August 2021;17(4):125 高シュウ酸尿症1型(PH1)は常染色体劣性遺伝疾患で、肝臓のペルオキシソームに局在するアラニン・グリオキシル酸アミノトランスフェラーゼ(AGT)の欠損により、シュウ酸が過剰に産生される疾患である。過剰なシュウ酸はシュウ酸カルシウムとなり、腎結石・腎不全・全身のシュウ酸カルシウム沈着(皮膚、骨、網膜、心血管など)をきたす予後不良の疾患である。発症頻度は10万人に1人~100万人に1人の希少疾患である。小児期に腎結石で発症する患者が多いが、診断が困難で43%の患者が腎不全となってから診断され、14%が15.5歳(中央値)で死亡すると報告されている(1)。根治手術には肝移植が有効であるが、併存する進行性の腎不全により肝腎同時移植が必要な患者もある。 ルマシランはRNA干渉治療薬でAGT上流にあるグリコール酸オキシダーゼをエンコードするmRNAを阻害することで、肝臓でのシュウ酸産生を抑制する。今回の研究は、6歳以上で慢性腎臓病(CKD)ステージ 3以下の遺伝子診断されたPH1患者にルマシランを6カ月間使用し、皮下(3mg/kgを最初の1 ~ 3カ月は月1回、その後3カ月ごとに1回)投与群(26人)とプラセボ群(13人)に割り付け比較検討する、無作為化二重盲検第3相試験として実施された。ルマシラン投与群で推定糸球体濾過量(eGFR)に変化を認めなかったが、24時間尿中シュウ酸排泄量および血漿シュウ酸濃度で有意な低下を認めた。腎結石症状もルマシラン投与群で減少した。ルマシラン投与による主な有害事象は皮下注射部位の発赤・痛み・掻痒感であったが、一過性であった。ルマシランはPH1患者に安全に投与可能で、尿中シュウ酸排泄量を正常値近くまで減少することが可能であった。 PH1患者にはビタミン B6内服や水分摂取などの治療法が試みられてきたが、進行性の腎障害、腎不全、骨病変、眼病変、心機能不全を認めることがあり、肝移植や肝腎移植が適用されてきた。ルマシランは臓器移植に代わるPH1患者の革新的な根治治療薬になりえ、希少疾患患者のアンメット・メディカル・ニーズに応える薬剤である。今後他の希少疾患でRNAi治療薬の基礎的研究・臨床応用が期待される。 1. Mandrile G, et al. Kidney Int. 2014;86(6):1197-1204.
うつ病に用いるpsilocybinとエスシタロプラムを比較した試験
うつ病に用いるpsilocybinとエスシタロプラムを比較した試験
Trial of Psilocybin versus Escitalopram for Depression N Engl J Med. 2021 Apr 15;384(15):1402-1411. doi: 10.1056/NEJMoa2032994. 原文をBibgraph(ビブグラフ)で読む 上記論文の日本語要約 【背景】psilocybinは抗うつ作用を有する可能性があるが、psilocybinと実証済みのうつ病治療との直接比較はない。 【方法】罹病歴の長い中等症ないし重症の大うつ病性障害患者を対象とした第II相二重盲検無作為化比較試験で、psilocybinと選択的セロトニン再取り込み阻害薬エスシタロプラムを6週間にわたって比較した。患者をpsilocybin 25mgの3週間隔2回投与かつプラセボ6週間連日投与群(psilocybin群)とpsilocybin 1mgの3週間隔2回投与かつエスシタロプラム6週間連日経口投与群(エスシタロプラム群)に1対1の割合で割り付けた。全例に心理的サポートを実施した。主要評価項目は、自己報告による簡易抑うつ症状尺度16項目スコア(QIDS-SR-16、スコア範囲0~27点、スコアが高いほど抑うつが重症)のベースラインから6週時までの変化量とした。6週時のQIDS-SR-16の改善(スコア50%以上低下と定義)およびQIDS-SR-16の寛解(スコア5点以下と定義)など16項目を副次評価項目とした。 【結果】59例を組み入れ、30例をpsilocybin群、29例をエスシタロプラム群に割り付けた。ベースラインの平均QIDS-SR-16スコアは、psilocybin群14.5点、エスシタロプラム群16.4点であった。ベースラインから6週時までの平均スコア変化量(±SE)は、psilocybin群-8.0±1.0点、エスシタロプラム群-6.0±1.0点で、群間差は2.0点(95%CI -5.0~0.9、P=0.17)であった。psilocybin群の70%とエスシタロプラム群の48%にQIDS-SR-16の改善が認められ、群間差は22%ポイント(95%CI -3~48)であった。それぞれ57%と28%にQIDS-SR-16の寛解が認められ、群間差は28%ポイント(95%CI 2~54)であった。その他の副次評価項目は概ねpsilocybin群の方がエスシタロプラム群よりも良好であったが、解析では多重比較を補正しなかった。有害事象の発現率は両群で同等であった。 【解釈】6週時のQIDS-SR-16うつ病スコアの変化量を基にすると、この試験では選択した患者群でpsilocybinとエスシタロプラムの抗うつ作用に有意差は認められなかった。副次評価項目は概ねpsilocybinの方がエスシタロプラムよりも良好であったが、この評価項目の解析では多重比較を補正しなかった。psilocybinと検証済みの抗うつ薬を比較するには、大規模で長期的な試験が必要である。 第一人者の医師による解説 シロシビンの効果検証 より大規模で長期の試験が必要 高橋 英彦 東京医科歯科大学大学院医歯学総合研究科精神行動医科学主任教授 MMJ. August 2021;17(4):108 シロシビン(psilocybin)はヒカゲシビレタケなどのマジックマッシュルームに含まれる幻覚成分で、セロトニンと類似した化学構造を有し、セロトニン 5-HT2A受容体にアゴニストとして主として作用する(1)。シロシビンには抗うつ作用があると考えられるが、シロシビンと既存のうつ病治療法との直接的な比較は行われていない。そこで、著者らは第2相二重盲検無作為化対照試験で、中等度〜重度の大うつ病性障害の患者を対象に、シロシビンまたは選択的セロトニン再取り込み阻害薬であるエスシタロプラムを6週間経口投与し、有効性と安全性を比較した。患者は、シロシビン25mgの2回投与(1、4週目)に加えプラセボを6週間連日投与する群(シロシビン群)と、薬理作用が無視できるシロシビン1mgの2回投与(1、4週目)に加えエスシタロプラムを6週間連日投与する群(エスシタロプラム群)に割り付けられた。主要評価項目は、6週目の16項目のQuick Inventory of Depressive Symptomatology-Self-Report(QIDS-SR-16)の得点のベースラインからの変化量であった。副次評価項目として、6週目にQIDS-SR-16の奏効(スコアがベースラインよりも50%以上減少)、QIDS-SR-16の寛解(スコアが5以下になった場合)など16項目を設定した。30人がシロシビン群に、29人がエスシタロプラム群に割り付けられた。ベースライン時のQIDS-SR-16の平均スコアは、シロシビン群で14.5点、エスシタロプラム群で16.4点であった。ベースラインから6週目までのスコアの変化量の平均(± SE)は、シロシビン群で−8.0±1.0ポイント、エスシタロプラム群で−6.0±1.0ポイントとなったが有意差はなかった。QIDS-SR-16上の奏効はシロシビン群で70%、エスシタロプラム群で48%で得られ、QIDS-SR-16上の寛解はそれぞれ57%と28%に認められたが、いずれも有意差はなかった。有害事象の発生率は両群間で同程度であった。本試験の限界として、エスシタロプラムは効果発現にもっと時間がかかる場合もあり、6週間は短い可能性が挙げられる。シロシビン25mgは1mgに比べて高揚感や解放感を感じる人が多く盲検化に影響を与えたかもしれない。結論として、シロシビンを既存の抗うつ薬と比較するには、より大規模で長期の試験が必要である。 1. Madsen MK, et al. Neuropsychopharmacology. 2019;44(7):1328-1334.
肝硬変の入院患者に用いるアルブミン点滴の無作為化試験
肝硬変の入院患者に用いるアルブミン点滴の無作為化試験
A Randomized Trial of Albumin Infusions in Hospitalized Patients with Cirrhosis N Engl J Med. 2021 Mar 4;384(9):808-817. doi: 10.1056/NEJMoa2022166. 原文をBibgraph(ビブグラフ)で読む 上記論文の日本語要約 【背景】非代償性肝硬変患者では、感染や全身性炎症の亢進が臓器障害や死亡の原因となる。前臨床試験の結果から、アルブミンの抗炎症作用が期待されているが、それを検証する大規模臨床試験がない。このような患者で、血清アルブミン値30g/L以上を目標とした20%ヒトアルブミン溶液の連日点滴によって、標準治療と比較して感染症、腎障害および死亡の発生率が低下するかは明らかになっていない。 【方法】組み入れ時の血清アルブミン値が30g/L未満の非代償性肝硬変入院患者を対象に、多施設共同無作為化非盲検並行群間比較試験を実施した。患者を目標値を設定した20%ヒトアルブミン溶液の最長14日間または退院のいずれかまで投与するグループと標準治療を実施するグループに割り付けた。入院後3日以内に治療を開始することとした。複合主要評価項目は、治療開始後3~15日目に発生した新たな感染症、腎機能障害または死亡とした。 【結果】777例を無作為化し、患者の大部分でアルコールが肝硬変の原因であることが報告された。患者1例当たりのアルブミン総投与量中央値は、目標値設定アルブミン群(アルブミン値を30g/Lまで上昇させる)では200g(四分位範囲140~280)、標準治療群では20g(四分位範囲0~120)であった(調整後の平均差143g、95%CI 127~158.2)。主要評価項目が発生した患者の割合は、目標値設定アルブミン群(380例中113例、29.7%)と標準治療群(397例中120例、30.2%)の間で有意差が認められなかった(調整オッズ比0.98、95%CI 0.71~1.33、P=0.87)。データを退院時または15日目で打ち切りとした生存時間解析でも、群間差は認められなかった(ハザード比1.04、95%CI 0.81-1.35)。アルブミン群の方が標準治療群よりも、重度または生命を脅かす重篤な有害事象の発生率が多かった。 【結論】非代償性肝硬変入院患者で、アルブミン値30mg/L以上を目標とするアルブミン点滴に、英国の現行標準治療を上回る有益性は認められなかった。 第一人者の医師による解説 数値目標に固執したアルブミン投与は推奨されず 必要に応じ適切な投与を 岡田 啓 東京大学糖尿病・生活習慣病予防講座特任助教 MMJ. August 2021;17(4):117 アルブミンは、70年以上前から肝硬変患者に対して投与されてきたが、その投与の是非については意見が分かれていた。基礎研究では、アルブミン投与が全身炎症抑制や腎障害リスク低下に寄与することで予後を改善することが示唆されていたが、臨床試験においてはアルブミン投与が一貫して有効だというエビデンスは存在しなかった。 今回報告されたATTIRE試験は、非代償性肝硬変患者における急性合併症で入院した18歳以上の患者を対象とした、多施設共同ランダム化非盲検並行群間比較試験である。入院後72時間以内の血清アルブミン値が3.0g/dL未満、ランダム化時点で5日以上の入院を見込む患者を組み入れた。主な除外基準は、進行肝細胞がんを有し予後8週未満と予測され緩和治療を受けている患者とした。介入群では、非介入群で行う通常治療に加えて、血清アルブミン値3.5g/dLを目標として設定したアルブミン補充を行った。ただし、腹水穿刺で大量排液の施行や突発性細菌性腹膜炎や肝腎症候群を発症した場合、ガイドラインでアルブミン投与が推奨されているため1,2、非介入群でもアルブミン投与を医師の判断で行うこととした。主要エンドポイントは試験開始から3〜15日での感染症・腎機能障害・死亡の複合エンドポイントとし、副次エンドポイントは28日、3カ月、6カ月時点の死亡や合併症発症であった。 結果は、患者777人がランダム化され、intention-to-treat(ITT)解析において、主要エンドポイント発生割合は介入群29.7%、非介入群30.2%であった(調整済みオッズ比,0.98;95%信頼区間 ,0.71〜1.33;P=0.87)。発生したイベントの内訳は、新規感染症、腎機能障害、死亡の順に多く、個々の発症割合に関しても、両群で有意差を認めなかった。副次エンドポイントにおいても、有意なリスク差を認めず、むしろ介入群において肺水腫や細胞外液過剰状態が多く発現した(23人対8人[非介入群])。 本論文の結論は、血清アルブミン値目標を定めたアルブミン投与は推奨されないというものである。しかしその対象は、あくまで、数値目標に固執したアルブミン投与であって、既存の方法でのアルブミン投与を否定するものではないことに注意したい。海外ガイドライン 1,2と同様、日本の「肝硬変診療ガイドライン2020改訂第3版」でも、特発性細菌性腹膜炎や1型肝腎症候群合併例に対して同様にアルブミン製剤投与が推奨されている。なお、本研究はアルブミン数値目標や体液管理困難な患者が対象であるため二重盲検化は難しく、非盲検が正当化される状況であった。 1. European Association for the Study of the Liver. J Hepatol. 2018;69 (2) :406-460. 2. Runyon BA; Hepatology. 2013;57 (4) :1651-1653.
グリセミック指数、グリセミック負荷および心血管疾患と死亡
グリセミック指数、グリセミック負荷および心血管疾患と死亡
Glycemic Index, Glycemic Load, and Cardiovascular Disease and Mortality N Engl J Med. 2021 Apr 8;384(14):1312-1322. doi: 10.1056/NEJMoa2007123. Epub 2021 Feb 24. 原文をBibgraph(ビブグラフ)で読む 上記論文の日本語要約 【背景】グリセミック指数と心血管疾患の関連性に関するデータのほとんどが高所得の欧米諸国の集団から得られたものであり、低所得または中所得の非欧米諸国から得られた情報はほとんどない。この隔たりを埋めるため、地理的に多様な大規模集団からデータを得る必要がある。 【方法】この解析は5大陸に住む35~70歳の参加者13万7,851例を対象とし、中央値9.5年にわたり追跡した。国別の食物摂取頻度調査票を用いて栄養摂取量を求め、炭水化物食品7分類の摂取量に基づきグリセミック指数とグリセミック負荷を推定した。多変量Cox frailty modelを用いてハザード比を算出した。主要評価項目は、主要心血管事象(心血管死、非致命的心筋梗塞、脳卒中、心不全)または全死因死亡の複合とした。 【結果】対象集団では、追跡期間中に死亡8,780件、主要心血管事象8,252件が発生した。グリセミック指数の最低五分位群と最高五分位群を比較する広範な調整を実施した後、心血管疾患の既往がある参加者(ハザード比1.51;95%CI、1.25~1.82)とない参加者(ハザード比1.21;95%CI、1.11~1.34)ともにグリセミック指数の高い食事で主要心血管事象または死亡のリスクが上昇することが明らかになった。主要評価項目の構成要素のうち、グリセミック指数高値には心血管死のリスク上昇との関連も認められた。グリセミック負荷に関する結果は、ベースラインで心血管疾患がある参加者ではグリセミック指数の結果とほぼ同じであったが、この関連は心血管疾患がない参加者では有意ではなかった。 【結論】本試験では、グリセミック指数の高い食事に心血管疾患および死亡のリスク上昇との関連が認められた。 第一人者の医師による解説 食事が健康に与える影響の解析において GIとGLは有用な指標 宮下 和季 慶應義塾大学医学部腎臓内分泌代謝内科特任准教授 MMJ. October 2021;17(5):159 グリセミックインデックス(GI)は、ある食品を摂取した後の血糖上昇を示す指標である。ブドウ糖50gを摂取した後の血糖値 -時間曲線下面積(AUC)を100として、一定量の炭水化物(50g)を含む食品を摂取した際のAUCがGIと定義される。食後血糖が上昇しやすく長時間高血糖が持続する食品はGI値が高くなり、一般には炭水化物の中でブドウ糖を多く含む食品が高 GIとなる。糖尿病の食事管理におけるGIの有用性が1998年に報告され、食品の質を表す指標としてGIが注目されるようになった。グリセミックロード(GL)は実際に食べる量を考慮した糖負荷の指標で、GI値に各食品の炭水化物量をかけ合わせて算出する。食事が血糖値に与える影響の予測指標として、摂取カロリーを勘案するGLは、GIよりも有用と考えられている。 血糖に影響を与えにくい低 GI食により糖尿病の発症リスクが低下するとの報告があるものの、心血管病の発症や全死亡に与える影響についてはエビデンスに乏しい。GI値と心血管病の関連に関するデータは、欧米諸国から得られたものであり(1),(2)、発展途上国のデータはほとんどない。そこで本研究では、5大陸20カ国に住む35~70歳の14万人弱を対象に、中央値9.5年の追跡期間で、GI値、GL値と、主要心血管イベントの発症(心血管死、非致死的心筋梗塞、脳卒中、心不全)、全死亡との関連を検討した。質問票を用いて食事摂取量を決定し、炭水化物を7つのカテゴリーに分類してGI値とGL値を推定した。多変量 Cox解析により、GI値とGL値が心血管病および全死亡に与える影響を算出した。 GI値、GL値により参加者を5段階に分けて、広範な調整を行い検討したところ、高 GI食は、心血管病の既往や体格指数(BMI)にかかわらず、世界のどの地域においても、心血管病と全死亡のリスクを高めた。心血管病の既往のある母集団では、高GI食に伴う心血管病発症または死亡のハザード比は1.51(95%信頼区間[CI], 1.25~1.82)であった。GL値も心血管病および全死亡のリスクと同程度に関連していたが、心血管病の既往のない参加者では関連性が有意ではなかった。 以上の結果より、これまでほとんど検討されていなかった発展途上国においても、高 GI食が心血管病および全死亡のリスクとなることが明らかとなった。このことからGIとGLは、食事が健康に与える影響の解析において、有用な指標と判断された。高 GI / GL食を摂取すると、低 GI / GL食よりも、心血管病と全死亡のリスクが高まることが示された。 1. Levitan EB, et al. Am J Clin Nutr. 2007;85(6):1521-1526. 2. Nagata C, et al. Br J Nutr. 2014;112(12):2010-2017.
栄養不良の小児への細菌叢を標的とした食事介入
栄養不良の小児への細菌叢を標的とした食事介入
A Microbiota-Directed Food Intervention for Undernourished Children N Engl J Med. 2021 Apr 22;384(16):1517-1528. doi: 10.1056/NEJMoa2023294. Epub 2021 Apr 7. 原文をBibgraph(ビブグラフ)で読む 上記論文の日本語要約 【背景】世界で3,000万人以上の小児が中等度急性栄養不良の状態にある。現在の治療は効果に乏しく、この病態の原因について明らかになっていない点も多い。中等度急性栄養不良の小児では、腸内細菌叢の発達が阻害されている。 【方法】この試験では、スラムに居住し中等度急性栄養不良がある12~18カ月齢のバングラデシュ人小児123例に、細菌を標的とした補完食のプロトタイプ(microbiota-directed complementary food prototype:MDCF-2)またはすぐに食べられる栄養補助食(ready-to-use supplementary food:RUSF)を提供した。補充は3カ月間にわたり1日2回実施し、その後1カ月間モニタリングした。介入開始前、介入期間中の2週に1回および4カ月時、身長に対する体重、年齢に対する体重、年齢に対する身長のzスコアを求め、上腕中央部周囲長を計測した。介入開始前と3カ月時、介入開始前と4カ月時で、この関連表現型の変化率を比較した。このほか、タンパク質4,977種の血漿濃度と糞便中の細菌群209種の量を測定した。 【結果】小児118例(各試験群59例)が介入を完遂した。身長に対する体重と年齢に対する体重のzスコアの変化率は、1カ月間の追跡調査を含む試験期間中、MDCF-2が成長にもたらす便益と一致していた。MDCF-2の摂取に、血漿タンパク質70種の濃度と関連細菌群21種の量に見られた変化の度合いとの関連が認められ、この変化の度合いには身長に対する体重のzスコアと正の相関が認められた(タンパク質と細菌群、いずれの比較でもP<0.001)。このタンパク質には、骨成長や神経発達の媒介因子が含まれていた。 【結論】この結果は、中等度急性栄養不良の小児に用いる栄養補助食としてMDCF-2を支持し、細菌叢の構成要素を標的とした操作が小児の成長につながる機序を知る上での手掛かりをもたらすものである。 第一人者の医師による解説 危険性がなくコスト面でも有利 適応年齢や継続的な投与の必要性が今後の検討課題 金森 豊 国立成育医療研究センター小児外科系専門診療部外科診療部長 MMJ. October 2021;17(5):157 本論文の臨床試験には先行研究(1)があり、腸内細菌叢の成熟が遅れた低栄養児に対して、腸内細菌叢を正常栄養児の年齢相応の状態に誘導する補助栄養剤MDCF(microbiota-directed complementary food)の開発が報告されている。今回はそのうちの1製剤(MDCF-2)の有効性が従来の補助栄養剤(ready-to-use supplementary food;RUSF)と無作為化対照試験で比較された。バングラデシュ・ダッカのミルプール地域在住の中等度の低栄養児(身長体重比がコホート中央値の−2〜−3SD未満)118人がMDCF-2またはRUSFを3カ月間投与され、投与終了後1カ月間追跡された。その結果、MDSF-2群では、RUSFと比較し、身長体重比と年齢体重比のz-スコアが増加傾向を示した。また、身長体重比の上昇と変化量が有意な正の相関を示す蛋白として、骨成長および中枢神経系の成長に関連する蛋白が特定された。これらの蛋白群はMDCF-2群で増加が顕著であった。また、腸内細菌叢の変化から、身長体重比のz-スコアと正の相関を示す21種の細菌が特定された。これらはMDCF-2群で有意に増加しており、正常発達している児の腸内細菌叢に近づいていることが示唆された。一方、RUSF群では有意な変化がみられなかった。 腸内細菌叢の異常がさまざまな疾患と関連していることは最近多くの研究が示しており、低栄養の改善に腸内細菌叢が関与していることは想像に難くない。実際、低栄養の改善を目的に腸内細菌叢をコントロールしようという研究は活発である。1つの方法論は、プロバイオティクスという概念に包含される、宿主に有用な細菌の腸管内投与である。古くはビフィズス菌や乳酸菌などの単独投与が行われたが、最近では糞便移植や有用と考えられるいくつかの細菌を選別して投与する方法などが脚光を浴びている。しかし、有用菌の選別や有害菌の除去などに問題があり、コストの面からも難しい側面を持つ。そこで今回の報告のような、腸内細菌叢を宿主にとって有利な方向に誘導する栄養補助食品(プレバイオティクスと呼んでもいい方法論)の開発が注目される。この方法は大量に菌を投与するような危険性がなく、コストの面でも有利で、発展途上国などに多い低栄養児に応用するには有利である。本研究もそのような利点を十分に考慮に入れた研究で、今後のさらなる発展が期待される報告である。一方、この方法論が世界的にどの地域でも通用するかどうか、また適応年齢の制限がないかどうか、継続的な投与が必要かどうか、など今後検討するべき点も多い。 1. Gehrig JL, et al. Science. 2019;365(6449):eaau4732.
転移性トリプルネガティブ乳がんに用いるsacituzumab govitecan
転移性トリプルネガティブ乳がんに用いるsacituzumab govitecan
Sacituzumab Govitecan in Metastatic Triple-Negative Breast Cancer N Engl J Med. 2021 Apr 22;384(16):1529-1541. doi: 10.1056/NEJMoa2028485. 原文をBibgraph(ビブグラフ)で読む 上記論文の日本語要約 【背景】転移性トリプルネガティブ乳がんは予後が不良である。sacituzumab govitecanは、乳がんの大多数に発現するヒト栄養膜細胞表面抗原2(Trop-2)を標的とする抗体とSN-38(トポイソメラーゼI阻害薬)を特許取得済みの加水分解性リンカーで結合させた抗体薬物複合体である。 【方法】この第III相無作為化試験では、再発または難治性の転移性トリプルネガティブ乳がんを対象に、sacituzumab govitecanと医師が選択した単剤化学療法(エリブリン、ビノレルビン、カペシタビン、ゲムシタビンのいずれか)を比較した。主要評価項目は、脳転移のない患者の無増悪生存期間(盲検下の独立中央判定委員会が判定)とした。 【結果】脳転移がない患者468例をsacituzumab govitecan群(235例)、化学療法群(233例)に無作為化により割り付けた。年齢中央値は54歳であり、全例にタキサン系薬剤使用歴があった。無増悪期間中央値は、sacituzumab govitecan群が5.6カ月(95%CI、4.3~6.3、166件)、化学療法群が1.7カ月(95%CI、1.5~2.6、150件)であった(進行または死亡のハザード比、0.41;95%CI、0.32~0.52;P<0.001)。総生存期間中央値は、sacituzumab govitecan群が12.1カ月(95%CI、10.7~14.0)、化学療法群で6.7カ月(同5.8~7.7)であった(死亡のハザード比、0.48;95%CI、0.38~0.59、P<0.001)。 客観的奏効率はsacituzumab govitecan群が35%、化学療法群が5%であった。グレード3以上の特記すべき治療関連有害事象は、好中球減少症(sacituzumab govitecan群51%、化学療法群33%)、白血球減少症(10%、5%)、下痢(10%、1%未満)、貧血(8%、5%)、発熱性好中球減少症(6%、2%)であった。有害事象に起因する死亡が各群3例あったが、sacituzumab govitecanと関連があると判断したものはなかった。 【結論】転移性トリプルネガティブ乳がんで、sacituzumab govitecan群の無増悪生存期間および総生存期間が単剤化学療法群よりも有意に長かった。sacituzumab govitecan群の方が骨髄抑制と下痢が多かった。 第一人者の医師による解説 前治療を伴う転移性トリプルネガティブ乳がんに対する単剤での有意な予後延長 井本 滋 杏林大学医学部乳腺外科教授 MMJ. October 2021;17(5):149 ホルモン受容体陰性・HER2陰性のいわゆるトリプルネガティブ乳がん(TNBC)は予後不良である。遠隔転移を伴わない浸潤がんでは、術前後のアントラサイクリン系およびタキサン系薬剤を用いた薬物療法が標準治療である。転移再発時は残りの化学療法やPD-L1陽性免疫細胞浸潤を伴う腫瘍には抗 PD-L1抗体アテゾリズマブ+ナノ粒子アルブミン結合パクリタキセルの免疫化学療法が選択されるが、奏効例は限定的である。 サシツズマブゴビテカン(SG)は、トロフォブラスト細胞表面抗原2(Trop-2)を標的とするIgG1抗体とイリノテカンの活性代謝物であるSN-38を加水分解性リンカーで結合させた抗体薬物複合薬である(1)。既治療の転移性 TNBC患者108人を対象とした第1/2相試験では、奏効率は33%で奏効期間の中央値は7.7カ月であった。本論文はその第3相試験(ASCENT)の結果報告である。対象はタキサン系薬剤を含む2レジメン以上が実施された進行再発 TNBCである。進行していない脳転移例も登録されたが、主要評価項目である無増悪生存期間(PFS)の解析からは除かれた。SG群と化学療法群(エリブリン、ビノレルビン、カペシタビン、またはゲムシタビンの単剤投与)における有効性が比較された。その結果、脳転移を伴わないTNBC患者468人(年齢中央値 54歳)が登録され、SG群は235人、化学療法群は233人であった。PFS中央値はSG群が5.6カ月、化学療法群が1.7カ月であった(ハザード比[HR], 0.41;P<0.001)。全生存期間の中央値はそれぞれ12.1カ月と6.7カ月であった(HR, 0.48;P<0.001)。奏効率はSG群で35%、化学療法群で5%であった。グレード3以上の治療に関連する有害事象の発現率は、それぞれ好中球減少が51%と33%、白血球減少が10%と5%、下痢が10%と1%未満、貧血が8%と5%、発熱性好中球減少が6%と2%であった。有害事象に伴う死亡が各群で3人発生したが、SGに関連する死亡はなかった。サブグループ解析では、前治療におけるPD-1またはPD-L1阻害薬使用の有無、肝転移の有無、前治療の数に関わらず、SG群で化学療法群に比べPFSが改善した。以上から、標準治療が実施された転移性 TNBCでは、骨髄抑制や下痢が高頻度であるものの、SG単剤による有意な生命予後の延長が示された。 1. Bardia A, et al. N Engl J Med. 2019;380(8):741-751.
早期アルツハイマー病に用いるdonanemab
早期アルツハイマー病に用いるdonanemab
Donanemab in Early Alzheimer's Disease N Engl J Med. 2021 May 6;384(18):1691-1704. doi: 10.1056/NEJMoa2100708. Epub 2021 Mar 13. 原文をBibgraph(ビブグラフ)で読む 上記論文の日本語要約 【背景】アルツハイマー病の特徴にアミロイドβ(Aβ)の蓄積がある。沈着したAβの修飾部位を標的とする抗体、donanemabは、早期アルツハイマー病の治療薬として開発中である。 【方法】陽電子放出断層(PET)でタウとアミロイドの沈着を認める早期症候性アルツハイマー病患者を対象にdonanemabを検討する第II相試験を実施した。患者をdonanemab(初めの3回は700mg、その後1400mg)とプラセボに1対1の割合で割り付けた(いずれも4週間に1回、最長72週間投与)。主要評価項目は、治療前から76週時までのIntegrated Alzheimer’s Disease Rating Scale(iADRS;範囲、0-144点、低スコアほど認知障害および機能障害が重い)の変化量とした。Clinical Dementia Rating Scale-Sum of Boxes(CDR-SB)、Alzheimer’s Disease Assessment Scaleの13項目認知下位尺度(ADAS-Cog13)、Alzheimer’s Disease Cooperative Study-Instrumental Activities of Daily Living Inventory(ADCS-iADL)、Mini-Mental State Examination(MMSE)のスコア変化量およびPET画像で確認したアミロイドとタウの蓄積の変化量を副次評価項目とした。 【結果】計257例を組み入れ、131例をdonanemab、126例をプラセボに割り付けた。治療前のiADRSスコアは両群ともに106点であった。76週時のiADRSスコア変化量は、donanemab群-6.86点、プラセボ群-10.06点であった(差3.20点;95%CI、0.12~6.27;P=0.04)。副次評価項目のほとんどの結果に大きな差はなかった。76週時、アミロイド斑および脳内タウ蓄積量の減少量は、donanemabの方がプラセボよりもそれぞれ85.06センチロイド、0.01大きかった。donanemab群にアミロイド関連の脳浮腫(ほとんどが無症状)が見られた。 【結論】早期アルツハイマー病で、donanemabによる認知機能の複合スコアおよび日常生活動作能力の改善度がプラセボより良好であったが、副次評価項目の結果に大きな差が見られなかった。アルツハイマー病に用いるdoanemabの有効性と安全性を検証するため、大規模で長期間にわたる試験が必要である。 第一人者の医師による解説 現在治験中のガンテネルマブ、レカネマブと合わせて 今後の展開に注目 岩坪 威 東京大学大学院医学系研究科神経病理学分野教授・国立精神・神経医療研究センター神経研究所長・J-ADNI主任研究者 MMJ. October 2021;17(5):140 アミロイドβ(Aβ)の凝集・蓄積はアルツハイマー病(AD)の主要な病因の1つと考えられており、Aβを標的とする疾患修飾療法(DMT)が注目されている。本論文は、ヒト化抗Aβ抗体ドナネマブの早期ADに対する無作為化プラセボ対照第2相試験(TRAILBLAZER-ALZ)の結果を報告している。早期ADとは「軽度認知障害(MCI)」期と「軽症認知症」期のADを合わせた区分である。ドナネマブは蓄積したAβに特異的に生じるピログルタミル化修飾を認識し、高いAβ除去能を示す。 本試験では、北米56施設において60〜85歳の早期 AD患者にドナネマブまたはプラセボが4週ごとに72週間静脈内投与された。ドナネマブは投与4回目から1,400mgに増量されたが、アミロイドPETでAβ陰性化が確認された場合、減量ないし休薬された。主要評価項目である76週時点のiADRSスコアのベースラインからの変化量(増悪)は、プラセボ群の10.06に対し、ドナネマブ群では6.86と、32%の有意な症状進行遅延効果が認められた。アミロイド PET陰性化率は25週で40%、52週で59.8%、76週で67.8%であった。Aβ除去抗体に共通の有害事象である一過性の局所性脳浮腫(ARIA-E)はドナネマブ群の26.7%に生じた。これらの結果を踏まえ、後続のTRAILBLAZER-ALZ2試験が第3相試験に拡大されて、2021年中に米国食品医薬品局(FDA)への申請が予定されている。さらに、より早期の無症候期であるプレクリニカルADを対象とするTRAILBLAZER-ALZ3試験の開始が公表されている。 早期ADに対する抗Aβ抗体療法としては、アデュカヌマブの第3相試験 ENGAGEとEMERGEも報告されている。両試験は無益性の予測から早期終了したが、EMERGE試験ではアデュカヌマブにより78週時点で主要評価項目Clinical Dementia Rating Scale Sum of Boxesに22%の改善が得られ、PETでは両試験ともにAβ除去効果が認められた。これらの結果に基づき2021年6月、FDAはアデュカヌマブを迅速承認した。ドナネマブ、アデュカヌマブの試験結果を考え合わせると、MCIから軽症認知症期という有症状期でもAβ除去によりADの臨床症状の進行が抑制できる可能性が示唆される。また、十分なAβ除去達成後に休薬しても、一定期間にわたってAβ再蓄積や臨床症状の増悪が抑えられることが示されれば、治療期間の短縮による医療費節減も期待できよう。ドナネマブの開発では、アミロイド・タウPETや血漿リン酸化タウなどのバイオマーカーを患者選択、薬効評価に導入したことにより、先発の同類抗体医薬に肉薄する状況にある。ガンテネルマブ、レカネマブなど現在治験中の抗Aβ抗体薬と合わせて、今後の展開が注目される。 iADRS:Integrated Alzheimer's Disease Rating Scale(認知機能尺度 ADAS-Cog13 と日常生活機能尺度 ADCS-iADL の複合スコア)
反復性急性中耳炎に用いる鼓膜換気チューブ留置術と薬物療法の比較
反復性急性中耳炎に用いる鼓膜換気チューブ留置術と薬物療法の比較
Tympanostomy Tubes or Medical Management for Recurrent Acute Otitis Media N Engl J Med. 2021 May 13;384(19):1789-1799. doi: 10.1056/NEJMoa2027278. 原文をBibgraph(ビブグラフ)で読む 上記論文の日本語要約 【背景】反復性急性中耳炎の小児に用いる鼓膜換気チューブ留置について、公式な推奨事項が一致していない。 【方法】6カ月以内に3回以上急性中耳炎を発症したか、12カ月以内に4回以上急性中耳炎を発症し、そのうち1回以上が6カ月以内の発症であった6~35カ月齢の小児を鼓膜換気チューブ留置術群と発症時に抗菌薬を投与する薬物療法群に無作為化により割り付けた。主要評価項目は、2年間の1人年当たりの急性中耳炎平均発症回数(発症率)とした。 【結果】主解析のintention-to-treat解析の結果、2年間の1人年当たりの急性中耳炎発症率(±SE)は、鼓膜チューブ群1.48±0.08、薬物療法群1.56±0.08であった(P=0.66)。鼓膜換気チューブ群の10%が鼓膜換気チューブ留置術を受けず、薬物療法群の16%が親の要望で鼓膜換気チューブ留置術を受けたため、per-protocol解析を実施すると、対応する発症率はそれぞれ1.47±0.08、1.72±0.11となった。主解析の副次評価項目で、結果にばらつきが見られた。急性中耳炎初回発症までの期間、発症に伴う臨床所見、事前に定めた治療失敗の基準を満たす患児の割合は、鼓膜換気チューブの方が良好であった。耳漏が見られた日数の累積は、薬物治療の方が良好であった。大きな差が認められなかった項目に、急性中耳炎発症頻度の分布、重症と考えられた急性中耳炎の割合、呼吸器分離菌の抗菌薬耐性があった。試験関連の有害事象は、試験の副次評価項目に含まれるもののみであった。 【結論】反復性急性中耳炎がある6~35カ月齢の小児で、2年間の急性中耳炎発症率は、鼓膜換気チューブ留置術と薬物療法で有意な差がなかった。 第一人者の医師による解説 耳鼻咽喉科医と小児科医で推奨度の違う鼓膜チューブ留置術 適応については再度見直しが必要 神崎 晶 慶應義塾大学医学部耳鼻咽喉科専任講師 MMJ. October 2021;17(5):158 鼓膜チューブ留置術は、新生児期以降の小児では頻繁に行われる手術であり、全身麻酔のリスク、留置チューブの閉塞、体外への逸脱、中耳腔への落下、鼓膜構造的変化穿孔残存や軽度伝音難聴の可能性がある。 米国では反復性(再発性)急性中耳炎の小児に対する鼓膜チューブ留置術に関する推奨が耳鼻咽喉科医向けと小児科医向けで異なっており(耳鼻咽喉科医の方が本施術に積極的である)、確実なエビデンスに乏しいことから、著者らは本試験により鼓膜チューブ留置術の有効性を検討した。対象は年齢が生後6〜35カ月で、(1)6カ月以内に急性中耳炎のエピソードが3回以上、または(2)12カ月以内に急性中耳炎のエピソードが4回以上あり、そのうち少なくとも1回は6か月以内に発生していた小児であった。対象児を鼓膜チューブ留置術もしくは抗菌薬による薬物療法群にランダムに割り付けた。その結果、2年の経過観察期間における人・年あたりの急性中耳炎の平均エピソード数(率)(±SE)は、鼓膜チューブ留置群1.48±0.08、薬物療法群1.56±0.08であり(P=0.66)、有意差はなかった。 小児の反復性急性中耳炎に対する鼓膜チューブ留置術の適応は、米国の耳鼻咽喉科ガイドラインでは「中耳の滲出液が少なくとも片耳に存在する場合」としているが、小児科ガイドラインでは「臨床医が提供しても良い選択肢の1つ」としており、推奨度に差がある。日本の「小児急性中耳炎診療ガイドライン2018年版(金原出版)」のCQ3-9(P73-75)では、有効とする論文と生活の質(QOL)に寄与しないとする論文もあり、限定的な効果としている。このように、これまであいまいな点が多かったが、今回の論文で、鼓膜チューブ留置術の適応については再度見直しを要することとなる。 なお、小児では、鼻と耳をつなぐ耳管が太くて短いことから反復性急性中耳炎や滲出性中耳炎の原因として、アレルギー性鼻炎や副鼻腔炎などの鼻疾患との関連性が指摘されている。本報告では、各群におけるアレルギー性鼻炎を含む割合については触れられておらず、この点について検討の余地がある。また、鼓膜チューブ留置術は反復性急性中耳炎以外に、滲出液が中耳に貯留して難聴をきたす滲出性中耳炎に対して行う場合の方が多いが、本結論が滲出性中耳炎に対しても同様に当てはまるのかどうかは今後の研究が待たれる。
院外心停止後の低体温と正常体温の比較
院外心停止後の低体温と正常体温の比較
Hypothermia versus Normothermia after Out-of-Hospital Cardiac Arrest N Engl J Med. 2021 Jun 17;384(24):2283-2294. doi: 10.1056/NEJMoa2100591. 原文をBibgraph(ビブグラフ)で読む 上記論文の日本語要約【背景】心停止後の患者に体温管理療法が推奨されているが、それを裏付ける根拠は確実性が低い。【方法】盲検下で転帰を評価する試験で、心原性または原因不明の院外心停止を起こし蘇生後に昏睡状態に陥った成人患者1,900例を33℃の低体温目標(その後制御下で復温)と正常体温目標(37.8℃以上の発熱を早期に治療)に無作為化により割り付けた。主要評価項目は、6カ月時の全死因死亡とした。6カ月時の機能的転帰を副次評価項目とし、修正Rankin尺度で評価した。性別、年齢、初期心調律、自己心拍再開までの時間、入院時のショックの有無に従って部分集団を事前に定義した。肺炎、敗血症、出血、血行動態を損なう不整脈、体温管理装置による皮膚合併症を有害事象とした。【結果】計1,850例で主要評価項目を評価した。6カ月時点で、低体温群925例中465例(50%)が死亡していたのに対して、正常体温群では925例中446例(48%)が死亡していた(低体温群の相対リスク、1.04;95%CI、0.94~1.14;P=0.37)。機能的転帰を評価した1,747例のうち、中等度以上(修正Rankin尺度スコア4点以上)の障害が認められたのは、低体温群では881例中488例(55%)であったのに対し、正常体温群では866例中479例(55%)であった(低体温群の相対リスク、1.00;95%CI、0.92~1.09)。事前に定めた部分集団でも転帰が一定であった。低体温群の方が正常体温群よりも血行動態を損なう不整脈の発現率が高かった(24% vs. 17%、P<0.001)。その他の有害事象の発現率に両群間で有意差は認められなかった。【結論】院外心停止後の昏睡患者で、低体温療法により6カ月後までの死亡率が常温療法より改善することはなかった。 第一人者の医師による解説 心停止蘇生後の体温管理 上昇させないことが大切 櫻井 淳 日本大学医学部救急医学系救急集中治療医学分野診療教授 MMJ. December 2021;17(6):187 心停止蘇生後の脳障害に対する治療として、2002年に蘇生後の低体温管理群は体温管理をしない群より神経学的転帰が有意に良かったと報告され(1)、2013年には体温管理目標で常温(36℃)と低体温(33℃)で転帰に差がないと報告された(2)。心停止蘇生後は体温管理療法(targetedtemperaturemanagement;TTM)が必要であるが、最適な目標温度に関してはいまだ議論中である。本論文はこの最適な体温決定のための国際共同多施設ランダム化比較試験の報告である。対象は18歳以上の成人で、心原性か心停止の原因が不明の院外心停止蘇生後で、従命反応がない患者であった。低体温群と常温群で死亡率や機能的な回復に関して比較が行われた。低体温群では割り付け後直ちに33℃まで体温を低下させ、28時間維持した後に3時間に1℃で37℃まで復温した。常温群では37.5℃以下を維持するように薬剤を用いて40時間体温を管理し、37.8℃以上となった際には冷却器具で体温を低下させた。両群とも、TTM終了後に割り付け時点から72時間までは36.5~37.7℃の常温を維持した。1,850人の生命予後が比較され、低体温群の死亡率は50%(465/925人)であり、正常体温群の48%(446/925人)と比較し有意差はなかった(相対リスク[RR],1.04;95%信頼区間[CI],0.94?1.14;P=0.37)。1,747人の機能的予後が評価され、低体温群では55%(488/881人)に中等度の機能障害以上の障害があり、常温群では55%(479/866人)と差はみられなかった(RR,1.00;95%CI,0.92?1.09)。性別、年齢、心停止時間、初期心電図波形といったサブグループ解析でも両群間に差はなかった。一方、低体温群では常温群に比べ合併症として循環に影響のある不整脈が多かった(24%対17%;P<0.001)。本論文の出版前に出された日本蘇生協議会(JRC)蘇生ガイドライン2020では、TTM実施時は32~36℃の間で目標体温を設定し一定期間維持することが推奨されている。本ガイドラインではTTMの精度として、目標体温への到達時間や維持期の厳格な体温管理の重要性が述べられており(3)、本検討に関しこの部分で批判的な意見もある。また、脳障害の程度によっては低体温療法の方が有効である群が存在する可能性も否定できない。ただ、本試験は十分な患者数の検討であり、GRADEシステムによる今後のガイドライン作成には大きな影響があると考えられる。今回の結果を踏まえると、心停止蘇生後の集中治療でのTTMでは正常体温より上昇させないことが重要であるといえる。 1.Hypothermia after Cardiac Arrest Study Group. N Engl J Med.2002;346(8):549-556. 2.Nielsen N, et al. N Engl J Med. 2013;369(23):2197-2206. 3. 一般社団法人日本蘇生協議会.JRC 蘇生ガイドライン 2020. 医学書院 .2021.
アデノシンデアミナーゼ欠損症に用いる自家ex vivoレンチウイルス遺伝子治療
アデノシンデアミナーゼ欠損症に用いる自家ex vivoレンチウイルス遺伝子治療
Autologous Ex Vivo Lentiviral Gene Therapy for Adenosine Deaminase Deficiency N Engl J Med. 2021 May 27;384(21):2002-2013. doi: 10.1056/NEJMoa2027675. Epub 2021 May 11. 原文をBibgraph(ビブグラフ)で読む 上記論文の日本語要約【背景】アデノシンデアミナーゼ(ADA)欠損による重症複合免疫不全症(ADA-SCID)は、生命を脅かすまれな原発性免疫不全症である。【方法】ADA-SCID 50例(米国で30例、英国で20例)にヒトADA遺伝子をコードする自己不活型レンチウイルスベクターを用いて体外で(ex vivo)形質導入した自己CD34+造血幹細胞および前駆細胞(HSPC)による開発中の遺伝子治療を実施した。24カ月間追跡した2つの米国試験のデータ(新鮮細胞および凍結保存細胞使用)を36カ月追跡した英国試験のデータ(新鮮細胞使用)と併せて解析した。【結果】最長24カ月間および36カ月間の全試験の総生存率が100%であった。(酵素補充療法の再開や救済療法としての同種造血幹細胞移植がない)無事象生存率は、12カ月時で97%(米国の試験)と100%(英国の試験)、24カ月時でそれぞれ97%と95%、36カ月時で95%(英国の試験)であった。米国試験の30例中29例、英国試験の20例中19例が遺伝子組み換えHSPCの生着を維持した。代謝解毒作用とADA活性値の正常化が保たれていた。免疫再構築が達成され、米国試験の90%および英国試験の100%がそれぞれ24カ月時および36カ月時までに免疫グロブリン補充療法を中止した。単クローン性増殖や白血球増殖性の合併症、複製可能なレンチウイルス出現の根拠は認めず、自己免疫疾患または移植片対宿主病の事象は発現しなかった。有害事象の大部分は低グレードであった。【結論】ex vivoレンチウイルスHSPC遺伝子治療でADA-SCIDを治療した結果、総生存率および無事象生存率が高く、ADA発現の持続、代謝補正および機能的な免疫再構築を認めた。 第一人者の医師による解説 レンチウイルスベクター遺伝子治療 第1選択のHSCTより安全性、有効性で優れる 有賀 正 社会医療法人 母恋 理事長 MMJ. December 2021;17(6):184 アデノシンデアミナーゼ(ADA)欠損症は重症複合免疫不全症(SCID)の一型で、従来から遺伝子治療の標的疾患として注目されていた。1990年、初の遺伝子治療臨床研究がこの疾患で実施され(1)、さまざまな遺伝性疾患へも遺伝子治療が応用されていった。しかし、2002年、X-SCIDに対するレトロウイルスベクター遺伝子治療で白血病様副作用が重ねて報告された(2)。これを契機に遺伝子治療の安全性が再強化され、現在、より安全で効果のある遺伝子治療が展開している。今回の報告は米国と英国で実施された異なる3つの遺伝子治療臨床研究「ADA欠損症に対するexvivoレンチウイルスベクター遺伝子治療」の結果を示している。ベクターはEFS-ADALVを共通して使用したが、標的細胞の採取方法、導入細胞の処理、移植細胞数など多くの点で異なっていた。対象はドナー不在のADA欠損症患者50人(米国30人、英国20人[5人は5歳超、このうち3人は10歳以上])で、自己の骨髄由来、または末梢血CD34+細胞が遺伝子導入標的である。必要最低標的細胞数は1x106/kg~4x106/kgであった。標的細胞採取後に、非骨髄破壊的量のブスルファンによる前処置を実施した。ADA酵素補充は診断直後から遺伝子治療終了後30日まで続けられた。生存率は100%(50/50)であった。2人が生着不良(1人は血液幹細胞移植[HSCT]を実施[米国]、1人は酵素補充を再開・継続[英国])で、無イベント生存率は96%(48/50)であった。この2人を除いた全例で、顆粒球、単核球で導入遺伝子が確認され、3カ月後には赤血球中のADA酵素活性の上昇と毒性代謝産物の著減を認めた。T細胞とそのサブセットは一時減少したが、漸次増加し、増加は継続している。ナイーブT細胞、T細胞特異マーカーのTcellreceptorexcisioncircle(TREC)は上昇し、T細胞の新生を示している。B細胞、NK細胞は一時減少も、正常レベルとなった。血清IgA、IgM値は上昇、ほとんどの患者で免疫グロブリン補充が中止できている。また、ワクチンへの反応も確認された。末梢血リンパ球数は正常レベルである。重症感染症の頻度は低く、コントロールできた。注目すべき合併症の免疫再構築炎症症候群が4人にみられたが、ステロイドが奏効した。レトロウイルスベクター遺伝子治療時に認めた白血病様副作用は認めず、増殖性ウイルスも検出されていない。ADA欠損症に対する第1選択治療はこれまでHLA一致ドナーからのHSCTであったが、最近では遺伝子治療もこれと同等と考えられている。今回のレンチウイルスベクター自己血液幹細胞遺伝子治療の結果は、従来のHSCTよりも安全性、有効性に優れていた。 1. Blaese RM et al. Science. 1995; 270(5235):475-480. 2. Hacein-Bey-Abina S, et al. New Engl J Med. 2003; 348(3): 255-256.
再発または難治性の有毛細胞白血病に用いるベムラフェニブとリツキシマブの併用
再発または難治性の有毛細胞白血病に用いるベムラフェニブとリツキシマブの併用
Vemurafenib plus Rituximab in Refractory or Relapsed Hairy-Cell Leukemia N Engl J Med. 2021 May 13;384(19):1810-1823. doi: 10.1056/NEJMoa2031298. 原文をBibgraph(ビブグラフ)で読む 上記論文の日本語要約【背景】有毛細胞白血病(HCL)は、悪性度の低いCD20陽性無痛性B細胞性悪性腫瘍であり、BRAF V600Eキナーゼ活性化変異が病原性の役割を演じている。再発または難治性のHCL患者を対象とした臨床試験で、経口BRAF阻害薬ベムラフェニブでBRAF V600Eを標的とすることで、患者の91%が奏効、35%が完全寛解を得た。しかし、治療中止後の無再発生存期間は中央値でわずか9カ月であった。【方法】この再発または難治性HCL患者を対象とした公益機関単施設第II相試験では、ベムラフェニブ(960mg、1日2回8週間投与)とリツキシマブ(体表面積1m2当たり375mg、18週間で8回投与)の同時および逐次投与の安全性および有効性を評価した。主要評価項目は、計画した治療終了時点の完全奏効とした。【結果】登録したHCL患者30例の前治療数は中央値で3回であった。intention-to-treat(ITT)解析集団では、26例(87%)が完全寛解を得た。化学療法(10例)またはリツキシマブ(5例)に抵抗性を示した全HCL患者およびBRAF阻害薬による治療歴がある全患者(7例)が完全寛解を得た。中央値で2週間後に血小板減少症が解消し、中央値で4週間後に好中球減少症が解消した。完全寛解を得た26例のうち17例(65%)に微小残存病変(MRD)が検出されなかった。全30例の無増悪生存率(PFS)は、中央値で37カ月間の追跡で78%、奏効が得られた26例の無再発生存率は、中央値で34カ月間の追跡で85%であった。事後解析で、MRD陰性およびBRAF阻害薬投与歴がないことに無再発生存期間の改善との相関関係が認められた。毒性は主にグレード1または2で、試験薬剤に既知のものだった。【結論】小規模な試験では、化学療法を使用せず、骨髄毒性のないベムラフェニブとリツキシマブを用いた短期間の併用療法により、再発または難治性HCL患者のほとんどが長期間に及ぶ完全寛解を得た。 第一人者の医師による解説 骨髄抑制のリスクがなく感染症を有する患者でも安全な治療が可能 今井 陽一 東京大学医科学研究所附属病院血液腫瘍内科准教授 MMJ. December 2021;17(6):182 ヘアリー細胞白血病(HCL)は腫瘍細胞に毛髪状の絨毛と細胞膜の波打ちという特徴的な形態を認める慢性Bリンパ球増殖性疾患で、汎血球減少と脾腫を特徴とする。高齢男性が多く罹患する(男性/女性=4/1)(1)。HCLの原因遺伝子変異BRAFV600EはRAS-RAF-MEK-ERKシグナルの恒常的活性化をもたらす。抗がん剤治療として、クラドリビン、ペントスタチンなどのプリンアナログによく反応する。しかし58%は再発し、徐々に感受性が低下し血液毒性や免疫能低下の副作用が進行する(2)。BRAF阻害薬ベムラフェニブ投与の全奏効率(完全奏効[CR]+部分奏効[PR])は91%で、CR率は35%と深い反応が得られた(3)。一方、CRが得られた患者でも骨髄生検の免疫組織染色で5~10%の腫瘍細胞が残存し、再発のリスクが高い。本論文は再発・難治HCLに対するベムラフェニブと抗CD20抗体リツキシマブ併用療法の有効性と安全性を評価した第2相試験の結果を報告している。BRAFV600E変異を有するHCL患者に対してベムラフェニブは8週間投与され、リツキシマブは18週にわたって8回投与された。年齢中央値61歳の31人が参加し、全例にプリンアナログの投与歴があった。治療企図解析(intention-to-treatanalysis)でCRは87%で得られた。アレル特異的PCR検査によるBRAFV600E検出に基づく微小残存病変(MRD)はCRが得られた患者の65%で陰性であった。追跡期間中央値37カ月時点の無増悪生存率は78%であった。治療終了時にMRD陰性であった17人ではMRD陰性を確認してから中央値28.5カ月時点において全例のMRD陰性が維持された。このように長期にわたって病勢がコントロールされた。リツキシマブ関連の副作用として急性輸液反応や一過性の好中球減少がみられ、重篤な副作用は一過性の好中球減少(12.9%)のみであった。ベムラフェニブとリツキシマブの併用療法は短期間で投与でき安全性も高く、深い治療反応を持続することができた。抗がん剤と異なり骨髄抑制のリスクがなく感染症を有する患者でも安全に治療が可能となり、新型コロナウイルス感染症パンデミックにおける治療として注目される。 1. Falini B, et al. Blood. 2016;128(15):1918-1927. 2. Rosenberg JD, et al. Blood. 2014;123(2):177-183. 3. Tiacci E, et al. N Engl J Med. 2015;373(18):1733-1747.
筋層浸潤尿路上皮がんに用いるニボルマブ補助療法とプラセボの比較
筋層浸潤尿路上皮がんに用いるニボルマブ補助療法とプラセボの比較
Adjuvant Nivolumab versus Placebo in Muscle-Invasive Urothelial Carcinoma N Engl J Med. 2021 Jun 3;384(22):2102-2114. doi: 10.1056/NEJMoa2034442. 原文をBibgraph(ビブグラフ)で読む 上記論文の日本語要約【目的】高リスク筋層浸潤尿路上皮がんの根治的手術後に用いる術後補助療法の役割は明らかになっていない。【方法】第III相多施設共同二重盲検無作為化比較試験で、根治的手術を受けた筋層浸潤尿路上皮がん患者をニボルマブ(240mg静脈内投与)群とプラセボ群に1対1の割合で割り付け、2週間ごとに最長1年間投与した。試験登録前にシスプラチンを用いた術前化学療法を実施していてもよいこととした。主要評価項目は、全患者(intention-to-treat集団)および腫瘍のPD-L1発現レベル1%以上の患者の無病生存とした。尿路外無再発生存を副次的評価項目とした。【結果】計353例をニボルマブ群、356例をプラセボ群に割り付けた。intention-to-treat集団の無病生存期間中央値は、ニボルマブ群が20.8カ月(95%CI、16.5~27.6)、プラセボ群が10.8カ月(95%CI、8.3~13.9)であった。6カ月時の無病生存率はニボルマブ群が74.9%、プラセボ群が60.3%であった(再発または死亡のハザード比、0.70;98.22%CI、0.55~0.90;P<0.001)。PD-L1発現レベルが1%以上の患者では、割合はそれぞれ74.5%と55.7%であった(ハザード比、0.55、98.72%CI、0.35~0.85、P<0.001)。intention-to-treat集団の尿路外無再発生存期間の中央値は、ニボルマブ群が22.9カ月(95%CI、19.2~33.4)、プラセボ群が13.7カ月(95%CI、8.4~20.3)であった。6カ月時の尿路外無再発生存率は、ニボルマブ群が77.0%、プラセボ群が62.7%(尿路外の再発または死亡のハザード比、0.72;95%CI、0.59~0.89)。PD-L1発現レベル1%以上の患者では、それぞれ75.3%および56.7%であった(ハザード比、0.55;95%CI 0.39~0.79)。ニボルマブ群の17.9%とプラセボ群の7.2%にグレード3以上の治療関連有害事象が発現した。ニボルマブ群では、間質性肺炎による治療関連死が2件報告された。【結論】根治的手術を受けた高リスク筋層浸潤尿路上皮がん患者を対象とした本試験では、術後ニボルマブによりintention-to-treat集団およびPD-L1発現レベル1%以上の患者の無病生存期間がプラセボよりも長くなった。 第一人者の医師による解説 日本でも21年3月に適応拡大申請 手術療法+ニボルマブがいずれ標準治療に 水野 隆一 慶應義塾大学医学部泌尿器科学教室准教授 MMJ. December 2021;17(6):183 筋層浸潤性尿路上皮がんに対する標準治療は、膀胱がんであれば膀胱全摘除術、腎盂尿管がんであれば腎尿管全摘除術とされている。これらは根治を目的とした術式であるが、病理学的に固有筋層浸潤や所属リンパ節転移を認める患者における術後再発率は50%以上と報告されており、再発抑制を目的とした術後補助療法の確立は緊急の課題である。今回報告されたCheckMate274試験は、根治切除後の再発リスクが高い筋層浸潤性尿路上皮がん患者を、ニボルマブ(240mg)群とプラセボ群に1:1に割り付けて比較評価した第3相試験である。根治切除後120日以内で画像再発がない、病理学的に尿路上皮がんが確認された患者を対象とした。ニボルマブ、プラセボともに2週間ごとに最長1年間投与された。主要評価項目は、全無作為化患者(ITT)およびPD-L1発現レベル1%以上の患者における無病生存期間(DFS)、副次評価項目は尿路外無再発生存期間(NUTRFS)、疾患特異的生存期間(DSS)、全生存期間(OS)であった。探索的評価項目は、無遠隔転移生存期間(MFS)、安全性、健康関連の生活の質(QOL)などであった。その結果、ITT解析による主要評価項目DFSは、ニボルマブ群において20.8カ月と、プラセボ群10.8カ月に比べ有意に延長していた(ハザード比[HR],0.70;P<0.001)。PD-L1発現レベル1%以上の集団における6カ月時点の無病生存率もニボルマブ群74.5%、プラセボ群55.7%とニボルマブ群で有意に改善していた(HR,0.55;P<0.001)。NUTRFS、DSS、OS、MFSについても、ITT、PD-L1発現レベル1%以上の集団のどちらでもニボルマブ群で延長が認められた。グレード3以上の治療関連有害事象はニボルマブ群で17.9%、プラセボ群で7.2%に認められた。治療関連有害事象による投薬中止はニボルマブ群12.8%、プラセボ群2.0%であった。ITT、PD-L1発現レベル1%以上の集団ともに、ニボルマブ群ではプラセボ群に比べ健康関連QOLの悪化は認められなかった。本試験の結果から高リスク筋層浸潤性尿路上皮がんの術後補助療法として、ニボルマブがプラセボよりも有意にDFSを延長できることが明らかとなった。局所進行腎盂尿管がん患者に対するプラチナ製剤の術後補助療法によるDFS延長は示されているが、コンセンサスはない。ニボルマブは、高リスク筋層浸潤性尿路上皮がんの術後補助療法として2021年8月に米食品医薬品局(FDA)が承認し、日本でも21年3月に適応拡大が申請された。手術療法+ニボルマブ術後補助療法が高リスク筋層浸潤性尿路上皮がんの標準治療になる日は近い。
1999~2018年の米国の成人にみられる糖尿病の治療とコントロールの傾向
1999~2018年の米国の成人にみられる糖尿病の治療とコントロールの傾向
Trends in Diabetes Treatment and Control in U.S. Adults, 1999-2018 N Engl J Med. 2021 Jun 10;384(23):2219-2228. doi: 10.1056/NEJMsa2032271. 原文をBibgraph(ビブグラフ)で読む 上記論文の日本語要約 【背景】糖尿病の治療および危険因子のコントロールの現在の傾向を記録することにより、公衆衛生に関する政策や計画に有用な情報が得られる。 【方法】National Health and Nutrition Examination Survey(NHANES)に参加した米国の糖尿病成人のデータを横断的に解析し、1999年から2018年にかけての糖尿病の治療と危険因子のコントロールに関する全国的な傾向を評価した。 【結果】1999年から2010年代前半にかけて、参加者の糖尿病コントロールが改善したが、その後は停滞し、低下した。2007~2010年の期間から2015~2018年の期間までに、糖尿病を有する成人NHANES参加者のうち、血糖コントロール(糖化ヘモグロビン値7%未満)を達成した人の割合は、57.4%(95%CI、52.9~61.8)から50.5%(95%CI、45.8~55.3)に低下した。2000年代前半に脂質コントロール(非高比重リポタンパクコレステロール値130mg/dL未満)が大きく改善したのち、2007~2010年(52.3%;95%CI、49.2~55.3)から2015~2018年(55.7%;95%CI、50.8~60.5)までにわずかな改善がみられた。血圧コントロール(140/90 mmHg未満)を達成した参加者の割合は、2011~2014年から2015~2018年までに74.2%(95%CI、70.7~77.4)から70.4%(95%CI、66.7~73.8)に低下した。3つの目標を同時に達成した参加者の割合は、2010年以降頭打ちとなり、2015~2018年は22.2%(95%CI、17.9~27.3)であった。何らかの血糖降下薬または降圧薬を使用した参加者の割合は、2010年以降変化がみられず、スタチンを使用した参加者の割合は2014年以降横ばいとなった。併用療法使用者の割合は、2010年以降、血圧コントロールが不良な参加者では低下し、血糖コントロールが不良な参加者では頭打ちになった。 【結論】糖尿病を有する成人NHANES参加者で、1999年から2010年代前半までの10年以上にわたり向上がみられたのち、血糖と血圧のコントロールが低下したが、脂質のコントロールが横ばいとなった。 第一人者の医師による解説 低血糖回避の重要性が認識され、低血糖が生じにくい薬剤の上市による生命予後改善が期待 入江 潤一郎 慶應義塾大学医学部腎臓内分泌代謝内科准教授 MMJ. February 2022;18(1):14 全世界で糖尿病患者は増加の一途を辿っているが、実臨床における糖尿病治療の変遷は明らかではない。本研究では、米国の大規模な国民健康栄養調査(NHANES)のデータ(1999~2018年)を用いて、糖尿病と診断されている成人6,653人について、糖尿病関連の指標の検討を行った。 血糖管理に関しては、HbA1c値7.0%未満を達成した人の割合は、57.4%(2007 ~ 10年)から50.5%(2015 ~ 18年)に低下していた。しかし、その10年前である1999 ~ 2002年 の44.0%より上昇していた。血圧管理に関しては、収縮期 /拡張期血圧140/90mmHg未満を達成している人の割合が、74.2%(2011 ~ 14年)から70.4%(2015 ~ 18年)に低下していた。脂質に関しては、非高比重リポ蛋白(non-HDL)コレステロール値130mg/dL未満を達成している人の割合は、52.3%(2007 ~ 10年)から55.7%(2015~18年)に上昇を認めた。血糖・血圧・脂質の3つの管理目標をすべて達成した人の割合は、2007 ~ 10年には24.9%、2015 ~ 18年には22.2%と増減を認めなかった。糖尿病治療薬に関しては、1999年と比較し、2018年ではメトホルミン、インスリンの使用が増えた一方で、スルフォニル尿素薬、チアゾリジン薬の使用が減少していた。また2003年以降はNa+/グルコース共役輸送担体(SGLT)2阻害薬やグルカゴン様ペプチド(GLP)-1受容体作動薬の使用が増加していた。 これまで、DCCT(米国・カナダ)やUKPDS(英国)など、糖尿病患者の血糖管理が合併症に与える影響を検討した大規模研究から、厳格な血糖管理によりHbA1c値を低くすることが、糖尿病合併症の発症・進展を抑制できることが示されていた。これらの結果に基づき、HbA1c値を下げることは心血管事故死も予防すると考えられていたが、2008年に発表されたACCORD試験とADVANCE試験では、対象によっては厳格な血糖管理によっても心血管事故死の抑制が得られないことが明らかになった。その理由として、血糖値をより低く管理しようとすると、一部の糖尿病患者では低血糖のリスクが高くなり、不整脈や昏睡などが生じ、全死亡が増加した可能性が考えられた。その結果、低血糖を回避することの重要性が認識されるようになり、糖尿病患者の血糖管理目標が患者に応じて緩和されるようになったため、2015 ~ 18年の血糖管理が悪化した可能性がある。これらの試験以降、ジペプチジルペプチダーゼ(DPP)-4阻害薬、GLP-1受容体作動薬、SGLT2阻害薬など、低血糖を起こしにくい薬剤が上市されており、今後はこれらの薬を用いた糖尿病患者の生命予後の改善が期待される。 臨床試験略号:DCCT;Diabetes Control and Complications Trial、UKPDS;UK Prospective Diabetes Study、ACCORD;Action to Control Cardiovascular Risk in Diabetes、ADVANCE;Action in Diabetes and Vascular Disease
脳卒中の予防を目的とした心臓手術中の左心耳閉鎖術
脳卒中の予防を目的とした心臓手術中の左心耳閉鎖術
Left Atrial Appendage Occlusion during Cardiac Surgery to Prevent Stroke N Engl J Med. 2021 Jun 3;384(22):2081-2091. doi: 10.1056/NEJMoa2101897. Epub 2021 May 15. 原文をBibgraph(ビブグラフ)で読む 上記論文の日本語要約 【背景】手術による左心耳の閉鎖は、心房細動患者の虚血性脳卒中を予防すると言われているが、証明されていない。この手技は、別の理由による心臓手術中に施行することが可能である。 【方法】別の理由による心臓手術を予定しており、CHA2DS2-VAScスコア(0~9、スコアが高いほど脳卒中リスクが高いことを示す)が2点以上の心房細動を有する患者を対象に、多施設共同無作為化試験を実施した。患者を無作為化により、手術中に左心耳閉鎖術を施行するグループと左心耳閉鎖術を施行しないグループに割り付けた。全患者に追跡期間中、経口抗凝固薬など通常の治療を実施する予定とした。主要評価項目は、虚血性脳卒中(神経画像検査で所見がみられる一過性脳虚血発作を含む)または全身性塞栓症の発生とした。患者、試験担当者および(外科医を除く)プライマリケア医に試験群の割り付けを伏せた。 【結果】主解析は、閉鎖群2,379例、非閉鎖群2,391例を対象とした。平均年齢は71歳、CHA2DS2-VASc平均スコアは4.2点だった。参加者を平均3.8年間追跡した。92.1%が割り付けた手術を受け、76.8%が3年時点で経口抗凝固薬服用を継続していた。閉鎖群の114例(4.8%)、非閉鎖群の168例(7.0%)が脳卒中または全身性塞栓症を発症した(ハザード比、0.67;95%CI、0.53~0.85;P=0.001)。周術期出血、心不全または死亡の発生率には両試験群間に有意差は認められなかった。 【結論】心臓手術を施行した心房細動患者で、ほとんどが経口抗凝固薬の服用を継続しており、虚血性脳卒中または全身性塞栓症のリスクは心臓手術中に左心耳閉鎖術を同時に施行した方が左心耳を閉鎖しないよりも低かった。 第一人者の医師による解説 左心耳閉鎖術は抗凝固療法の脳梗塞予防効果を増強 さらなる研究を期待 浅井 徹 順天堂大学医学部心臓血管外科学教授 MMJ. February 2022;18(1):9 心房細動は高齢者によくみられ、脳梗塞の原因のうち約25%を占めている。心原性脳塞栓症の多くは心房細動に起因し、左心房にある左心耳が血栓塞栓物の発生部位と考えられている(1)。経口抗凝固薬は左心耳内の血栓形成抑制効果を有する安全な薬剤であり、心房細動のある患者で脳梗塞発症率をおよそ3分の2に低下させることが明らかになっている。しかし実際には、投薬の中断や用量のコントロールが不十分であるといった問題がある。これに対し、左心耳閉鎖術は心房細動合併患者の脳梗塞発症リスクを半永久に低下させる効果が期待されているが、ランダム化試験ではまだ証明されていない。 本論文は、心臓手術を受ける患者のうち心房細動を有するCHA2DS2-VAScスコア2以上の患者を対象に追加手技として左心耳閉鎖術の実施群(2,379人)と非実施群(2,391人)で術後遠隔期までの脳梗塞発症を比較した多施設ランダム化試験(LAAOS III試験)の報告である。左心耳閉鎖術の方法は、切除閉鎖(推奨)、ステイプラーによる切除、デバイスによる閉鎖、または左心房内からの閉鎖が用いられ、術中経食道心エコーによる確認が推奨された。手術後の患者は経口抗凝固薬を含めた通常の薬物療法を受けた。平均追跡期間は3.8年、術後3年の時点で全体の76.8%の患者が経口抗凝固療法を継続していた。その結果、主要評価項目である脳梗塞または他臓器の塞栓症の発症率は、左心耳閉鎖術実施群4.8%、非実施群7.0%であった(ハザード比 , 0.67;95%信頼区間 , 0.53~0.85;P=0.001)。また、周術期の出血合併症、心不全、死亡率で両群間に有意差はなかった。著者らは、心房細動を有する患者が心臓手術を受ける際に左心耳閉鎖術を併施した場合、併施しない場合と比較して、術後の脳梗塞または他臓器の塞栓症の発症リスクが低くなると結論づけている。 左心耳は心房性ナトリウム利尿ペプチド(ANP)の産生部位であり、左心耳の切除によって腎における塩と水の排出が損なわれ心不全が増悪する懸念があるが、今回の試験では術後早期も遠隔期も心不全による入院や死亡率に関して両群間で差を認めなかった。LAAOS III試験は、左心耳閉鎖術自体と経口抗凝固療法を比較した研究ではないため、左心耳閉鎖術が経口抗凝固療法の代用となりうるとは解釈できないが、左心耳閉鎖術が経口抗凝固療法の脳梗塞発症率をさらに3分の2ほどに低下させる効果があることを示しており、今後のさらなる研究結果が待たれるところである。 1. Blackshear JL, et al. Ann Thorac Surg. 1996;61(2):755-759.
尋常性乾癬でのbimekizumabとアダリムマブの比較
尋常性乾癬でのbimekizumabとアダリムマブの比較
Bimekizumab versus Adalimumab in Plaque Psoriasis N Engl J Med. 2021 Jul 8;385(2):130-141. doi: 10.1056/NEJMoa2102388. Epub 2021 Apr 23. 原文をBibgraph(ビブグラフ)で読む 上記論文の日本語要約 【背景】bimekizumabは、インターロイキン-17Aおよびインターロイキン-17Fを選択的に阻害するIgG1モノクローナル抗体である。中等度ないし重度の尋常性乾癬で腫瘍壊死因子阻害薬アダリムマブと比較したbimekizumabの有効性と安全性は、大規模に検討されてこなかった。 【方法】中等度ないし重度の尋常性乾癬の患者を無作為化により、bimekizumab 320 mgを4週ごと56週間皮下投与するグループ、bimekizumabを320 mgの4週ごと16週間投与したのち第16~56週に8週ごとに投与するグループ、アダリムマブ40 mgを2週ごとに24週間皮下投与したのちbimekizumab 320 mgを4週ごとに第56週まで投与するグループに1対1対1の割合で割り付けた。主要評価項目は、16週時点のPsoriasis Area and Severity Index(PASI)スコアの治療前から90%以上の低下(PASI 90の改善;PASIスコアは0~72でスコアが高いほど重症であることを示す)およびInvestigator’s Global Assessment(IGA)スコア0または1(病変が消失またはほぼ消失)とした。主要評価項目の解析では、-10ポイントのマージンで非劣性を検定し、その後優越性を検定した。 【結果】614例をスクリーニングし、478例を登録した。158例がbimekizumab を4週に1回投与するグループ、161例がbimekizumabを4週に1回投与したのち8週に1回投与するグループ、159例がアダリムマブを投与するグループに割り付けられた。患者の平均年齢は44.9歳、治療前のPASIスコアは平均で19.8であった。16週時、bimekizumabを投与した患者(両用量群の統合)319例中275例(86.2%)およびアダリムマブを投与した患者159例中75例(47.2%)がPASI 90を達成した(調整後リスク差、39.3ポイント;95%CI、30.9~47.7;非劣性および優越性のP<0.001)。bimekizumabを投与した患者319例中272例(85.3%)およびアダリムマブを投与した患者159例中91例(57.2%)のIGAスコアが0または1であった(調整後リスク差、28.2ポイント;95%CI、19.7~36.7;非劣性および優越性のP<0.001)。bimekizumabで頻度が高かった有害事象は、上気道感染症、口腔カンジダ症(試験担当者により主に軽度ないし中等度と記録)、高血圧および下痢であった。 【結論】56週間にわたる試験で、bimekizumabは、尋常性乾癬の症状および徴候の軽減に関して16週間を通じてアダリムマブにする非劣性および優越性を示したが、口腔カンジダ症および下痢の頻度が高かった。尋常性乾癬の治療で、bimekizumabの有効性と安全性を他の薬剤と比較し判断するには、さらに長期間にわたる大規模な試験が必要である。 第一人者の医師による解説 ビメキズマブはIL-17AとIL-17Fの両方を阻害 即効性とともに持続性も期待 多田 弥生 帝京大学医学部皮膚科学講座主任教授 MMJ. February 2022;18(1):19 尋常性乾癬は難治な慢性炎症性皮膚疾患であり、外観による患者 QOL(quality of life)の障害度が高く、有病率は世界全体ではおよそ3%、国内有病率は約0.3%とされる。治療選択肢の1つに生物学的製剤があり、日本では現在10種類が使用可能である。その標的サイトカインは大きく腫瘍壊死因子(TNF)、インターロイキン(IL)-23、IL-17の3つである。IL-17にはIL-17A~Fの6種類が存在する。このうちIL-17AとIL-17Fは作用や構造が似ており、同じIL-17受容体 Aに結合することもわかっている。IL-17AとIL-17Fはホモおよびヘテロ結合体を形成でき、生理活性はIL-17A/A、A/F、F/Fの順番に高い。そのため乾癬においては、IL17Aの阻害が最も効果的であると考えられてきたが、IL-17F阻害の意義も注目されていた。 本論文の内容は、IL-17AとIL-17Fを両方阻害するビメキズマブの投与開始後16週時点での有効性と安全性を、TNF阻害薬のアダリムマブと直接比較した海外第3相無作為化二重盲検試験(BE SURE)の結果報告である。対象は中等度~重度の成人尋常性乾癬患者478人で、以下の3群に1:1:1で割り付けられた;(1)56週までビメキズマブ 320mgを4週ごとに投与(2)16週までビメキズマブ320mgを4週ごとに投与した後、56週まで同用量で8週ごとに投与(3)24週までアダリムマブ40mgを2週ごとに投与し、その後56週までビメキズマブ 320mgを4週ごとに投与。16週時点での乾癬の皮疹面積、重症度指標(PASI)で90%以上の改善(PASI 90)を達成した割合、および医師による全般的評価(IGA)スコアの消失またはほぼ消失(IGA 0/1)を達成した割合において、ビメキズマブはアダリムマブと比較し有意に高い効果を示した(P<0.001)。これらの効果はいずれの投与量の群においても、56週まで維持された。ビメキズマブは4週時点でのPASI 75達成患者割合でもアダリムマブより高いことから、即効性も示した。安全性については、口腔内カンジダ症と下痢がビメキズマブ群でアダリムマブ群よりも多く認められたが、口腔内カンジダ症の97%は軽症から中等症であり、これまでの報告と大きく変わらなかった。これまでIL-17阻害薬は維持期の投与間隔が長くても4週であったが、今回、16週以降8週間隔としても治療効果が低下しないことも示されており、ビメキズマブは即効性とともに持続性も有する薬剤である可能性が示されている。
心不全のため入院した高齢患者の身体リハビリテーション
心不全のため入院した高齢患者の身体リハビリテーション
Physical Rehabilitation for Older Patients Hospitalized for Heart Failure N Engl J Med. 2021 Jul 15;385(3):203-216. doi: 10.1056/NEJMoa2026141. Epub 2021 May 16. 原文をBibgraph(ビブグラフ)で読む 上記論文の日本語要約 【背景】急性非代償性心不全のため入院した高齢患者は、身体的フレイル、生活の質の低下、回復の遅れおよび再入院の頻度が高い。この集団の身体的フレイルに対処する介入方法が十分に確立されていない。 【方法】多施設共同無作為化比較試験を実施し、4つの身体機能(筋力、平衡性、可動性および持久力)の強化を目的とした移行期の個別化段階的リハビリテーションによる介入を評価した。介入は、心不全による入院中または入院早期に開始し、退院後も36回の外来リハビリを実施した。主要評価項目は、3ヵ月後のShort Physical Performance Batteryスコア(総スコア0~12点、低スコアほど身体機能障害が重度であることを示す)とした。副次評価項目は、6ヵ月後の原因を問わない再入院率とした。 【結果】計349例を無作為化し、175例をリハビリテーション介入群、174例を通常治療(対照)群に割り付けた。介入前は両群の患者ともに身体機能が顕著に低下しており、97%がフレイルまたはフレイル予備軍であり、平均併存疾患数は両群ともに5疾患であった。介入群の患者継続率が82%であり、リハビリへの参加率が67%であった。介入前のShort Physical Performance Batteryスコアおよびその他の患者データを調整すると、3ヵ月後のShort Physical Performance Batteryスコアの最小二乗平均値(±SE)スコアは、介入群が8.3±0.2、対照群が6.9±0.2であった(平均群間差、1.5;95%信頼区間[CI]、0.9~2.0、P<0.001)。原因を問わない6ヵ月後の再入院率は、介入群が1.18、対照群が1.28であった(率比、0.93;95%CI、0.66~1.19)。介入群の21例(15例が心血管疾患に起因)、対照群の16例(8例が心血管疾患に起因)が死亡した。あらゆる原因による死亡率は、介入群が0.13、対照群が0.10であった(率比、1.17;95%CI、0.61~2.27)。 【結論】急性非代償性心不全のため入院したさまざまな高齢患者の集団で、多数の身体機能強化を目的とした移行期の個別化段階的リハビリテーションによる介入を早期に開始することによって、通常治療よりも身体機能が大きく改善した。 第一人者の医師による解説 死亡率と再入院率の改善には在宅医療を含めた多角的治療戦略が必要 酒向 正春 ねりま健育会病院院長 MMJ. February 2022;18(1):10 急性非代償性心不全で入院する高齢患者では、身体的フレイル、生活の質(QOL)の低下、回復の遅延、繰り返す再入院が高頻度にみられる(1)。しかし、急性心不全の高齢患者群の身体的フレイルに対するリハビリテーション(以下、リハ)治療の有効性は十分に確立されていない(2)。そこで本論文の著者らは、その有効性を検討するために米国で多施設ランダム化対照試験(REHAB-HF試験)を行った。 本試験では、入院患者27,300人のうち適格基準を満たした急性非代償性心不全の高齢患者349人が積極的リハ治療群175人(平均73.1歳)と通常ケア(対照)群174人(平均72.7歳)に無作為に割り付けられ、比較・評価された。積極的リハ治療群では、筋力強度、バランス、可動性、体力耐久性の4つの機能を強化するために、患者別のリハ介入が計画された。その介入は入院後早期に開始され、入院中継続し、退院後も外来で1回60分のセッションが週3日、12週間(計36回)継続された。主要評価項目は3カ月目のShort Physical Performance Battery(SPPB)のスコア(0から12の範囲で、スコアが低いほど重度の身体機能障害を示す)、副次評価項目は6カ月間の原因を問わない再入院率とされた。 両群は、入院時に身体機能が著しく低く、97%がフレイル状態かプレフレイル状態であり、平均5つの合併症を認めた。積極的リハ治療群の入院治療達成率は82%であり、外来を含めた治療順守率は67%であった。入院時 SPPBスコアの平均± SDは積極的リハ治療群6.0±2.8、対照群6.1±2.6であったが、3カ月の時点で積極的リハ治療群8.3±0.2、対照群6.9±0.2(群間差 , 1.5;95%信頼区間[CI], 0.9 ~ 2.0;P<0.001)となり、積極的リハ治療群で有意な改善が認められた。さらに、積極的リハ治療群では歩行バランス・速度・距離、フレイル、QOL、うつ状態が改善する傾向を示した。6カ月間の再入院件数は積極的リハ治療群194件、対照群213件(発生率比 , 0.93;95% CI, 0.66 ~ 1.19)、全死亡数は積極的リハ治療群21人(心血管系原因15人)、対照群16人(心血管系原因8人)(発生率比 , 1.17;95% CI, 0.61~2.27)であり、いずれも有意差は認められなかった。 以上より、急性非代償性心不全で入院した多様な高齢患者群に対して、入院早期から継続的に患者別に調整された積極的リハ治療を行うことにより身体機能面が大きく改善する有効性が明らかになった。死亡率と再入院率の改善には、在宅医療を含めた多角的治療戦略が必要である。 1. Warraich HJ, et al. Circ Heart Fail. 2018;11(11):e005254. 2. Mudge AM, et al. JACC Heart Fail. 2018;6(2):143-152.
アナルトリーを有する麻痺患者の発話を読み取るための神経補綴
アナルトリーを有する麻痺患者の発話を読み取るための神経補綴
Neuroprosthesis for Decoding Speech in a Paralyzed Person with Anarthria N Engl J Med. 2021 Jul 15;385(3):217-227. doi: 10.1056/NEJMoa2027540. 原文をBibgraph(ビブグラフ)で読む 上記論文の日本語要約 【背景】発話不能の麻痺患者のコミュニケーション能力を回復させる技術により、自律性と生活の質が向上する可能性がある。このような患者の大脳皮質活動から単語や文章を直接読み取るアプローチにより、従来のコミュニケーション補助方法から進歩を遂げる。 【方法】脳幹梗塞によりアナルトリー(明瞭に発話する能力の喪失)および痙性四肢不全麻痺を有する患者1例を対象に、発話を制御する感覚運動皮質領域の硬膜下に高密度のマルチ電極アレイを埋め込んだ。48回のセッションで、患者が50語の語彙の中から選んだ単語を個別に発話しようとする際の皮質活動を22時間分記録した。深層学習アルゴリズムを用いて、記録した皮質活動のパターンから単語を検出し、分類する計算モデルを作成した。この計算モデルに加え、先行する一連の単語から次の単語の確率を算出する自然言語モデルを用いて、患者が発話しようとしている文の完成形を解読した。 【結果】患者の皮質活動から、1分間に中央値で15.2語、リアルタイムで文章を解読し、誤り率が中央値で25.6%であった。事後解析で、患者が個々の単語を発話しようとした試みの98%を検出していた。皮質シグナルを用いて47.1%の正確性で単語を分類し、皮質シグナルは81週間の試験期間を通じて安定していた。 【結論】脳幹梗塞によるアナルトリーおよび痙性四肢不全麻痺を有する患者1例で、深層学習モデルと自然言語モデルを用いて、発話しようとする際の皮質活動から直接、単語と文章を読み取った。 第一人者の医師による解説 単語数の制限や精度の改良が必要だがリアルタイムでコミュニケーション可能な社会の実現を期待 井口 はるひ 東京大学医学部附属病院リハビリテーション科助教 MMJ. February 2022;18(1):23 近年、機械の発達により、発話でコミュニケーションが困難な患者に対してさまざまなコミュニケーション代替装置を提案できるようになってきた。しかし、四肢麻痺などの身体機能障害を有すると、機械操作が困難となるため、コミュニケーションに苦慮することがある。一方、四肢の機能が障害された患者に対して機能を代替するbrain machine interfaceが数多く開発され、商品化もされている。本論文の著者らが所属するサンフランシスコ大学で開発されたspeech neuroprosthesis(会話のための神経代替装置)は、脳に埋め込んだ電極を通じて文字変換・出力し、ディスプレイ画面に表示するbrain-computer interfaceである。 本研究では、16年前に脳幹梗塞のためにアナルトリー(anarthria)と痙性四肢麻痺を生じた男性患者に対して、皮質感覚野周辺の硬膜下に多数の高密度皮質脳波電極を外科的に埋め込み、発語課題を行った際の脳活動から単語と文章を再構築することを試みたものである。アナルトリーとは、発語障害のうち、構音の歪み(音が不明瞭化し、母語の表記方法では表現できない音になること)と語を構成する音と音のつながりに障害(「まいにち」というところを「ま、いーにっち」などになる現象)を持つ状態である(1)。81週間かけて50回のセッションを行い、データ収集することで再構築の精度を上げようとした。患者は提示された①単語と②文章を音読し、その際の脳活動を記録した。データ収集の際、著者らは深層学習モデルを使用し、脳活動から予測して発話検出および単語分類モデルを作成した。また、前の単語から次の単語を予測する自然言語モデルを作成した。その結果、脳活動から、文章をリアルタイムで1分間に中央値15.2語の頻度で読み取り、間違いの割合は自然言語モデルを使わない場合の60.5%に対し、使った場合は25.6%であった。事後解析では、単語レベルでも、発語のタイミングを98%検出でき、単語分類の予測は47.1%の精度で可能であった。 本研究の結果から、発語が困難になっても言語機能の障害がなく脳活動が維持されていれば、言語の抽出が脳波から可能であることが示された。単語については、50単語のみを使用しているため、現時点では状況を限定しての使用になる。今後、電極設置の侵襲の低下、多言語での正確性の確認などが必要と思われる。研究が進み、構音・発声障害に加え、運動機能障害でコミュニケーションが困難となっている患者がリアルタイムでコミュニケーション可能になる社会が現実化することが期待される。 1. 大槻美佳 . 臨床神経学 . 2008;48(11):853-856.
認知症に起因する精神病症状に対するpimavanserinの試験
認知症に起因する精神病症状に対するpimavanserinの試験
Trial of Pimavanserin in Dementia-Related Psychosis N Engl J Med. 2021 Jul 22;385(4):309-319. doi: 10.1056/NEJMoa2034634. 原文をBibgraph(ビブグラフ)で読む 上記論文の日本語要約 【背景】神経変性疾患による認知症を有する患者は、認知症に起因する精神病症状を併発することがある。5-HT2Aの逆作動薬および拮抗薬となる経口薬、pimavanserinが、認知症のさまざまな原因に起因する精神病症状にもたらす効果は不明である。 【方法】アルツハイマー病、パーキンソン病認知症、レビー小体型認知症、前頭側頭型認知症または血管性認知症に起因する精神病症状を有する患者を対象に、第III相二重盲検無作為化プラセボ対照中止試験を実施した。患者に非盲検下でpimavanserinを12週間投与した。8週時および12週時にScale for the Assessment of Positive Symptoms-Hallucinations and Delusions(SAPS–H+D、スコアが高いほど精神症状が重い)スコアが治療前から30%以上低下し、Clinical Global Impression-Improvement(CGI-I)スコアが1(very much improved)または2(much improved)であった患者を無作為化によりpimavanserinの投与を継続するグループとプラセボを投与するグループに1対1の割合で割り付け、26週まで投与した。主要評価項目は、time-to-event解析で評価した精神病症状の再発とし、SAPS-H+Dスコア30%以上上昇かつCGI-Iスコア6(much worse)または7(very much worse)、認知症に起因する精神病症状による入院、有効性欠如による試験治療の中止または試験からの脱落、認知症に起因する精神病症状に対する抗精神病薬の使用のいずれかと定義した。 【結果】非盲検期間中、392例のうち41例が有効性の観点から試験を中止したため、管理上の理由で脱落した。残る351例のうち217例(61.8%)に効果の持続が認められ、105例をpimavanserin群、112例をプラセボ群に割り付けた。pimavanserin群95例中12例(13%)およびプラセボ群99例中28例(28%)に再発が認められた(ハザード比、0.35;95%CI、0.17~0.73;P=0.005)。二重盲検期間中、pimavanserin群105例中43例(41.0%)およびプラセボ群112例中41例(36.6%)に有害事象が発現した。pimavanserin群に、頭痛、便秘、尿路感染症および無症候性QT延長が発現した。 【結論】有効性の観点から早期に中止された試験で、認知症に起因する精神病症状を有し、pimavanserinの効果が認められた患者で、試験薬の継続により試験薬を中止した場合よりも再発のリスクが低かった。認知症に起因する精神病症状に対するpimavanserinの効果を明らかにするために、長期間にわたる大規模な試験が必要である。 第一人者の医師による解説 抗精神病薬とは異なる薬理作用 精神病症状の新しい治療選択肢として期待 稲田 健 北里大学医学部精神科学教授 MMJ. February 2022;18(1):5 認知症の原因となる、アルツハイマー病、レビー小体型認知症、パーキンソン病認知症、血管性認知症、前頭側頭型認知症に対する根本的な治療方法はまだない。実臨床で特に問題となるのは、認知症に関連した精神病症状(認知症関連精神病症状)であり、患者本人にも家族や支援者にも多大な負担となる。このため、認知症そのものの改善がなくとも、精神病症状を改善させる治療法は強く求められている。 本論文は、認知症関連精神病症状に対して、セロトニン受容体5-HT2Aの逆アゴニストおよびアンタゴニストであるピマバンセリンの有効性と安全性を検証したHARMONY試験の報告である。本試験では無作為化前に認知症関連精神病症状を有する患者392人に非盲検下でピマバンセリンが12週間投与され、8週後と12週後に、217人が陽性症状評価尺度で30%以上改善し、臨床的全般性印象尺度(CGI)で1(非常に改善)または2(かなり改善)となった。ピマバンセリンが有効であった患者217人を実薬(ピマバンセリン継続)群とプラセボ群に無作為に割り付け、主要評価項目である再発について二重盲検下で比較した。その結果、再発率はピマバンセリン群13%、プラセボ群28%であった(ハザード比 , 0.35; 95%信頼区間 , 0.17 ~ 0.73;P=0.005)。有害事象は、ピマバンセリン群では41.0%、プラセボ群では36.6%に発生し、事象としては頭痛、便秘、尿路感染症、無症候性 QT延長などであった。 本試験の結果、ピマバンセリンに反応した認知症関連精神病症状を有する患者において、ピマバンセリンを継続した方が再発リスクは低くなることが示された。 認知症関連精神病症状に対しては、適応外使用として、抗精神病薬が使用されているのが現状である。しかし、抗精神病薬の長期使用は、認知機能の悪化、錐体外路作用、鎮静作用、転倒、代謝異常などを伴うことがあり、最小用量、最短期間の使用にとどめることが求められている(1)。ピマバンセリンは抗精神病薬とは異なる薬理作用でありながら、パーキンソン病に関連する精神病症状に対する有効性が確認されており(2)、今回の試験では認知症関連精神病症状に対する有効性が確認された。このことから、今後の認知症関連精神病症状の治療選択肢となることが期待される。 1. Reus VI, et al. Am J Psychiatry. 2016;173(5):543-546. 2. Cummings J, et al. Lancet. 2014;383(9916):533-540.
重症左横隔膜ヘルニアに対する胎児手術の無作為化試験
重症左横隔膜ヘルニアに対する胎児手術の無作為化試験
Randomized Trial of Fetal Surgery for Severe Left Diaphragmatic Hernia N Engl J Med. 2021 Jul 8;385(2):107-118. doi: 10.1056/NEJMoa2027030. Epub 2021 Jun 8. 原文をBibgraph(ビブグラフ)で読む 上記論文の日本語要約 【背景】観察研究から、胎児鏡下気管閉塞術(FETO)により、孤発性先天性横隔膜ヘルニアのため重度の肺低形成を来した胎児の生存率が向上することが示されているが、無作為化試験によるデータがない。 【方法】FETOなどの出生前手術の経験がある施設で非盲検試験を実施し、左側の重症孤発性先天性横隔膜ヘルニアの単生児を妊娠した女性を無作為化により、妊娠27~29週にFETOを施行するグループと待期的治療を実施するグループに1対1の割合で割り付けた。両群ともに、出生後に標準的な治療を実施した。主要評価項目は、新生児集中治療室の生存退室とした。優越性に関する中間解析を5回実施することを事前に定め、群逐次デザインを用いた。最大症例数を116例とした。 【結果】3回目の中間解析ののち、有効性が認められたため試験は早期に中止された。80例を対象としたintention-to-treat解析で、FETO群の出生児の40%(40例中16例)および待期的治療群の出生児の15%(40例中6例)が新生児集中治療室退室まで生存した(相対リスク2.67、95%CI 1.22-6.11、両側のP=0.009)。生後6ヵ月までの生存率は、生存退室率とほぼ同じであった(相対リスク、2.67;95%CI、1.22~6.11)。早期前期破水の発生率は、FETO群の女性の方が待期的治療群の女性よりも高く(47% vs. 11%;相対リスク、4.51;95%CI、1.83~11.9)、早産の発生率も同様であった(75% vs. 29%;相対リスク、2.59;95%CI、1.59~4.52)。胎児鏡下バルーン抜去に伴う胎盤裂傷のため緊急分娩後に1例が死亡し、バルーン抜去失敗のため1例が死亡した。試験中止後にデータが得られた11例を追加した解析で、FETO群の生存退室率が36%、待機的治療群では14%であった(相対リスク、2.65;95%CI、1.21~6.09)。 【結論】左側の重症孤発性先天性横隔膜ヘルニア胎児で、在胎27~29週時点でのFETO施行により、待期的治療群より生存退室率の有意な便益が得られ、この便益は生後6ヵ月まで持続した。FETOにより早期前期破水と早産のリスクが上昇した。 第一人者の医師による解説 出生前治療の有効性を示唆する注目すべき前向き研究 黒田 達夫 慶應義塾大学医学部小児外科教授 MMJ. February 2022;18(1):18 先天性横隔膜ヘルニアは横隔膜に形成不全による欠損孔があり、そこを通して腹部臓器が胸腔内に脱出する疾患である。胎生期の肺は脱出臓器により圧排され、欠損側のみならず対側の肺までもが低形成となり、生後に重篤な呼吸障害を呈する。死亡率は日本小児外科学会の2020年の集計でも4.7%に上り、特に重症例では世界的にも3割近い。この疾患は胎生期からの病態により肺低形成を来すため、古くから出生前治療の適応と考えられてきた。歴史的には1990年にHarrisonらが母体を開腹して妊娠子宮を開き、胎児手術を行った成功例を報告しているが、侵襲が大きい治療で成績は悪かった。その後、ボストンのグループが、胎生期の気道に閉塞機転のある場合に胎児肺発育が助長される現象を発見し、以後、本疾患に対する出生前治療として気管の閉塞が試みられてきた。 本論文で述べられているfetoscopic endoluminal tracheal occlusion(FETO)はその最終到達点といえる手技で、母体の経腹的に挿入された胎児鏡を胎児気管まで進め、胎児気管内でバルーンを膨らませてこれを分娩直前まで気管内に留置することにより胎児の肺発育を促進しようとするものである。この手技の有用性について今世紀はじめに欧米でそれぞれ臨床試験が行われた。先天性横隔膜ヘルニアの危険因子に健側肺容積を頭囲で除して標準化したlung-to-head ratio (LHR)があるが、米国ではLHR 1.4未満の重症例を対象に生後治療例とFETO施行例の生存率を比較した。その結果、生後治療例の生存率77%に対してFETO施行例の生存率は73%で出生前治療による有意の成績改善が得られず、以後、米国におけるFETO施行は限定的になっている。一方、その2年後に出された欧州の臨床試験の報告では、対象をLHR 0.9未満の超重症例にしたところ、生後治療例の生存率9%に対してFETO施行例では63.3%が生存したとしており、欧州ではその後も世界中に呼び掛けてFETOの臨床試験を継続していた。 本論文はその最新の成績として日本を含む世界12カ 国 のFETO施行群40例 と 通常治療群40例の成績を比較した報告で、FETO施行群の生存率が40%と先行報告よりも低いものの、通常治療群の生存率15%を大きく上まわったとする。これより従来の主張と変わらず、FETOは超重症例に対しては生後治療よりも有意に有効であると結論している。一方で米国の研究者の間では、医療レベルや臨床試験体制の異なる多くの国が参加している臨床試験でのデータの質の担保や、適応基準などを疑問視する向きもある。ともあれ出生前治療の有効性を示唆する前向き研究の報告として、注目すべきものと思われる。
思春期児を対象としたBNT162b2 Covid-19ワクチンの安全性、免疫原性および有効性
思春期児を対象としたBNT162b2 Covid-19ワクチンの安全性、免疫原性および有効性
Safety, Immunogenicity, and Efficacy of the BNT162b2 Covid-19 Vaccine in Adolescents N Engl J Med. 2021 Jul 15;385(3):239-250. doi: 10.1056/NEJMoa2107456. Epub 2021 May 27. 原文をBibgraph(ビブグラフ)で読む 上記論文の日本語要約 【背景】重症急性呼吸器症候群コロナウイルス2(SARS-CoV-2)に対するワクチンは、ごく最近まで16歳未満の小児への緊急使用が許可されていなかった。この集団を保護し、対面学習や人の集まりを促進し、集団免疫に貢献するため、安全かつ有効なワクチンが必要である。 【方法】この進行中の国際共同プラセボ対照観察者盲検試験では、参加者を無作為によりBNT162b2ワクチン30μgを21日間隔で2回注射するグループとプラセボを注射するグループに1対1の割合で割り付けた。12~15歳にみられるBNT162b2に対する免疫応答の16~25歳の免疫応答に対する非劣性を免疫原性の目的とした。12~15歳のコホートで安全性(反応原性および有害事象)および確定した新型コロナウイルス感染症2019罹患(Covid-19;発症、2回目接種から7日以上経過後)に対する有効性を評価した。 【結果】全体で、12~15歳の思春期児2,260例に接種した。1,131例にBNT162b2、1,129例にプラセボを投与した。他の年齢群にみられたように、BNT162b2は安全性と副反応が良好で、反応原性事象も主に一過性で軽度から中等度であった(主に注射部位疼痛[79~86%]、疲労[60~66%]、頭痛[55~65%])。ワクチンに起因する重篤な有害事象はなく、重度の有害事象も全体的に少なかった。12~15歳の参加者にみられた2回目接種後SARS-CoV-2 50%中和抗体価の16~25歳に対する幾何平均比は、1.76(95%信頼区間[CI]、1.47~2.10)であり、両側95%信頼区間の下限が0.67を超える非劣性基準を満たし、12~15歳のほうが応答が大きいことが示された。SARS-CoV-2感染歴の根拠がない参加者では、BNT162b2接種群に2回目接種から7日以上経過後のCOVID-19発症例はいなかったが、プラセボ接種群に16例認められた。観察されたワクチンの有効性は100%(95%CI、75.3~100)であった。 【結論】12~15歳の小児に接種したBNT162b2ワクチンは、安全性が良好で、若年成人よりも免疫応答が大きく、COVID-19に対する高い有効性が示された。 第一人者の医師による解説 思春期児童は接種後の血管迷走神経反射の発生率が高く 対策を取ることが重要 新井 智 国立感染症研究所感染症疫学センター第11室室長 MMJ. February 2022;18(1):4 本論文は、ファイザー・ビオンテック社製のメッセンジャー RNAワクチン(BNT162b2)を評価している第1/2/3相試験(C4591001試験)の一部として、米国内の12 ~ 15歳の思春期児童約2,300人をBNT162b2 30μg群とプラセボ(生理食塩水)群に1:1の比で無作為に割り付け安全性、免疫原性、有効性を比較し、さらに米国以外の国からも登録した16 ~ 25歳の青年集団約1,100人と免疫応答での非劣性の検証等を行った結果の報告 である。1回目接種 受けたBNT162b2群1,131人とプラセボ群1,129人、2回目接種を受けたそれぞれ1,124人と1,117人の解析結果から、BNT162b2 30μ g接種における12 ~ 15歳の思春期児童の副反応は、全体的に一過性で軽度~中等度であった。最も頻度が高かった局所反応は注射部位の疼痛で79 ~ 86%、頻度の高かった全身性反応は倦怠感(60 ~ 66%)、頭痛(55 ~ 65%)、悪寒(28 ~ 42%)、筋肉痛(24 ~ 32%)であった。局所反応の頻度は1回目接種時の方が高かったが(1回目86%、2回目79%)、全身性反応の倦怠感、頭痛、悪寒、筋肉痛のいずれも2回目接種時の方が高頻度にみられた。 また、12 ~ 15歳の思春期児童と16 ~ 25歳の青年集団におけるBNT162b2 30μg接種の比較を行ったところ、12 ~ 15歳の思春期児童の1、2回目接種時の副反応発生率は、16 ~ 25歳の青年集団とほとんど違いがなく(16 ~ 25歳の青年:倦怠感60 ~ 66%、頭痛54 ~ 61%、悪寒25 ~40%、筋肉痛27 ~ 41%)、安全性に違いは認められなかった。有効性の指標である50%中和幾何平均抗体価(GMT)は、2回目接種1カ月後の12 ~15歳の思春期児童群では1,283.0、16 ~ 25歳の青年群では730.8、両集団間の幾何平均比は1.76(95%信頼区間 , 1.47 ~ 2.10)であり、免疫応答は12 ~ 15歳の思春期児童群の方が良好であった。 BNT162b2やモデルナ製スパイクバックのメッセンジャー RNAワクチンでは、初回よりも2回目接種の方が発熱、倦怠感、頭痛、悪寒などの全身性反応の発生割合が高い(1),(2)。これらの副反応への対応に関してはアセトアミノフェンの服用も選択肢として提案されており、事前に十分な情報提供を行い安心して接種を受けられる環境の提供が重要である。思春期児童では、ワクチン接種後の血管迷走神経反射の発生割合が他の年齢群に比べ高いため、横たわって接種を受ける、あるいは接種後30分程度様子を見るなど対策を取ることも重要である。 1. Polack FP, et al. N Engl J Med. 2020;383(27):2603-2615. 2. Baden LR, et al. N Engl J Med. 2021;384(5):403-416.
ワクチン起因性免疫性血小板減少症/血栓症 血小板減少と頭蓋内出血で死亡率上昇
ワクチン起因性免疫性血小板減少症/血栓症 血小板減少と頭蓋内出血で死亡率上昇
Clinical Features of Vaccine-Induced Immune Thrombocytopenia and Thrombosis N Engl J Med. 2021 Oct 28;385(18):1680-1689. doi: 10.1056/NEJMoa2109908. Epub 2021 Aug 11. 上記論文のアブストラクト日本語訳 ※ヒポクラ×マイナビ 論文検索(Bibgraph)による機械翻訳です。 【背景】ワクチン誘発性免疫性血小板減少症・血栓症(VITT)は、重症急性呼吸器症候群コロナウイルス2に対するChAdOx1 nCoV-19アデノウイルスベクターワクチンに関連する新しい症候群である。本疾患の臨床的特徴や予後基準に関するデータは不足している。 【方法】2021年3月22日から6月6日の間に英国内の病院を受診したVITTが疑われる患者を対象に前向きコホート研究を実施した。データは匿名化された電子フォームを使用して収集し,事前に指定した基準に従って症例を確定VITTまたはprobable VITTと同定した。患者のベースライン特性および臨床病理学的特徴、危険因子、治療、予後不良のマーカーを決定した 【結果】評価対象となった294例中、VITTの確定例170例と確率例50例を同定した。全例がnCoV-19ワクチンChAdOx1の初回接種を受けており,接種後5~48日(中央値,14日)で発症した.年齢層は18歳から79歳(中央値48歳)で,性差はなく,内科的危険因子もなかった.総死亡率は22%であった。死亡のオッズは,脳静脈洞血栓症患者では2.7倍(95%信頼区間[CI],1.4~5.2),ベースラインの血小板数が50%減少するごとに1.7倍(95% CI,1.3~2.3 ),1.3倍,1.6倍,1.8倍と増加し,脳静脈洞血栓症患者では,ベースラインの血小板数が1%減少すると,1.7倍,1.8倍と増加した.ベースラインのdダイマー値が10,000単位増加するごとに2(95%CI、1.0~1.3)、ベースラインのフィブリノゲン値が50%減少するごとに1.7(95%CI、1.1~2.5)因子増加しました。多変量解析では、ベースラインの血小板数と頭蓋内出血の有無が死亡と独立して関連していることが確認されました。 【結論】VITTに伴う高い死亡率は、血小板数が少なく、頭蓋内出血のある患者で最も高くなっています。治療法はまだ不明であるが、予後マーカーの同定は効果的な管理の指針になる可能性がある。(オックスフォード大学病院NHS財団トラストより資金提供). 第一人者の医師による解説 抗血小板第4因子抗体(ELISA法)の測定が診断のキーポイント 荒岡 秀樹 虎の門病院臨床感染症科部長 MMJ. April 2022;18(2):49 本論文は、アデノウイルスベクターワクチンであるChAdOx1 nCoV-19(アストラゼネカ社製)に関連する、ワクチン起因性免疫性血小板減少症/血栓症(vaccine-induced immune thrombocytopenia and thrombosis;VITT)の臨床像と予後悪化因子を調査した前向きコホート研究の報告である。2021年3〜6月に英国の病院を受診し、VITTが疑われた患者が対象とされた。VITTの診断基準として、以下の5項目が設定された:(1)ワクチン接種後5 ~ 30日に発症(2)血栓症の存在(3)血小板減少(15万 /μ L未満)(4)Dダイマー高値(4,000 fibrinogen-equivalent unit[FEU]*超)(5)抗血小板第4因子抗体陽性(ELISA法)。Definite VITTは5項目すべてを満たすもの、probable VITTは(4)を必須項目として残り4項目中3項目を満たす、あるいはDダイマーが2,000 ~ 4,000 FEUかつ残り4項目(1、2、3、5)を満たすものとした。 期間中、definite VITTが170人、probable VITTが50人抽出され評価された。全例がワクチンの1回目接種を受けており、97%の患者が接種後5 ~ 30日で発症していた。発症年齢は18 ~ 79歳(中央値48歳)、男女比は女性が54%であった。全死亡率は22%と高く、多変量解析では血小板数と頭蓋内出血が死亡に寄与する独立した危険因子であることが特定された。特に血小板数が3万/μL未満、かつ頭蓋内出血を認めた患者では73%が死亡していた。 VITTはthrombosis with thrombocytopenia syndrome(TTS)という名称が用いられることもあり、適切な名称については議論が残る。ヘパリン起因性血小板減少症と類似の病態として捉えられ、特にアデノウイルスベクターワクチン接種後に多いとされている。しかしながら、ワクチン接種後の血栓塞栓症や血小板減少症はmRNAワクチンでも生じるとされ、その解明と対策は重要である。 現時点では、血小板第4因子とワクチンに含まれる成分が複合体を形成し、複合体に対する抗体が血小板の活性化を惹起することが推定されており、抗血小板第4因子抗体(ELISA法)の測定が診断のキーポイントと考えられている。 本研究の潜在的な弱点は、主に診断確定のバイアスにある。しかしながら220症例ものVITTの臨床像を解析し、死亡の危険因子を抽出したことは大きな成果である。今後の治療法の確立に向けて、本病態のkey paperの1つになりうるものである。 *D ダイマーの単位については、日本では一般的にμg/mL が用いられているため、注意を要する。各測定試薬の添付文書の確認が望ましい。また、日本脳卒中学会、日本血栓止血学会から出ている「COVID-19 ワクチン接種後の血小板減少症を伴う血栓症の診断と治療の手引き・第 3 版」(2021 年 10 月)内の記述も参考になる。
PIK3CAの体細胞変異が孤発性海綿状血管腫を引き起こす
PIK3CAの体細胞変異が孤発性海綿状血管腫を引き起こす
Somatic PIK3CA Mutations in Sporadic Cerebral Cavernous Malformations N Engl J Med. 2021 Sep 9;385(11):996. doi: 10.1056/NEJMoa2100440. 上記論文のアブストラクト日本語訳 ※ヒポクラ×マイナビ 論文検索(Bibgraph)による機械翻訳です。 【背景】脳海綿状奇形(CCM)は、散発性および遺伝性の中枢神経系血管奇形としてよく知られている。家族性CCMはKRIT1(CCM1)、CCM2、PDCD10(CCM3)の機能喪失型変異と関連しているが、80%を占める散発性CCMの遺伝子原因はまだ不完全に理解されていない。 【方法】プロスタグランジンD2合成酵素(PGDS)プロモーターを用いて、ヒト髄膜腫で同定された変異を保有する2つのモデルマウスを開発した。患者から外科的に切除されたCCMの標的DNA配列決定を行い、液滴デジタルポリメラーゼ連鎖反応分析で確認した。 【結果】髄膜腫の2つの共通の遺伝的ドライバーであるPik3caH1047RまたはAKT1E17KをPGDS陽性細胞で発現するマウスでは、髄膜腫ではなく典型的なCCMのスペクトラムが発生(それぞれ22%と11%)することがわかり、88例の散発性のCCMから組織標本を解析することになった。患者の病変組織の39%と1%にそれぞれ活性化型のPIK3CAとAKT1の体細胞変異が検出された。病変の10%のみがCCM遺伝子に変異を有していた。活性化変異Pik3caH1074RとAKT1E17Kによって引き起こされる病変をマウスで解析し,PGDS発現周皮細胞を起源細胞として推定した。 【結論】散発性のCCMからの組織試料において,他のどの遺伝子における変異よりPIK3caにおける変異が大きく表れていることがわかった。家族性CCMの原因となる遺伝子の体細胞変異の寄与は比較的小さかった。(ARC対がん研究財団他より資金提供). 第一人者の医師による解説 予想外の発見を見逃さず 病態解明につなげたserendipityの重要さが伝わる論文 武内 俊樹 慶應義塾大学医学部小児科専任講師 MMJ. April 2022;18(2):51 海綿状血管腫は、比較的よく遭遇する中枢神経系の血管奇形であり、家族性あるいは孤発性に発症する。このうち、家族性海綿状血管腫は、KRIT1(CCM1)、CCM2、PDCD10(CCM3)遺伝子のいずれかの機能喪失型変異によって発症することが知られている。一方で、海綿状血管腫の80%を占める孤発性症例の原因は解明されていなかった。 本研究では、ヒト患者の髄膜腫で同定された遺伝子変異およびプロスタグランジン D2合成酵素(PGDS)プロモーターをもつ2種類の疾患モデルマウスを開発した。髄膜腫の2つの一般的なドライバー変異であるPik3caH1047RまたはAKT1E17Kのいずれかを発現するマウスにおいて、予想外に、髄膜腫ではなく海綿状血管腫が発生した(それぞれマウスの22%と11%に発生した)。これに着想を得て、孤発性海綿状血管腫患者88人の病変組織の遺伝子解析を行った。その結果、PIK3CAおよびAKT1に、機能亢進型の体細胞変異をそれぞれ39%と1%に同定した。既知の家族性海綿状血管腫の原因遺伝子に変異を認めたのは10%に過ぎなかった。さらに、Pik3caH1047RとAKT1E17K変異を有するマウスの病変部位の解析から、PGDSを発現する周皮細胞が海綿状血管腫の発生母地となっていることを突き止めた。以上の結果をまとめると、孤発性海綿状血管腫の病変組織では、PIK3CAの体細胞変異が多く認められ、他のどの遺伝子よりも大きな割合を占めていた。既知の家族性海綿状血管腫の原因遺伝子の体細胞変異によるものはむしろ少なかった。 本研究の最も重要な知見は、同じ海綿状血管腫であっても、家族性と孤発性で原因遺伝子が異なることを示した点である。一般に、家族性疾患の原因遺伝子変異が体細胞変異となった場合には、家族性疾患でみられる病変と似た病変を起こすことが多い。しかしながら、今回の研究では、孤発性海綿状血管腫は、家族性海綿状血管腫の原因遺伝子の体細胞変異によって発症しているのではなく、髄膜腫の原因として知られていたPIK3CAの体細胞変異が発症原因の多くを占めていたという驚くべき結果であった。さらに本研究で特筆すべき点は、もともと髄膜種の研究を目的に作製されたマウスにおいて、想定外の海綿状血管腫が発生したことが今回の研究成果の端緒となったことである。これらのマウスは髄膜に発現するPGDSをもつため、PGDSが海綿状血管腫の発症に関与することも突き止めることができた。予想外の発見を見逃さず当初目的としていなかった孤発性海綿状血管腫の病態解明につなげたserendipityの重要さが伝わる論文である。
アトゲパントが片頭痛予防第3相試験の12週間投与期間で高い有効性を示す
アトゲパントが片頭痛予防第3相試験の12週間投与期間で高い有効性を示す
Atogepant for the Preventive Treatment of Migraine N Engl J Med. 2021 Aug 19;385(8):695-706. doi: 10.1056/NEJMoa2035908. 上記論文のアブストラクト日本語訳 ※ヒポクラ×マイナビ 論文検索(Bibgraph)による機械翻訳です。 【背景】 アトゲパントは、片頭痛の予防治療薬として検討されている経口低分子カルシトニン遺伝子関連ペプチド受容体拮抗薬です。 【方法】 第3相二重盲検試験において、1ヶ月に4~14日の片頭痛のある成人を1:1:1:1の割合で、アトゲパン経口投与(10mg、30mg、60mg)またはプラセボ12週間無作為に割り付けたところ、1日1回の投与で、片頭痛の予防効果が認められました。主要評価項目は、12週間にわたる1ヶ月あたりの片頭痛平均日数のベースラインからの変化としました。副次的評価項目は、1ヵ月あたりの頭痛日数、1ヵ月あたりの片頭痛日数の3ヵ月平均のベースラインからの50%以上の減少、QOL、Activity Impairment in Migraine-Diary(AIM-D)スコアなどであった。 【結果】合計2270名がスクリーニングされ、910名が登録され、873名が有効性解析に含まれた。214名がアトジェパント10mg群に、223名がアトジェパント30mg群に、222名がアトジェパント60mg群に、214名がプラセボ群に割り付けられた。ベースライン時の1ヶ月の片頭痛日数の平均は4群とも7.5日から7.9日であった。12週間のベースラインからの変化は、アトジェパント10mg群で-3.7日、アトジェパント30mg群で-3.9日、アトジェパント60mg群で-4.2日、そしてプラセボ群で-2.5日でした。ベースラインからの変化量のプラセボとの平均差は、10mgアトガパントで-1.2日(95%信頼区間[CI]、-1.8~-0.6)、30mgアトガパントで-1.4日(95%CI、-1.9~-0.8)、60mgアトガパントで-1.7日(95%CI、-2.3~-1.2)でした(すべての比較でプラセボとP<0.001)。副次的評価項目の結果は,10 mg 用量の AIM-D 日常生活動作スコアと AIM-D 身体障害スコアを除き,アトジェパントがプラセボを上回った.主な有害事象は便秘(アトジェパント投与期間中6.9~7.7%)および悪心(アトジェパント投与期間中4.4~6.1%)でした。重篤な有害事象は、アトジェパント10mg投与群で喘息と視神経炎が各1例ずつありました。有害事象は、便秘と吐き気であった。片頭痛予防のためのアトゲパントの効果と安全性を明らかにするために、より長期で大規模な試験が必要である。(アラガン社からの資金提供;ADVANCE ClinicalTrials. gov 番号、NCT03777059.). 第一人者の医師による解説 片頭痛発作予防の臨床試験で経口投与 CGRP拮抗薬アトゲパントが好結果 鈴木 則宏 湘南慶育病院院長・慶應義塾大学名誉教授 MMJ. April 2022;18(2):34 アトゲパントは低分子の経口投与カルシトニン遺伝子関連ペプチド受容体(CGRP)拮抗薬(1),(2)であり、本論文は、片頭痛の予防的治療薬として本剤の有効性を検討したADVANCE試験の報告である。この無作為化第3相二重盲検試験では、1カ月当たりの片頭痛日数が4 ~ 14日の成人を12週にわたりアトゲパント 10、30、または60mg、もしくはプラセボを1日1回経口投与する4群に割り付けた。主要エンドポイントは12週間における1カ月当たりの片頭痛日数の平均値のベースラインからの変化量とした。副次エンドポイントは、1カ月当たりの頭痛日数、1カ月当たりの片頭痛日数の3カ月間の平均値のベースラインから50%以上の減少、生活の質(QOL)、片頭痛活動障害・ダイアリー指標(AIM-D)スコアなどとした。スクリーニング後、参加者2,270人のうち910人が組み入れられた。 ベースライン時の1カ月当たりの片頭痛日数の平均値は4群で7.5 ~ 7.9日であった。12週間におけるベースラインからの変化量は、アトゲパント 10mg群-3.7日、30mg群-3.9日、60mg群-4.2日、プラセボ群-2.5日であった。ベースラインからの変化量におけるプラセボ群との平均差は、アトゲパント 10mg群-1.2日、30mg群-1.4日、60mg群-1.7日であった(プラセボ群とのすべての比較でP<0.001)。副次エンドポイントの結果は、アトゲパント 10mg群のAIM-Dの日常活動機能スコアと身体障害スコアを除いて、アトゲパント群の方がプラセボ群よりも良好であった。特に頻度の高かった有害事象は、便秘(6.9 ~ 7.7%[アトゲパント群] 対 0.5%[プラセボ群])と悪心(4.4 ~ 6.1%[アトゲパント群] 対1.8%[プラセボ群])であった。アトゲパント群での重篤な有害事象は10mg群における気管支喘息1人、視神経炎1人であった。以上のように、アトゲパントの1日1回経口投与は、12週間における片頭痛日数と頭痛日数の減少に有効であり、有害事象は便秘、悪心などであった。 近年、片頭痛の予防的治療として抗 CGRPモノクローナル抗体あるいは抗CGRP受容体モノクローナル抗体が開発され、日本でも2021年から3種類の製剤が臨床の場に登場し、高い有効性を示している。しかし、いずれも皮下投与製剤であり、自己注射はまだ認可されていないのが現状である。このような状況と予防効果の高さから、アトゲパントの実臨床への早期の導入が期待されるが、本剤の有効性と安全性をより明らかにするためには、より長期かつ大規模な臨床試験が必要であろう。 1. Min KC, et al. Clin Transl Sci. 2021;14(2):599-605. 2. Goadsby PJ, et al. Lancet Neurol. 2020;19(9):727-737.
S1P受容体調節作用を持つ経口薬オザニモド 潰瘍性大腸炎の寛解導入と寛解維持に有用
S1P受容体調節作用を持つ経口薬オザニモド 潰瘍性大腸炎の寛解導入と寛解維持に有用
Ozanimod as Induction and Maintenance Therapy for Ulcerative Colitis N Engl J Med. 2021 Sep 30;385(14):1280-1291. doi: 10.1056/NEJMoa2033617. 上記論文のアブストラクト日本語訳 ※ヒポクラ×マイナビ 論文検索(Bibgraph)による機械翻訳です。 【背景】選択的スフィンゴシン-1-リン酸受容体モジュレーターであるオザニモドは、炎症性腸疾患の治療薬として研究されている。 【方法】我々は、中等度から重度の活動性を有する潰瘍性大腸炎患者を対象に、オザニモドの導入療法および維持療法に関する第3相多施設共同無作為化二重盲検プラセボ対照試験を実施した。10週間の導入期において、コホート1の患者には、1日1回、塩酸オザニモドを1mg(オザニモド0.92mg相当)またはプラセボとして経口投与することを二重盲検法で行い、コホート2の患者には、1日1回、同じ用量のオザニモドを非盲検法で投与しました。10週目に、いずれかのコホートでオザニモドに臨床的に反応した患者は、維持期間(52週目まで)に二重盲検法でオザニモドまたはプラセボを投与するよう、再び無作為化されました。両期間の主要評価項目は、臨床的寛解を示した患者の割合であり、Mayoスコアの3要素で評価された。主要な副次評価項目である臨床的、内視鏡的、組織学的評価項目は、順位付けされた階層的な検定を用いて評価した。また、安全性についても評価した。 【結果】導入期間では、第1コホートに645名、第2コホートに367名の患者が参加し、維持期間では457名の患者が参加した。臨床的寛解の発生率は、導入期(18.4%対6.0%、P<0.001)および維持期(37.0%対18.5%(10週目に奏効した患者)、P<0.001)のいずれにおいても、オザニモドを投与された患者の方がプラセボを投与された患者よりも有意に高かった。臨床反応の発生率も、導入期(47.8%対25.9%、P<0.001)および維持期(60.0%対41.0%、P<0.001)において、プラセボよりもオザニモドの方が有意に高かった。その他の主要な副次的評価項目は、いずれの期間においてもプラセボと比較してオザニモドにより有意に改善された。オザニモドによる感染症(重症度を問わず)の発生率は,導入期ではプラセボと同程度,維持期ではプラセボよりも高かった.重篤な感染症は、52週間の試験期間中、各群の患者の2%未満に発生しました。肝アミノトランスフェラーゼ値の上昇は、オザニモドでより多く見られました。 【結論】オザニモドは、中等度から重度の活動性を有する潰瘍性大腸炎患者の導入療法および維持療法として、プラセボよりも有効でした。(Bristol Myers Squibb社から資金提供を受けています。True North ClinicalTrials.gov番号、NCT02435992)。 第一人者の医師による解説 既存薬と機序が全く異なるオザニモドの位置づけ 市販後の十分な検討が重要 日比 紀文 北里大学北里研究所病院 炎症性腸疾患先進治療センター長・特任教授 MMJ. April 2022;18(2):44 潰瘍性大腸炎(UC)は、クローン病(CD)を含めて炎症性腸疾患(IBD)と総称される。日本でもUCの患者数は20万人を超え一般的となったが、原因が不明なため根本治療がなく、治療は炎症抑制に加え免疫異常の是正が中心であり、寛解導入療法に加えて長期の寛解維持療法が求められる(1)。近年の生物学的製剤の出現は、その目覚ましい治療効果から、難治と考えられてきたIBDや関節リウマチなど慢性炎症性疾患の治療にパラダイムシフトを起こした。しかし、生物学的製剤の多くは高分子の注射剤であり、経口分子標的薬の開発が待たれている。 本論文は、オザニモドのUCにおける寛解導入療法、維持療法としての有効性および安全性を検証した第3相臨床試験の報告である。オザニモドは選択的にリンパ球表面のスフィンゴシン -1-リン酸(S1P)受容体に働き(2)、リンパ球の炎症部位への動員を抑制するという新しい機序を有し、1日1回経口投与で寛解導入・維持を目指す画期的な薬剤である。本試験には30カ国285施設が参加し、寛解導入は 約1,000人、寛解維持は457人の患者で比較検討された。有効性の主要評価項目である「臨床的寛解」においてプラセボ群と比較し有意に高い治療効果を示し、安全性については想定される「徐脈」「肝障害」がオザニモド群でも比較的少なかったという成績で、特に帯状疱疹は少数例にしか見られず、UCの新たな治療選択肢としてのオザニモドの有用性を証明した貴重な報告である。 一方、本試験は事前に心疾患患者を除き、帯状疱疹には細心のチェックをした状態で実施されており、本剤が実臨床に導入された場合は安全性の面で細心の注意が求められる。生物学的製剤の使用は今後さらに増加すると予想されるが、無効例や長期使用で効果減弱などの問題点があること、注射剤より経口薬を好む患者が多いことなどを踏まえると、本剤への期待は大きい。 近年、日本が参加するグローバル試験も多くなったが、本試験の参加者は大多数が欧米人であり(アジア参加国は韓国のみ;5.8%)、日本人を含む東洋人(モンゴロイド)での有効性や安全性が同等であるかは不明である。日本人でも同様の成績が証明されれば臨床面で重要な薬剤になると考えられる。起こりうる副作用としての徐脈・心電図での伝道異常や肝障害は少しみられたが、日本人では異なる反応を示す可能性もある。日本でもすでに有効性と安全性が海外と同様であるかを検証する試験が終了しており、UCの治療選択肢として加えられることに期待している。さらに、既存薬と機序が全く異なる本剤の位置づけを市販後に十分検討していくことが重要となろう。 1. 日比紀文ら . 日本臨床 . 2017;75(3):364-369. 2. Scott FL, et al. Br J Pharmacol. 2016;173(11):1778-1792. 3. Sandborn WJ, et al. N Engl J Med. 2016 May 5;374(18):1754-1762.
血栓回収術前のアルテプラーゼ静注療法併用 血栓回収単独療法に対し優越性、非劣性とも示されず
血栓回収術前のアルテプラーゼ静注療法併用 血栓回収単独療法に対し優越性、非劣性とも示されず
A Randomized Trial of Intravenous Alteplase before Endovascular Treatment for Stroke N Engl J Med. 2021 Nov 11;385(20):1833-1844. doi: 10.1056/NEJMoa2107727. 上記論文のアブストラクト日本語訳 ※ヒポクラ×マイナビ 論文検索(Bibgraph)による機械翻訳です。 【背景】 急性虚血性脳卒中に対する血管内治療(EVT)の前にアルテプラーゼを静注することの価値については、特にアジア以外の地域では広く研究されていません。患者は、EVTのみを行う群と、アルテプラーゼ静注後にEVTを行う群(標準治療)に、1対1の割合で無作為に割り付けられました。主要評価項目は,90 日後の修正 Rankin スケールによる機能的転帰(範囲,0[障害なし]~6[死亡])であった.アルテプラーゼ+EVTに対するEVT単独の優越性を評価するとともに、両試験群のオッズ比の95%信頼区間の下限を0.8とするマージンで非劣性を評価した。安全性の主要評価項目は,あらゆる原因による死亡と症候性脳内出血であった。 【結果】解析対象は539例であった。90日後のmodified Rankin scaleのスコアの中央値は,EVT単独群で3(四分位範囲,2~5),アルテプラーゼ+EVT群で2(四分位範囲,2~5)であった。調整後の共通オッズ比は0.84(95%信頼区間[CI],0.62~1.15,P=0.28)で,EVT単独の優越性も非劣性も認められなかった。死亡率は,EVT 単独で 20.5%,アルテプラーゼ+EVT で 15.8%であった(調整オッズ比,1.39;95%信頼区間,0.84~2.30).症候性脳内出血は,それぞれの群で5.9%と5.3%に発生した(調整オッズ比1.30,95%CI,0.60~2.81)。 【結論】ヨーロッパの患者を対象とした無作為化試験において,脳卒中発症後90日目の障害の転帰に関して,EVT単独はアルテプラーゼ静注後にEVTを行う場合と比較して優越性も非劣性もなかった。症候性脳内出血の発生率は,両群で同等であった。(Collaboration for New Treatments of Acute Stroke コンソーシアムなどの助成を受け、MR CLEAN-NO IV ISRCTN 番号、ISRCTN80619088)。 第一人者の医師による解説 アルテプラーゼ静注併用の是非は決着せず 日本の実臨床ではアルテプラーゼ静注先行が標準 鶴田 和太郎 虎の門病院脳神経血管内治療科部長 MMJ. April 2022;18(2):37 急性期脳梗塞に対する再開通療法として、アルテプラーゼ静注療法は迅速に施行可能であり、高いエビデンスを持った治療法であるが、血管径の太い近位主幹動脈に対する効果は限定的である。一方、カテーテルを用いて行う血栓回収術は、治療設備や専門医が必要であるため施行可能な施設が限定され、治療開始までに時間を要するが、再開通率は高い。これまでアルテプラーゼ静注併用を前提とした血栓回収術の有効性・安全性のエビデンスが集積されてきた。アルテプラーゼ静注の併用は、血栓回収率を上げるという報告がある一方、血栓回収術開始までの時間が長くなることや出血性合併症のリスクが高くなるといったデメリットも考えられており、現在これら併用の是非についての検証が進行中である。 本論文は欧州人を対象とした血栓回収単独とアルテプラーゼ併用のランダム化対照試験(MR CLEAN–NO IV)の報告である。対象はアルテプラーゼ静注と血栓回収の両者が適応となる近位脳主幹動脈閉塞による脳梗塞急性期の患者で、血栓回収単独群またはアルテプラーゼ併用群(アルテプラーゼ静注 +血栓回収)に割り付けられた。主要評価項目は90日後の日常生活自立度(modified Rankin Scale;mRS)、主な安全性評価項目は全死亡、症候性頭蓋内出血とされた。対象患者539人の解析において、90日後のmRSスコア中央値は、血栓回収単独群で3(四分位範囲[IQR], 2〜5)、アルテプラーゼ併用群で2(IQR, 2〜5)であり、アルテプラーゼ併用群の優越性、非劣性とも示されなかった。全死亡、症候性頭蓋内出血の発生率についても両群でおおむね同程度であった。 脳梗塞急性期の血栓回収単独とアルテプラーゼ併用のランダム化対照試験としては、これまでに日本で行われたSKIP(1)と中国で行われたDIRECTMT(2)およびDEVT(3)の結果が報告されている。SKIPでは血栓回収単独のアルテプラーゼ併用に対する非劣性は示されなかったのに対し、DIRECTMTおよびDEVTでは非劣性が示された。 血栓回収前のアルテプラーゼ静注併用の是非については、未だ決着しておらず、進行中の他の研究結果が待たれる。日本の実臨床において、脳梗塞急性期の患者が必ずしも血栓回収を実施できる施設に搬送されるとは限らず、病院間転送が必要な患者にはアルテプラーゼ静注を先行すべきである。血栓回収単独の効率的な提供には、血栓回収適応患者を適切に血栓回収対応可能な施設に搬送するための脳卒中医療のセンター化と搬送システムの構築が喫緊の課題といえる。 1. Suzuki K, et al. JAMA. 2021;325(3):244-253. 2. Yang P, et al. N Engl J Med. 2020;382(21):1981-1993. 3. Zi W, et al. JAMA. 2021;325(3):234-243.
重度のCovid-19患者に対するレムデシビルの慈悲深い使用。
重度のCovid-19患者に対するレムデシビルの慈悲深い使用。
Compassionate Use of Remdesivir for Patients with Severe Covid-19 N Engl J Med 2020 Jun 11;382(24):2327-2336. 上記論文のアブストラクト日本語訳 ※ヒポクラ×マイナビ 論文検索(Bibgraph)による機械翻訳です。 【背景】SARS-CoV-2の感染によって引き起こされる病気であるCovid-19で入院している患者に、レムデシビルを同情的に使用した。患者は,SARS-CoV-2感染が確認された者で,常用空気を吸っているときの酸素飽和度が94%以下であるか,酸素のサポートを受けている者であった。患者はレムデシビルを10日間投与され、その内訳は、1日目に200mgを静脈内投与し、その後残りの9日間は1日100mgを投与するというものでした。本報告書は、2020年1月25日から2020年3月7日までの期間にレムデシビルの投与を受け、その後の少なくとも1日分の臨床データを有する患者のデータに基づいています。 【結果】レムデシビルの投与を少なくとも1回受けた61名の患者のうち、8名のデータが解析できませんでした(治療後のデータがない7名と投与ミスの1名を含む)。データが解析された53名の患者のうち、22名は米国、22名は欧州またはカナダ、9名は日本に在住していました。ベースラインでは、30名(57%)の患者が人工呼吸を受けており、4名(8%)の患者が体外式膜酸素療法を受けていました。中央値18日の追跡期間中、36人(68%)の患者で酸素サポートクラスが改善し、そのうち機械的換気を受けていた30人(57%)の患者のうち17人が抜管された。死亡率は、人工呼吸を受けている患者では18%(34人中6人)、人工呼吸を受けていない患者では5%(19人中1人)であった。 【結論】重症のCovid-19で入院し、思いやりのある使い方をしたレムデシビルで治療を受けたこのコホートでは、53人中36人(68%)に臨床的改善が認められた。有効性の測定には、レムデシビル療法に関する継続的な無作為化プラセボ対照試験が必要である。(Gilead Sciences社より資金提供を受けています。) 第一人者の医師による解説 速報として捉える必要あるが 重症COVID-19にレムデシビルは光明となりうる 葉 季久雄 平塚市民病院救急科・救急外科部長 MMJ.August 2020;16(4) 2019年12月に中国・武漢で報告されて以来、新型コロナウイルス感染症(COVID-19)は今や世界的な脅威となっている。2020年2月ごろの日本のCOVID-19 治療最前線では、日日に呼吸状態が増悪していく重症COVID-19患者を我々は目の当たりにしていた。確立された治療薬がない中、数少ない論文を拠り所に、他疾患で承認済みの薬剤を臨床試用していた。そのような状況で、重症治療にあたっていた医師が求めていたものは、今、目の前で苦しんでいる患者を治すための治療薬、すなわち抗ウイルス薬であった。 レムデシビル(remdesivir)はエボラ出血熱の治療薬として開発されたウイルスRNAポリメラーゼ阻害薬である。エボラ出血熱に対してはより有効な薬剤が開発されたため、レムデシビルは全世界で未承認の薬剤であった。レムデシビルはin vitroにおいてコロナウイルスを含む1本鎖RNAウイルスに活性を示すことが知られている。中国からの報告では、in vitroにおいてSARS-CoV-2に対しても強い活性を示していたため、このパンデミック下における治療薬として再び注目された。 本論文は、人道的見地から治療目的に提供されたレムデシビルの重症COVID-19患者に対するコホート研究の報告である。対象となったのは、SARSCoV-2への感染が確認され、室内気で酸素飽和度が94%以下であるか酸素療法中の入院患者で、レムデシビルは10日間連日投与(1日目200 mg、2~10日目100mg)された。評価項目は、酸素療法必要度、転帰であった。本研究では対照群がないため、レムデシビルがCOVID-19に有効であるか否かを明らかにすることは不可能であり、本論文を解釈する際は、速報として結果を捉える必要がある。 データが解析された患者53 人のうち、中央値18日間のフォローアップ中に、68 %(36 /53) で酸素療法の状況が改善した。その一方で、15%(8 /53)は増悪した。改善の具体例としては、57%(17 /30)の患者で抜管することができ、体外式膜型人工肺(ECMO)が導入された患者4人のうち3人において離脱することができた。転帰は47 %(25 /53)が退院、13 %(7 /53)が死亡した。死亡率は人工呼吸器で管理された患者で18%(6 /34)、人工呼吸器で管理がされなかった患者で5%(1/19)であった。 本研究は、この後に続くランダム化比較試験(RCT)へのイントロダクションである。そのRCTの結果ならびに米食品医薬品局(FDA)の緊急時使用許可を踏まえて、レムデシビルは日本において重症COVID-19に対する治療薬(ベクルリー®)として特例承認された。本研究は、重症COVID-19患者に対するレムデシビルの有効性に、最初に光を当てた研究である。
週1回皮下注持続性 GLP-1アゴニストは10〜18歳未満の肥満2型糖尿病患者の糖代謝を改善
週1回皮下注持続性 GLP-1アゴニストは10〜18歳未満の肥満2型糖尿病患者の糖代謝を改善
Once-Weekly Dulaglutide for the Treatment of Youths with Type 2 Diabetes N Engl J Med. 2022 Aug 4;387(5):433-443. doi: 10.1056/NEJMoa2204601. Epub 2022 Jun 4. 上記論文のアブストラクト日本語訳 ※ヒポクラ×マイナビ 論文検索(Bibgraph)による機械翻訳です。 [背景] 2 型糖尿病の発生率は若者の間で増加しています。グルカゴン様ペプチド-1受容体アゴニストであるデュラグルチドによる週1回の治療は、2型糖尿病の若者の血糖コントロールに関して有効である可能性があります.ライフスタイルの変更のみまたはメトホルミンで、基礎インスリンの有無にかかわらず、1:1:1 の比率で治療を受けている参加者 (10 歳から 18 歳未満、ボディマス指数 [BMI]、> 85 パーセンタイル) を無作為に割り当てました。週に 1 回、プラセボ、デュラグルチド 0.75 mg、またはデュラグルチド 1.5 mg の皮下注射を受けます。その後、参加者は 26 週間の非盲検延長試験に参加し、プラセボを投与されていた人が毎週 0.75 mg のデュラグルチドの投与を開始しました。主要評価項目は、26 週での糖化ヘモグロビン レベルのベースラインからの変化でした。二次エンドポイントには、7.0%未満の糖化ヘモグロビンレベルと、空腹時グルコース濃度およびBMIのベースラインからの変化が含まれていました。安全性も評価されました。 [結果] 合計 154 人の参加者が無作為化されました。 26 週の時点で、糖化ヘモグロビンの平均レベルはプラセボ群で増加し (0.6 パーセント ポイント)、デュラグルチド群で減少しました (0.75 mg 群で -0.6 パーセント ポイント、1.5 mg 群で -0.9 パーセント ポイント、プラセボに対する両方の比較で P < 0.001)。 26 週の時点で、プールされたデュラグルチド群の参加者の割合が、プラセボ群よりも高く、7.0% 未満の糖化ヘモグロビン レベルでした (51% 対 14%、P < 0.001)。空腹時血糖値はプラセボ群で増加し(1デシリットルあたり17.1mg)、プールされたデュラグルチド群で減少し(1デシリットルあたり-18.9mg、P <0.001)、BMIの変化にグループ間差はありませんでした.胃腸の有害事象の発生率は、プラセボよりもデュラグルチド療法の方が高かった.デュラグルチドの安全性プロファイルは、成人で報告されたものと一致していました。メトホルミンまたは基礎インスリンの有無にかかわらず、BMI に影響を与えずに治療を受けている。 (Eli Lilly から研究助成を受けた。AWARD-PEDS ClinicalTrials .gov 番号 NCT02963766)。 第一人者の医師による解説 消化器症状の懸念はあるが デュラグルチドは血糖管理に有効 内潟 安子 東京女子医科大学附属足立医療センター病院長 特任教授 MMJ.February 2023;19(1):18 肥満2型糖尿病患者、特に若い年代の当該患者は世界的に急増し、糖尿病性合併症の急速な進展もみられる。若い糖尿病患者に対する治療指針の根拠の1つに米国 TODAY試験が有名であるが、推奨の第1治療薬メトホルミンでは対象患者のおよそ半数が血糖管理に失敗し、膵β細胞機能の急激な悪化を阻止できなかった(1)。若い糖尿病患者は成人の糖尿病患者と比較し、その病因的背景は相等であると言われるのだが、インスリン抵抗性、膵β細胞機能不全ともに成人2型糖尿病より迅速に重症化しやすいのではないかと考えられる(2),(3)。その一方、若い糖尿病患者への薬物療法は成人2型糖尿病の薬物療法ほどには進歩していない。 本論文は、10歳~18歳未満の肥満2型糖尿病患者を対象に、成人の2型糖尿病に広く使用されているグルカゴン様ペプチド(GLP)-1アゴニストのデュラグルチドを0.75mg、1.5mg、もしくはプラセボを週1回皮下投与する群に無作為に割り付け、26週間後の経過を比較した第3相二重盲検優越性試験(AWARD-PEDS)の報告である。対象は各群約50人で、年齢分布、性別、人種構成、体格指数(BMI)(34前後)、HbA1c値(8.0%前後)、ベースライン治療(メトホルミン単独63%前後、メトホルミン+基礎インスリン併用25%前後、基礎インスリン治療単独3%前後)に関して3群間で同様であった。 26週時点の結果は以下のとおりである。平均HbA1c値はプラセボ群で0.6%上昇し、デュラグルチド 0.75mg群、1.5mg群ではそれぞれ0.6%、0.9%低下した(いずれもP<0.001)。HbA1c値が7.0%未満であった参加者の割合は、プラセボ群14%に対して、デュラグルチド 0.75mg群と1.5mg群の統合群では51%であった(P<0.001)。空腹時血糖値は、プラセボ群で17.1mg/dL上昇、デュラグルチド統合群では18.9mg/dL低下した(P<0.001)。BMIの増減に関して有意な群間差はなかった。消化器症状はデュラグルチド群に多くみられたが、デュラグルチドの安全性プロファイルは成人2型糖尿病での報告と同様であった。 今回の検討から、10代の肥満2型糖尿病患者において、メトホルミン服用の有無や基礎インスリン治療の有無に関係なく、デュラグルチドは0.75mgであれ1.5mgであれ投与26週間後に、減量効果はみられなかったが、血糖値の良好化に関してプラセボよりも有意な効果を得られることが明らかとなった。 1. TODAY Study Group. N Engl J Med. 2012;366(24):2247-2256. 2. RISE Consortium. Diabetes Care. 2018;41(8):1696-1706. 3. Utzschneider KM, et al. Diabetes Res Clin Pract. 2021;178:108948.
コロナ禍の2022年前半の英国で小児に重症急性肝炎が流行
コロナ禍の2022年前半の英国で小児に重症急性肝炎が流行
Clinical Spectrum of Children with Acute Hepatitis of Unknown Cause N Engl J Med. 2022 Aug 18;387(7):611-619. doi: 10.1056/NEJMoa2206704. Epub 2022 Jul 13. 上記論文のアブストラクト日本語訳 ※ヒポクラ×マイナビ 論文検索(Bibgraph)による機械翻訳です。 【背景】2022年1月以降、小児における原因不明の急性肝炎の報告が増加しています。症例は複数の大陸で報告されていますが、ほとんどは英国で報告されています。原因物質を特定するための調査が進行中です。血清に関連して、A型肝炎からE型肝炎ではなく、代謝的、遺伝的、遺伝的、先天的、または機械的原因を持たない、確認された急性肝炎の英国健康安全保障局の症例定義を満たす肝炎を持っていた1リットルあたり500 IUを超えるアミノトランスフェラーゼレベル。医療記録をレビューし、人口学的特徴、臨床的特徴、肝生化学的検査、血清学的検査、肝向性ウイルスやその他のウイルスの分子検査の結果、および放射線学的および臨床的転帰を文書化しました。結果は、状態の改善、肝移植、または死亡として分類されました。年齢の中央値は 4 歳 (範囲、1 ~ 7 歳) でした。一般的な症状は、黄疸 (93% の子供)、嘔吐 (54%)、下痢 (32%) でした。ヒトアデノウイルスの分子検査を受けた30人の患者のうち、27人(90%)が陽性でした。劇症肝不全は 6 人の患者 (14%) で発生し、その全員が肝移植を受けました。死亡した患者はいなかった。肝移植を受けた 6 人を含むすべての子供は退院した.この病気は確立されていません。 第一人者の医師による解説 アデノウイルス 41Fとアデノ随伴ウイルス 2型のツインデミックが関与か 森内 浩幸 長崎大学大学院医歯薬学総合研究科小児科学教授 MMJ.February 2023;19(1):6 2022年初頭、英国から原因不明の小児重症肝炎が多発していると報告され、世界を驚かせた。その後世界中から同様の症例が世界保健機関(WHO)に登録され、9月29日までに555例が集まったが、そのほとんどは地域的(スコットランド)・時間的(1~6月[ピーク 3~4月])に集積していた。 本論文は英国の小児肝移植センターで2022年1月1日~4月11日に経験された重症急性肝炎44人の報告である。いずれも従来は健康な1~7歳児で、6人に肝移植が行われた。既知の原因は否定されたが、30人中27人(90%)でアデノウイルスが検出された。肝組織中にウイルス封入体やアデノウイルス抗原は検出されなかったが、肝組織粉砕液からは6人中3人でアデノウイルス DNAがPCR法で検出され、塩基配列よりアデノウイルス41Fと推定された。アデノウイルスは免疫健常宿主に肝炎を起こさず、41Fは胃腸炎ウイルスに過ぎない。その後も英国ではアデノウイルスが過半数の症例から検出され、その多くが41Fであった(1)。 その後の知見に基づく仮説はこうだ。新型コロナウイルス感染症(COVID-19)予防策の緩和後、スコットランドにおいてアデノウイルス 41Fとアデノ随伴ウイルス 2型(AAV2)が同時流行した。AAV2は単独では増殖できず、ヘルパーウイルスを必要とし、アデノウイルスは代表的なヘルパーウイルスである。この2つのウイルスに同時感染した子どもの肝臓でAAV2が活発に増殖し、感染した肝細胞に対して宿主免疫系(特に細胞性免疫)が攻撃を加えることによって肝炎が起こる。このような免疫攻撃はある種のヒト白血球抗原(HLA)(DRB1*04:01)を有する人に起こりやすい。このHLA型は北欧に多く、スコットランド人の15.6%が保有する。スコットランドの研究では患児の多くからAAV2が検出されたが、健常対照児からは検出されなかった。また患児の89%が上述のHLA型であった(2)。 このような機序の肝炎には前例がある。D型肝炎ウイルスもB型肝炎ウイルスというヘルパーウイルスがいなければ増殖できず肝炎は起こせない。またウイルス性肝炎の多くは、ウイルスが直接肝細胞を破壊するのではなく、ウイルス感染肝細胞を宿主の細胞性免疫が認識して破壊する。そしてHLA型と感染症との相性も以前から知られている(HIV、C型肝炎など)。まだ完全に証明されたわけではないが、これが本当ならコロナ禍に生じた各 種感染症の疫学の乱れから起こったツインデミック(アデノウイルス 41FとAAV2)が起こした現象だったと言える。「新興感染症」が既知の2種類のウイルスと宿主の遺伝背景の組み合わせでも起こるかもしれないということだ。 1. Gong K, et al. Front Pharmacol. 2022;13:1056385. 2. Ho A, et al. medRxiv 2022.07.19.22277425.
2022年4月〜6月に16カ国でみられたサル痘感染症 典型的な特徴は発疹と性器病変
2022年4月〜6月に16カ国でみられたサル痘感染症 典型的な特徴は発疹と性器病変
Monkeypox Virus Infection in Humans across 16 Countries - April-June 2022 N Engl J Med. 2022 Aug 25;387(8):679-691. doi: 10.1056/NEJMoa2207323. Epub 2022 Jul 21. 上記論文のアブストラクト日本語訳 ※ヒポクラ×マイナビ 論文検索(Bibgraph)による機械翻訳です。 [背景] 2022 年 4 月以前は、サル痘ウイルスが風土病であるアフリカ地域以外では、ヒトのサル痘ウイルス感染はほとんど報告されていませんでした。現在、世界中で症例が発生しています。伝染、危険因子、臨床症状、および感染の転帰は十分に定義されていません。 -確認されたサル痘ウイルス感染。 [結果] 2022 年 4 月 27 日から 6 月 24 日の間に 16 か国の 43 か所で診断された 528 の感染を報告します。全体として、感染者の 98% がゲイまたはバイセクシュアルの男性で、75% が白人で、41% がヒト免疫不全ウイルスに感染していました。年齢の中央値は 38 歳でした。伝染は、感染者の 95% で性行為を介して発生した疑いがありました。この症例シリーズでは、95% の人に発疹があり (64% は 10 個未満の病変)、73% は肛門性器病変、41% は粘膜病変 (54 人は単一の性器病変) でした。発疹に先行する一般的な全身的特徴には、発熱 (62%)、無気力 (41%)、筋肉痛 (31%)、および頭痛 (27%) が含まれていました。リンパ節腫脹も一般的でした (56% で報告されました)。検査を受けた 377 人中 109 人 (29%) で性感染症の併発が報告されました。暴露歴が明らかな 23 人のうち、潜伏期間の中央値は 7 日 (範囲、3 ~ 20 日) でした。サル痘ウイルス DNA は、精液を分析した 32 人中 29 人で検出されました。全体の 5% の人に抗ウイルス治療が施され、70 人 (13%) が入院しました。入院の理由は疼痛管理であり、主に肛門直腸の重度の痛み(21 人)であった。軟部組織重複感染 (18);経口摂取を制限する咽頭炎 (5);眼病変 (2);急性腎障害 (2);心筋炎 (2);および感染制御目的(13)。死亡例は報告されていません。 [結論] このケース シリーズでは、サル痘はさまざまな皮膚科学的および全身的な臨床所見とともに現れました。サル痘が伝統的に風土病であった地域以外での症例の同時同定は、さらなる地域での広がりを封じ込めるために、症例の迅速な同定と診断の必要性を浮き彫りにしています。 第一人者の医師による解説 過去のサル痘と比べて非典型的な症状が多く 病歴などを詳細に聴取し積極的に診断すべき 谷口 俊文 千葉大学医学部附属病院感染制御部准教授 MMJ.February 2023;19(1):21 2022年4月前までサル痘(現在では世界保健機関[WHO]によりMpoxと名称変更)はアフリカ以外ではほとんど報告されなかったが、現在、世界中でみかけるようになった。本論文はその感染経路や危険因子、臨床的特徴などに関する報告である。サル痘は英国保健安全保障庁(UKHSA)の定義、すなわち病変からのサル痘 PCR陽性で確定診断している。 本研究では約2カ月間に英国を中心に16カ国から528人の感染者が報告された。サル痘患者の98%はゲイまたはバイセクシュアル男性で、2%は異性愛者であった。95%で性行為による感染が疑われた。9%が天然痘ワクチン接種の既往を報告。発疹(黄斑、膿疱、小水疱、痂皮)は95%に認められ、肛門、性器に最も多く(73%)、次いで体幹・上下肢(55%)、顔(25%)、手掌または足底(10%)であった。病変は10個以下が多く、5個以下は39%にみられた。その他の所見としては、性器潰瘍(10%)、直腸痛、直腸炎、テネスムスなどを伴う直腸粘膜病変(12%)、そして口腔咽頭症状(5%)などがみられた。発熱は62%と一般的であったが、すべての患者に全身性の前駆症状があったわけではない。時系列的な評価を受けた30人の患者では、最初の皮膚病変から追加の皮膚病変までの期間中央値は5日間であった。41%がHIVに感染しており、このうちほぼ全員が有効な抗 HIV薬で治療されていた。そしてサル痘の臨床症状は、HIV感染者と非感染者で同様であった。性感染症のスクリーニングを受けた患者の29%は、微生物学的に確定された性感染症を併発していた。全患者の13%が入院し、その多くは疼痛管理や軟部組織への2次感染のためであり、2人が心筋炎、1人が喉頭蓋炎を発症した。5%がサル痘に対する治療(テコビリマット、シドホビル外用など)を受けていた。 今回の流行におけるサル痘の典型的な特徴は、発疹と性器病変であり、後者は性的接触時の感染を反映している可能性がある。過去に報告されたサル痘と比べてあまり典型的ではない症状(単一の性器潰瘍、手掌と足底の病変など)が多いため、病歴などを詳細に取った上で積極的に診断する必要がある。WHOは最近、サル痘を世界的な健康上の緊急事態と宣言し、診断、検査、ワクチン接種、治療へのアクセスの迅速化を含む、効果的かつ公平な対応の必要性を強調している。 1. 循環器病の診断と治療に関するガイドライン;失神の診断・治療ガイドライン(2012 年改訂版) 2. Vyas A, et al. Int J Cardiol. 2013;167(5):1906-1911. 3. Sheldon R, et al. Eur Heart J. 2006;27(3):344-350.
FFRガイドのPCIはIVUSガイドのPCIと比較して 心血管イベント発生は非劣性
FFRガイドのPCIはIVUSガイドのPCIと比較して 心血管イベント発生は非劣性
Fractional Flow Reserve or Intravascular Ultrasonography to Guide PCI N Engl J Med. 2022 Sep 1;387(9):779-789. doi: 10.1056/NEJMoa2201546. 上記論文のアブストラクト日本語訳 ※ヒポクラ×マイナビ 論文検索(Bibgraph)による機械翻訳です。 [背景] 経皮的冠動脈インターベンション (PCI) の評価を受けている冠動脈疾患患者では、血行再建術とステント留置に関する意思決定のために、フラクショナル フロー リザーブ (FFR) または血管内超音波検査 (IVUS) によって手技をガイドできます。しかし、両方の目的に 1 つの方法のみを使用した場合の臨床転帰の違いは明らかではありません。血管造影) を 1:1 の比率で行い、FFR ガイドまたは IVUS ガイドのいずれかの手順を実行します。 FFR または IVUS を使用して、PCI を実行するかどうかを決定し、PCI の成功を評価する必要がありました。 FFR 群では、FFR が 0.80 以下の場合に PCI を実施することとした。 IVUS群では、PCIの基準は、70%を超えるプラーク負荷を伴う3mm 2以下または3~4mm 2の最小管腔面積であった。主要転帰は、無作為化から 24 か月後の死亡、心筋梗塞、または血行再建術の複合でした。 IVUS 群と比較した FFR 群の非劣性をテストしました (非劣性マージン、2.5 パーセント ポイント)。 [結果] PCI の頻度は、FFR 群の患者で 44.4%、IVUS 群の患者で 65.3% でした。 24 か月の時点で、FFR 群の患者の 8.1% と IVUS 群の患者の 8.5% で主要転帰イベントが発生しました (絶対差、-0.4 パーセント ポイント; 片側 97.5% 信頼度の上限間隔、2.2 パーセント ポイント; 非劣性の P = 0.01)。シアトル狭心症アンケートで報告された患者報告のアウトカムは、2 つのグループで類似していた。心筋梗塞、または 24 か月での血行再建術。 (Boston Scientific から研究助成を受けた。FLAVOR ClinicalTrials.gov 番号 NCT02673424)。 第一人者の医師による解説 FFR 対 IVUS 我が国のプラクティスに与える影響は少ない 阿古 潤哉 北里大学医学部循環器内科学教授 MMJ.February 2023;19(1):10 冠動脈の内圧を測定し冠動脈の狭窄が生理学的に有意かどうかを判断する方法がfractional flow reserve(FFR)である。また、冠動脈の解剖学的狭窄を正確に評価する方法がintravascular ultrasonography(IVUS)である。両者は基本的には全く異なるものを評価しているが、いずれの検査法も経皮的冠動脈インターベンション(PCI)を受ける患者の評価において、冠動脈造影を補完する診断法であるという点では共通項があると言える。 今回報告されたFLAVOUR試験は、PCIガイド用の検査法としてFFRはIVUSに勝るかという臨床的疑問を解くべく計画された。FFRガイド PCIは冠動脈造影ガイドのPCIに比べ心血管イベントが少なくなるというデータがすでにある(1)。対するIVUSガイド PCIにはそのようなデータがないため、FFRガイドの優越性を検証する試験デザインが組まれた。しかし試験開始後に、IVUSガイド PCIは冠動脈造影ガイド PCIに比べ予後の改善がみられるというデータが複数報告されたため、FFRガイド PCIのIVUSガイド PCIに対する非劣性を検討するデザインに変更された。韓国と中国の18施設で中等度狭窄のある患者1,682人を組み入れ、FFRあるいはIVUSでPCI適応の有無を判断した。結果、FFR群では44.4%、IVUS群では65.3%の患者にPCIが実施された。24カ月時点の心血管イベント(死亡、心筋梗塞、再血行再建)発生率はFFRガイド群で8.1%、IVUSガイド群で8.5%であり、非劣性が示された。 生理的虚血の評価法であるFFRと、解剖学的形態の評価法であるIVUSを比較するというのは、ともすると非常にわかりにくい試験デザインである。そもそも、安定冠動脈疾患に対する治療法の選択において、PCIか内科的治療先行かという比較試験がいくつか行われたが、死亡、心筋梗塞などのいわゆるハードエンドポイントにおいて有意差をつけた臨床試験はない。その意味では、PCIのガイド方法の変化程度ではイベントに有意差が出ないであろうというのは十分に予想される結果である。非劣性を検証するデザインに変更されたために本試験の意味合いが少し不明確になったと言える。この結果をもって、冠動脈造影ガイド PCIが標準となっている多くの国の医療環境下では、FFRガイドでad-hoc PCI(診断目的の冠動脈造影に続いて実施するPCI)を行うことにお墨付きを与える形になるかもしれない。しかし、従来から基本的にPCIをIVUSなどの冠動脈イメージングガイド下で行っている我が国では、今回の試験結果が与えるインパクトはほとんどなさそうである。 1. Xaplanteris P, et al. N Engl J Med. 2018;379(3):250-259.
PCI後1年でルーチンに負荷検査を実施しても 2年間での転帰改善なし
PCI後1年でルーチンに負荷検査を実施しても 2年間での転帰改善なし
Routine Functional Testing or Standard Care in High-Risk Patients after PCI N Engl J Med. 2022 Sep 8;387(10):905-915. doi: 10.1056/NEJMoa2208335. Epub 2022 Aug 28. 上記論文のアブストラクト日本語訳 ※ヒポクラ×マイナビ 論文検索(Bibgraph)による機械翻訳です。 [背景] 心筋血行再建術後の特定の追跡調査アプローチを導く無作為化試験から得られたデータは限られています。定期的な機能検査を含むフォローアップ戦略が、経皮的冠動脈インターベンション (PCI) を受けたハイリスク患者の臨床転帰を改善するかどうかは不明です。 PCI から 1 年後の定期的な機能検査 (核負荷試験、運動心電図検査、負荷心エコー検査) のフォローアップ戦略、または標準治療単独への PCI。主要転帰は、あらゆる原因による死亡、心筋梗塞、または不安定狭心症による 2 年間の入院の複合でした。重要な副次的アウトカムには、侵襲的冠動脈造影と繰り返しの血行再建術が含まれていました。 38.7% が糖尿病で、96.4% が薬剤溶出性ステントで治療されていました。 2 年後、機能検査群では 849 人中 46 人 (Kaplan-Meier 推定、5.5%)、標準治療群では 857 人中 51 人 (Kaplan-Meier 推定、6.0%) で、主要転帰事象が発生した。グループ (ハザード比、0.90; 95% 信頼区間 [CI]、0.61 から 1.35; P = 0.62)。主要アウトカムの構成要素に関して、グループ間で違いはありませんでした。 2 年後、機能検査群の患者の 12.3%、標準治療群の 9.3% の患者が侵襲的冠動脈造影を受けており (差、2.99 パーセント ポイント、95% CI、-0.01 ~ 5.99)、8.1% およびそれぞれ 5.8% の患者が繰り返し血行再建術を受けていた (差、2.23 パーセンテージ ポイント; 95% CI、-0.22 ~ 4.68)。 [結論] PCI を受けた高リスク患者では、定期的な機能検査の追跡戦略は、標準治療単独と比較して、2 年で臨床転帰を改善しませんでした。 (CardioVascular Research Foundation および Daewoong Pharmaceutical から研究助成を受けた。POST-PCI ClinicalTrials.gov 番号、NCT03217877)。 第一人者の医師による解説 高リスク患者でもルーチン負荷検査実施に臨床的意義なし 背景に冠動脈ステント性能の向上 清末 有宏 森山記念病院循環器センター長 MMJ.February 2023;19(1):11 金属ステントを冠動脈に留置していた時代にはステント内再狭窄が20~30%に見られたため、経皮的冠動脈インターベンション(PCI)実施後6カ月前後で再入院のうえ経皮的冠動脈造影(CAG)を必ず実施していたし、実際に20~30%の患者は再 PCIとなった。しかし2004年以降薬剤溶出性ステントの時代に突入し、免疫抑制薬の新生内膜増殖抑制作用により、ステント内再狭窄率は5~10%に低下した。第1世代薬剤溶出性ステントではポリマーによる血管炎症が問題となったが、その後第2世代、第3世代と日進月歩の勢いで進化を遂げ、近年ではどのステントを用いてもステント内再狭窄率は高リスクな背景を有する患者でなければ1~2%と驚異的な数字となっている。また同時に画像診断技術も大きく進歩し、心電図同期造影 CTによる冠動脈の描出はCAGに引けを取らないレベルまで質が高まりつつあり、各種の非侵襲的な機能的負荷試験も心筋虚血の検出において感度・特異度を上げてきている。 上記状況を踏まえPCI実施後の患者の臨床的経過観察をどのようにすべきか、ということに関しては各国の医療環境を考慮に入れながら長期にわたって世界的に議論が続いており、またその議論は常に最新のエビデンスとともにアップデートされることを余儀なくされている。日本では欧米諸国に比べ再CAGの敷居が低い時代が長く続いたが、近年は再 CAGをルーチンで実施することは推奨されていない。ガイドラインには本件に関する記載はないものの、日本のReACT試験では再 CAG実施群において非実施群に比べ冠疾患関連複合エンドポイントの改善は認められなかった(ハザード比 , 0.94;95%信頼区間[CI], 0.67~1.31)(1)。本論文のPOST-PCI試験ではさらに一歩進んで、PCI後の解剖学的または臨床的な高リスク患者に対して1年後にルーチンの機能的負荷試験(負荷心筋シンチ、運動負荷心電図、負荷心エコーのいずれか)を実施しても、2年間の冠疾患関連複合エンドポイントは非実施群と比較し改善しなかった(HR,0.90;95% CI, 0.61~1.35)。いろいろな方向性での解釈が可能な試験結果かとは思うが、一つ確実に言えることは冠動脈ステントの性能がいわば“Fire-and-forget”に耐えられるほどに向上したということであろうか。 1. Shiomi H, et al. JACC Cardiovasc Interv. 2017;10(2):109-117.
コンピュータに基づくICU患者の厳格な血糖管理 ICU入室期間や死亡率に差なし
コンピュータに基づくICU患者の厳格な血糖管理 ICU入室期間や死亡率に差なし
Tight Blood-Glucose Control without Early Parenteral Nutrition in the ICU N Engl J Med. 2023 Sep 28;389(13):1180-1190. doi: 10.1056/NEJMoa2304855. 上記論文のアブストラクト日本語訳 ※ヒポクラ 論文検索(Bibgraph)による機械翻訳です。 【背景】無作為化対照試験では、集中治療室(ICU)の患者の緊密な血液グルコース対照から利益と害の両方を示しています。初期の非経口栄養の使用とインスリン誘発性の重度の低血糖症の変動は、この矛盾を説明するかもしれません。 【方法】ICU入院時に、患者をリベラルグルコースコントロール(血糖値がデシリットルあたり215 mg> 215 mg [> 11.9 mmolあたり])またはタイトグルコースコントロール(血中グルコースレベルを標的とする場合にのみ開始されたインスリンをランダムに割り当てました。デシリットルあたり80〜110 mgでロジックインスリンアルゴリズムを使用して[4.4〜6.1 mmolあたり6.1 mmol]);非経口栄養は、両方のグループで1週間差し控えられました。プロトコルアドヒアランスは、グルコースメトリックに従って決定されました。主な結果は、ICUのケアが必要である時間の長さであり、ICUから生存する時間に基づいて計算され、死は競合するリスクを占めています。90日間の死亡率は安全性の結果でした。 【結果】ランダム化を受けた9230人の患者のうち、4622人がリベラルなグルコースコントロールに割り当てられ、4608人がタイトなグルコースコントロールに割り当てられました。朝の血糖値の中央値は、リベラルグルコースコントロールを備えたデシリットルあたり140 mg(四分位範囲、122〜161)で、密集したグルコースコントロールを備えたデシリットルあたり107 mg(四分位範囲、98〜117)でした。重度の低血糖は、リベラルコントロールグループの31人の患者(0.7%)と、緊密なコントロール群で47人の患者(1.0%)で発生しました。ICUケアが必要とした時間の長さは、2つのグループで類似していました(タイトなグルコース制御、1.00; 95%信頼区間、0.96〜1.04; P = 0.94での早期の放電のハザード比)。90日での死亡率も類似していました(リベラルグルコースコントロールで10.1%、タイトなグルコースコントロールで10.5%、p = 0.51)。8つの事前に指定された二次転帰の分析により、新しい感染症の発生率、呼吸器および血行動態のサポートの期間、病院から生きたまま退院する時間、ICUと病院での死亡率は2つのグループで類似していることが示唆されましたが、重度の急性腎障害は類似していました。胆汁うっ滞肝機能障害は、密集したグルコース制御ではあまり一般的ではないように見えました。 【結論】初期の非経口栄養を受けていない重症患者では、タイトなグルコース制御は、ICUケアが必要な時間または死亡率に影響を与えませんでした。(Research Foundation-Flandersなどから資金提供。TGC-FAST ClinicalTrials.gov番号、NCT03665207。)。 第一人者の医師による解説 重症低血糖の発生率に有意差なし 早期の非経口栄養投与は避けるべき 鈴木 優矢 虎の門病院内分泌代謝科・糖尿病/森 保道 虎の門病院内分泌代謝科・糖尿病部長 MMJ.April 2024;20(1):17 集中治療室(ICU)入室患者の厳格な血糖コントロールについては、有益性を支持する報告がある一方で低血糖による有害性を示唆する報告もある。本論文で報告されたTGC-Fast試験は、ベルギー国内のICUに入室した9,230人を、ICU入室後1週間の非経口栄養投与を控え医原性高血糖を避けたうえで、従来治療群(血糖215 mg/dL超でインスリン治療を開始;4,622人)と治療強化群(血糖80~ 110 mg/dLを目標にコンピュータによるアルゴリズムを用いたインスリン治療;4,608人)に分けて、ICUで治療を要した期間を主要評価項目、90日死亡率を安全性評価項目として検討している。 早朝血糖の中央値は従来治療群が140 mg/dL、治療強化群が107 mg/dLであった。40 mg/dL未満の重症低血糖は従来治療群で0.7%、治療強化群で1.0%に生じたが、有意差はなかった。主要評価項目であるICU入室期間は両群で有意差はなく(ハザード比[HR], 1.00;95%信頼区間[CI], 0.96~ 1.04;P=0.94)、90日死亡率も従来治療群で10.1%、治療強化群で10.5%と有意差はなかった(P=0.51)。副次評価項目のうち、急性腎障害や胆汁うっ滞性肝障害(γ -GTP・ALP高値)の発生率は治療強化群で低かった。サブグループ解析では、神経学的疾患で入室した患者において、治療強化群の90日死亡率が低い傾向にあった(HR, 0.69;95% CI, 0.46 ~ 1.02)。 本試験で約80%の患者は糖尿病の既往はないが、従来治療群では45.9%、治療強化群では98.8%の患者でインスリンが使用されており、急性期重症患者ではインスリン抵抗性の増大によりストレス性高血糖を来し、平常とは血糖推移が異なる。ICU入室患者の厳格な血糖コントロールを検討した大規模な無作為化対照試験(RCT)であるNICESUGAR試験では、治療強化群で死亡率が上昇しており、その要因として治療強化による低血糖の関与が示唆されている(1)。本試験では、コンピュータによるアルゴリズムを用いることで、重症低血糖の発生率は治療強化群と従来治療群でほぼ同等となっているが、ICUで治療を要した期間や90日死亡率に有意差はなかった。 特定の疾患群や患者背景で治療強化が有用である可能性はあるが、医原性低血糖を最小限に抑えたとしても、急性期において治療強化による正常の血糖を目指した厳格な血糖管理の有用性は限定的である。ICUに入室する急性期重症患者では、早期の非経口栄養投与を控えることで急性期の医原性高血糖を避けながら、目標血糖値を平時よりも高めに設定し、高血糖時に治療を開始することが望ましいと考えられる。 1. NICE-SUGAR Study Investigators. N Engl J Med. 2009;360(13):1283-1297.
大量補液血液濾過透析は ハイフラックス血液透析より全死亡リスクを低下
大量補液血液濾過透析は ハイフラックス血液透析より全死亡リスクを低下
Effect of Hemodiafiltration or Hemodialysis on Mortality in Kidney Failure N Engl J Med. 2023 Aug 24;389(8):700-709. doi: 10.1056/NEJMoa2304820. Epub 2023 Jun 16. 上記論文のアブストラクト日本語訳 ※ヒポクラ 論文検索(Bibgraph)による機械翻訳です。 【背景】いくつかの研究では、腎不全の患者が標準的な血液透析と比較して高用量の血液ろ過の恩恵を受ける可能性があることを示唆しています。ただし、さまざまな公開された研究の制限を考えると、追加データが必要です。 【方法】少なくとも3か月間高フラックス血液透析を受けていた腎不全患者を含む、実用的な多国籍の無作為化対照試験を実施しました。すべての患者は、セッションあたり少なくとも23リットルの対流量の候補者であるとみなされ(高用量血液ろ過に必要な場合)、患者報告の結果評価を完了することができました。患者は、高用量の血液ろ過または従来の高フラックス血液透析の継続を受けるように割り当てられました。主な結果は、あらゆる理由からの死でした。主要な二次的な結果は、原因固有の死亡、致命的または脂肪性のない心血管イベントの複合、腎臓移植、および再発性の全原因または感染関連の入院でした。 【結果】合計1360人の患者が無作為化を受けました。683人が高用量の血液ろ過を受け、677人が高フラックス血液透析を受けます。フォローアップの中央値は30か月でした(四分位範囲、27〜38)。ヘモディアフィルトレーショングループでの試験中の平均対流量は、セッションあたり25.3リットルでした。あらゆる原因による死亡は、血液硬化群で118人の患者(17.3%)と血液透析群の148人の患者(21.9%)で発生しました(ハザード比、0.77; 95%信頼区間、0.65〜0.93)。 【結論】腎不全療法を引き起こす腎不全の患者では、高用量の血液濾過を使用すると、従来の高フラックス血液透析よりも原因による死亡のリスクが低くなりました。(欧州委員会の研究とイノベーションによって資金提供されています。オランダの裁判登録番号、NTR7138を説得します。)。 第一人者の医師による解説 今後のOnline-HDFの積極的な適用を再考するための貴重な研究 長田 太助 自治医科大学内科学講座腎臓内科学部門教授 MMJ.April 2024;20(1):19 透析患者の心血管(CV)合併症の頻度は高い。血液透析(HD)における中分子量以上の大きさの溶質の除去効率の低さが理由の1つと考えられている。ポアサイズが従来型のローフラックス(LF)膜より大きく、透水性の大きなハイフラックス(HF)膜が登場し、そのCVイベント予防効果に期待が集まったが、臨床研究では芳しい結果は得られていない(1)。拡散に頼ったHDでは、HF膜を用いても中分子量物質の濾過による除去効率の改善は難しい。そこで中分子量以上の大きさの溶質を濾過により積極的に除去する方法として血液濾過透析(HDF)が注目されている。その中でも高度に清浄化された透析液を使い、低コストで大量補液が調達可能なOnline(OL)-HDFが主流になってきた。2012年にGrootemanら(2)は、後希釈 OL-HDFとLF膜使用 HDの間で、全死亡・CVイベントについて検討し、有意差はなかったものの、大量補液 HDFで抑制される可能性を示した。その翌年、Maduellら(3)は、後希釈法としては大量の20L以上の補液を用いたOL-HDFとHF膜を用いたHDを比較する無作為化対照試験を実施し、OL-HDFで全死亡は30%、CV死は33%のリスク低下が得られることを示した。 本論文で紹介されているCONVINCE試験は、3カ月以上 HF-HDを継続していた患者1,360人を、23L以上の大量補液を用いた後希釈 HDF群(683人)とHF膜を用いたHD群(677人)に無作為化し中央値30カ月間観察した国際共同臨床試験である。全死亡率 はHDF群17.3 %、HD群21.9 % とHDF群で有意に抑制されていた(ハザード比[HR],0.77;P=0.005)。ただしCV疾患の 既往や糖尿病合併が背景にあるとこの差がみられなかった。CV死、致死的 /非致死的 CVイベントのHRはそれぞれ0.81、1.07で両群間に有意差を認めず、また入院のイベントに関しても両群間で差を認めなかった。 この結果をそのままわが国の医療現場に持ち込むには注意が必要である。日本のOL-HDFは圧倒的に前希釈法が多いからである。前希釈法では、拡散による小分子量物質や濾過による小~大分子量蛋白の除去効率が後希釈法に劣ることが知られている。また本試験において、高リスク透析患者ではOL-HDFの効果を認めず、本来 OL-HDFの効果を期待したい対象に効いていない。さらにOL-HDFはCV死の抑制傾向を示すが有意ではなく、低リスク透析患者の生命予後改善効果だけということになれば、それなりの医療資源の投入が必要であることを踏まえるとすべてOL-HDFに置き換えてしまえば良いというわけではない。OL-HDFの臨床現場での適用についても、一度立ち止まって考えてみる必要があるだろう。 1. Locatelli F, et al. J Am Soc Nephrol. 2009;20(3):645-654. 2. Grooteman Mp, et al. J Am Soc Nephrol. 2012;23(6):1087-1096. 3. Maduell F, et al. J Am Soc Nephrol. 2013;24(3):487-497.
地中海食とDASH食を組み合わせたMIND食 認知症予防効果は見いだせず
地中海食とDASH食を組み合わせたMIND食 認知症予防効果は見いだせず
Trial of the MIND Diet for Prevention of Cognitive Decline in Older Persons N Engl J Med. 2023 Aug 17;389(7):602-611. doi: 10.1056/NEJMoa2302368. Epub 2023 Jul 18. 上記論文のアブストラクト日本語訳 ※ヒポクラ 論文検索(Bibgraph)による機械翻訳です。 【背景】観察研究からの調査結果は、食事パターンが認知機能低下に対する保護的利益を提供する可能性があることを示唆していますが、臨床試験からのデータは限られています。マインドダイエットとして知られる神経変性遅延に対する地中海のダッシュ介入は、地中海ダイエットとダッシュ(高血圧を停止するための食事アプローチ)のハイブリッドであり、認知症のリスクの低下と推定される食品を含む修正を加えて、。 【方法】認知障害のない高齢者を含むが、認知症の家族歴を含む2サイトの無作為化対照試験を実施しました。および14項目のアンケートによって決定されるように、軽度のカロリー制限を伴うカロリー制限のあるコントロール食と比較して、軽度のカロリー制限でマインドダイエットの認知効果をテストするために、最適ではない食事。参加者に1:1の比率で割り当てて、3年間介入または対照食に従いました。すべての参加者は、割り当てられた食事の順守と減量を促進するためのサポートに関するカウンセリングを受けました。主なエンドポイントは、グローバル認知スコアと4つの認知ドメインスコアのベースラインからの変化であり、そのすべては12テストバッテリーから派生しました。各テストからの生のスコアはZスコアに変換されました。これは、すべてのテストで平均化され、グローバル認知スコアとコンポーネントテストを作成して4つのドメインスコアを作成しました。スコアが高いほど、認知パフォーマンスが向上します。二次的な結果は、参加者の非ランダムサンプルにおける脳特性の磁気共鳴画像法(MRI)由来の測定におけるベースラインからの変化でした。 【結果】合計1929人がスクリーニングを受け、604人が登録されました。301はマインドディエトグループに、303はコントロールダイエットグループに割り当てられました。試験は参加者の93.4%増加しました。ベースラインから3年目まで、両方のグループでグローバル認知スコアの改善が観察され、マインドダイエットグループの0.205標準化ユニットとコントロールダイエットグループの0.170標準化ユニット(平均差、0.035標準化ユニット; 95%信頼性間隔、-0.022〜0.092; p = 0.23)。MRIの白い高強化性、海馬量、および総灰色と白の体積の変化は、2つのグループで類似していました。 【結論】認知症の家族歴史を持つ認知的に障害のない参加者の間で、認知と脳のMRIの結果の変化は、ベースラインから3年目への変化は、心の食事に従った人と、軽度のカロリー制限で対照食を追った人との間で有意な差はありませんでした。(老化に関する国立研究所の資金提供; ClinicalTrials.gov番号、NCT02817074。)。 第一人者の医師による解説 バイアスの問題があり MIND食の有用性を否定するものではない 浦上 克哉 鳥取大学医学部保健学科認知症予防学講座(寄附講座)教授 MMJ.April 2024;20(1):25 近年、認知症予防における食の重要性が指摘されている。しかし、単一の食品、栄養素やサプリメントなどのデータは多く報告されてきているが、食事の有効性の検討はまれである。地中海食が認知症予防に有効であることは多くの観察研究で報告されているが(1)、比較試験はごくわずかである。 本論文は地中海食とDASH食を組み合わせたMIND食の有効性を検討した無作為化対照試験(MIND試験 )の報告である。地中海食はすでに認知症予防の効果があると報告されている食事で あ る。DASH(Dietary Approaches to Stop Hypertension)食は高血圧を防ぐための食事である。本試験は2施設で実施され、対象者の選択基準は65歳以上85歳未満の高齢者であること、認知症がないこと、認知症の家族歴を有すること、肥満であることなどであった。参加者を1:1の割合でランダムに割り付け、軽度カロリー制限を伴うMIND食または軽度カロリー制限を伴う対照食を3年間摂取させた。参加者は全員、割り付けられた食事の遵守に関するカウンセリングを受け、さらに減量を促進するためのサポートを受けた。主要評価項目は認知機能検査スコアのベースラインからの変化量、副次評価項目は3テスラ MRI検査で計測した脳(海馬、大脳皮質、大脳白質)の体積のベースラインからの変化量で、それぞれの食事が認知機能に与える効果が比較検証された。合計1,929人がスクリーニングを受け、そのうち604人が登録された。軽度カロリー制限を伴うMIND食摂取群に301人、軽度カロリー制限を伴う対照食摂取群に303人が割り付けられた。結果、3年間の経過観察を行ったが両群間で認知機能に有意差を認めなかった。脳 MRI画像でも体積に有意差を認めなかった。結論として今回の検討ではMIND食の認知症予防効果を見いだすことができなかった。 著者らは、今回の無作為化対照試験において従来の多くの観察研究と一致する結果が得られなかった理由としてバイアスの問題を挙げている。今回の研究対象として認知症の家族歴を有する人、肥満のある人を選択基準に加えたこと、また結果として学歴の高い人が多くなってしまい、一般的な人口集団を対象にできなかった可能性を指摘している。 本論文の結果はネガティブデータであったがMIND食の有用性を否定するものではないと考える。食は認知症予防に重要な役割を果たしていると考えられるので、今後のさらなる検討が待たれる。 1. Radd-Vagenas S, et al. Am J Clin Nutr. 2018;107(3):389-404.
双極Ⅰ型障害うつ病相寛解後の抗うつ薬継続 52週は8週に比べ気分エピソード再発抑制傾向を示す
双極Ⅰ型障害うつ病相寛解後の抗うつ薬継続 52週は8週に比べ気分エピソード再発抑制傾向を示す
Duration of Adjunctive Antidepressant Maintenance in Bipolar I Depression N Engl J Med. 2023 Aug 3;389(5):430-440. doi: 10.1056/NEJMoa2300184. 上記論文のアブストラクト日本語訳 ※ヒポクラ 論文検索(Bibgraph)による機械翻訳です。 【背景】抗うつ薬は、双極性障害患者の急性うつ病の治療に使用されますが、うつ病の寛解後の維持治療としての効果は十分に研究されていません。 【方法】最近、抑うつエピソードの寛解があった双極I障害患者における抗うつ薬療法の中止と比較して、補助エスシタロプラムまたはブプロピオンXLによる治療の維持に関するマルチサイト、二重盲検、無作為化、プラセボ対照試験を実施しました。患者は、寛解後52週間抗うつ薬による治療を継続するために、または8週間でプラセボに切り替えるために、1:1の比率でランダムに割り当てられました。イベントまでの分析で評価された主要な結果は、軽mania病またはマニアの症状、うつ病、自殺、および気分エピソードの重症度の測定症状のスコアで定義されているように、あらゆる気分エピソードでした。気分症状のための追加の治療または入院;または自殺の試みまたは完了。主要な二次的な結果には、マニアのエピソード、軽mania症またはうつ病のエピソードへの時間が含まれていました。 【結果】非盲検治療段階に関与した双極I障害のある209人の患者のうち、うつ病の寛解を有する150人が、直接登録された27人の患者に加えて、二重盲検フェーズに登録されました。合計90人の患者が52週間(52週間グループ)、処方された抗うつ薬による治療を継続するために割り当てられ、87人が8週間(8週間グループ)にプラセボに切り替えるように割り当てられました。採用が遅いため、完全な募集に達する前に、試験は停止しました。52週間で、52週間のグループの患者のうち28人(31%)、8週間のグループ(46%)の40人が主要な結果イベントを行いました。8週間のグループと比較して、52週間のグループの任意のムードエピソードのハザード比は0.68(95%信頼区間[CI]、0.43〜1.10、ログランクテストによるP = 0.12)でした。8週間のグループ(6%)の5人の患者と比較して、52週間のグループ(12%)の合計11人の患者は、マニアまたは軽mania症(ハザード比、2.28; 95%CI、0.86〜6.08)、および6.08)、および35人の患者(40%)と比較して15人の患者(17%)がうつ病の再発でした(ハザード比、0.43; 95%CI、0.25〜0.75)。有害事象の発生率は、2つのグループで類似していた。 【結論】双極性障害の患者と最近送金した抑うつエピソードを含む試験では、52週間継続したエスシタロプラムまたはブプロピオンXLによる補助的な治療は、あらゆる気分エピソードの再発を防ぐ際に8週間の治療と比較して有意な利益を示しませんでした。採用と資金の制限が遅いため、試験は早期に停止しました。(カナダの健康研究所の資金提供; ClinicalTrials.gov番号、NCT00958633。)。 第一人者の医師による解説 双極性うつ病に新規抗うつ薬併用が奏効した場合は 継続も検討すべき 坪井 貴嗣 杏林大学医学部精神神経科学教室准教授 MMJ.April 2024;20(1):12 2023年に日本の双極性障害(双極症)診療ガイドラインの改訂版(1)が発行された。そこには双極性障害抑うつエピソードに対して、気分安定薬もしくは第2世代抗精神病薬への抗うつ薬の併用療法を行わないことを弱く推奨すると記載がある。一方、カナダのガイドライン(2)では、双極 I型障害の抑うつエピソードの第2選択治療に、選択的セロトニン再取り込み阻害薬(SSRI)またはノルエピネフリン・ドパミン再取り込み阻害薬(NDRI)であるブプロピオン(日本未承認)との併用療法が挙げられており、また日本の大規模な観察研究(3)では双極 I型障害患者の32.1%に抗うつ薬が処方されている実態がある。さらに日本の上記ガイドラインでは、急性期の抑うつエピソードで有効だった薬剤を、維持期にすぐには中止せず一定期間の使用を提案しているが、新規抗うつ薬の併用療法についてはどうすべきか言及されていない。 本研究は、上記のカナダのガイドラインの著者と同じYatham先生を中心としたグループが、双極 I型障害の急性期抑うつエピソードに対し抗うつ薬の併用療法で寛解に達した患者を対象に、維持治療としてその抗うつ薬をいつまで継続すべきか明らかにすべく、実施した無作為化二重盲検プラセボ対照試験である。具体的には、エスシタロプラムまたはブプロピオン徐放性製剤と気分安定薬/第2世代抗精神病薬の併用補助療法を52週間継続する群(52週群)、それらを8週間継続した上で抗うつ薬をプラセボに切り替える群(8週群)が比較された。177人(52週群90人、8週群87人)が最終解析集団に組み入れられ、主要評価項目であるすべての気分エピソードの再発までの期間に関して52週群のハザード比は8週群に対して0.68(95%信頼区間[CI], 0.43 ~ 1.10)であったが、有意差はなかった(P=0.12)。副次評価項目については、躁 /軽躁エピソードの再発は52週群で11人(12%)、8週群で5人(6%)にみられ、抑うつエピソードの再発は52週群で15人(17%)、8週群で35人(40%)に認められた。本試験の限界点としては、計画されたサンプルサイズは216人であったが、COVID-19のための登録遅延や研究費支出期限のための早期中止などが挙げられる。 国際双極性障害学会(ISBD)の勧告(4)では抗うつ薬中止によって抑うつエピソードが再燃する場合、維持治療での抗うつ薬併用は許容されるとしており、本研究の結果はこれを一部支持するものかもしれず、今後もどのような双極 I型障害に対し抗うつ薬の併用が望ましいのか研究を重ねていく必要があるだろう。 1. 日本うつ病学会診療ガイドライン 双極症 2023(日本うつ病学会監修、医学書院) 2. Yatham LN, et al. Bipolar Disord. 2018;20(2):97-170. 3. Shinozaki M, et al. Asian J Psychiatr. 2022;67:102935. 4. Pacchiarotti I, et al. Am J Psychiatry. 2013;170(11):1249-1262.
成人における胸腺摘出術は 全死亡率とがん、自己免疫疾患のリスクを上昇させる
成人における胸腺摘出術は 全死亡率とがん、自己免疫疾患のリスクを上昇させる
Health Consequences of Thymus Removal in Adults N Engl J Med. 2023 Aug 3;389(5):406-417. doi: 10.1056/NEJMoa2302892. 上記論文のアブストラクト日本語訳 ※ヒポクラ 論文検索(Bibgraph)による機械翻訳です。 【背景】人間の成人における胸腺の機能は不明であり、胸腺の日常的な除去はさまざまな外科的処置で行われます。私たちは、免疫能力と全体的な健康を維持するために、成体胸腺が必要であると仮定しました。 【方法】胸腺切除なしで同様の心臓胸部手術を受けた人口統計学的に一致するコントロールと比較して、胸腺切除を受けた成人患者の死亡、癌、および自己免疫疾患のリスクを評価しました。T細胞産生および血漿サイトカインレベルも、患者のサブグループで比較されました。 【結果】除外後、胸腺切除術と6021のコントロールを受けた1420人の患者が研究に含まれました。胸腺切除術を受けた患者の1146は、コントロールが一致し、一次コホートに含まれていました。手術後5年後、全死因死亡率はコントロールグループよりも胸腺切除群の方が高かった(8.1%対2.8%、相対リスク、2.9; 95%信頼区間[CI]、1.7〜4.8)、癌のリスク(7.4%対3.7%、相対リスク、2.0; 95%CI、1.3〜3.2)。自己免疫疾患のリスクは、一次コホート全体のグループ間で実質的に差はありませんでしたが(相対リスク、1.1; 95%CI、0.8〜1.4)、術前感染、癌、または自己免疫疾患の患者が除外された場合に違いが見つかりました。分析から(12.3%対7.9%、相対リスク、1.5; 95%CI、1.02〜2.2)。5年以上の追跡調査(対照の有無にかかわらず)を持つすべての患者が関与する分析では、全米国の人口(9.0%対5.2%)よりも胸腺切除群で全死因死亡率が高かった。癌による死亡率でした(2.3%対1.5%)。T細胞産生および血漿サイトカインレベルが測定された患者のサブグループでは(胸腺切除群で22、対照群で19;平均フォローアップ、術後14.2)、胸腺切除を受けた人は、新たな産生の産生が少なくなりました。コントロールよりもCD4+およびCD8+リンパ球(平均CD4+シグナル関節T細胞受容体切除円[SJTREC]カウント、1451対526 DNA [P = 0.009];平均CD8+ SJTRECカウント、1466対447 DNA [1466対447P <0.001])および血液中のより高いレベルの炎症性サイトカイン。 【結論】この研究では、全死因死亡率と癌のリスクは、コントロールよりも胸腺切除を受けた患者の方が高かった。胸腺切除術は、術前感染、癌、または自己免疫疾患の患者が分析から除外された場合、自己免疫疾患のリスクの増加に関連していると思われました。(TraceyとCraig A. Huff Harvard Stem Cell Institute Research Support Fundなどから資金提供。)。 第一人者の医師による解説 重症筋無力症での胸腺摘出術の実施 今回のエビデンス踏まえ協働意思決定を 下畑 享良 岐阜大学大学院医学系研究科脳神経内科学分野教授 MMJ.April 2024;20(1):10 成人における胸腺の機能は不明で、かつ生理的萎縮を受ける最初の臓器であるため、成人では重要な役割を果たさないと広く信じられている。このため胸腺摘出術がさまざまな外科手技でルーチンに行われている。脳神経内科領域でも重症筋無力症(MG)に対して行われてきた。 本研究は、胸腺摘出術を受けた患者の全死亡とがん、自己免疫疾患のリスクを後方視的に検討したものである。方法は、マサチューセッツ総合病院で胸腺摘出術を受けた成人患者の死亡、がん、自己免疫疾患のリスクを、類似の心臓胸部手術を受けた、胸腺摘出術の経験のない対照と比較している。胸腺摘出術を受けた1,420人と対照6,021人が研究に組み入れられ、このうち胸腺摘出術を受けた1,146人が対照とマッチし検討が行われた。術後5年の時点で、全死亡率は胸腺摘出術群のほうが対照群よりも有意に高く(8.1% 対 2.8%;相対リスク[RR],2.9[95%信頼区間〈CI〉, 1.7~4.8])、がんも同様に有意に高かった(7.4% 対 3.7%;RR, 2.0[1.3~3.2])。自己免疫疾患については、術前に感染症、がん、自己免疫疾患を認めた患者を解析から除外すると有意差が認められた(12.3% 対 7.9%;RR, 1.5[1.02~2.2])。追跡期間が5年を超える全例を対象とすると、全死亡率は胸腺摘出術群のほうが米国の一般集団よりも高く(9.0% 対 5.2%)、がん死亡率も同様であった(2.3% 対 1.5%)。 さらに、T細胞産生量と血漿中サイトカイン濃度を測定した胸腺摘出術群22人と対照群19人の検討では、胸腺摘出術群はCD4陽性リンパ球とCD8陽性リンパ球の新生量が有意に少なく、逆に血中炎症性サイトカイン濃度が高かった。具体的には胸腺摘出術群で15種類のサイトカイン値が有意に変化し、炎症性サイトカインのIL-23、IL-33、トロンボポエチンのレベルは対照群の10倍以上であった。つまり胸腺摘出術群患者の免疫環境は、免疫調節異常と炎症を引き起こすことが知られるサイトカイン環境にシフトしていると考えられる。 MGでは、「胸線摘除の有効性が期待でき、その施行が検討される非胸腺腫 MG は、50歳未満の発症で、発病早期のAChR抗体陽性過形成胸線例である(重症筋無力症/ランバート・イートン筋無力症候群診療ガイドライン 2022)」とされ、以前と比べその適応患者は限定されているが、上記患者であっても、今回の新しいエビデンスを提示し、協働意思決定により治療方針を決定する必要があるだろう。また今後、MG 患者全体においても今回の論文と同様の検討が必要と思われる。
RNAi治療薬ジレベシランの第 I相試験 単回皮下投与で24週後も降圧効果が持続
RNAi治療薬ジレベシランの第 I相試験 単回皮下投与で24週後も降圧効果が持続
Zilebesiran, an RNA Interference Therapeutic Agent for Hypertension N Engl J Med. 2023 Jul 20;389(3):228-238. doi: 10.1056/NEJMoa2208391. 上記論文のアブストラクト日本語訳 ※ヒポクラ 論文検索(Bibgraph)による機械翻訳です。 【背景】アンジオテンシノーゲンは、アンジオテンシンペプチドの唯一の前駆体であり、高血圧の病因に重要な役割を果たしています。Zilebesiranは、長期にわたる作用期間を持つ治験RNA干渉治療剤が肝臓のアンジオテンシノゲン合成を阻害します。 【方法】このフェーズ1の研究では、高血圧症の患者は2:1の比率でランダムに割り当てられ、Zilebesiranの1回の上行皮下用量(10、25、50、100、200、400、または800 mg)またはプラセボのいずれかを受け取り、24週間続いた(パートA)。パートBでは、低塩または高塩の食事条件下での血圧に対するZilebesiranの800 mgの用量の効果、およびイルベサルタンとの採用時のその用量の効果を評価しました。エンドポイントには、安全性、薬物動態および薬力学的特性、および24時間の外来血圧モニタリングで測定される収縮期および拡張期血圧のベースラインからの変化が含まれます。 【結果】登録された107人の患者のうち、5人は軽度の一時的な注射部位反応を示しました。低血圧、高カリウム血症、または腎機能の悪化の報告は、医学的介入をもたらしませんでした。パートAでは、Zilebesiranを投与された患者は、投与された用量と相関していた血清アンジオテンシノゲンレベルの減少を示しました(8週目のr = -0.56; 95%信頼区間、-0.69〜 -0.39)。Zilebesiran(≥200mg)の単回投与量は、8週目までに収縮期血圧(> 10 mm Hg)と拡張期血圧(> 5 mm Hg)の減少と関連していました。これらの変化は、日中のサイクル全体で一貫しており、24週間で維持されました。パートBとEの結果は、高塩の食事による血圧への影響の減衰と、それぞれイルベサルタンとの同時投与による増強効果と一致していました。 【結論】血清アンジオテンシンゲンレベルと24時間の外来血圧の用量依存性減少は、200 mg以上のZilebesiranの単回皮下用量の後、最大24週間維持されました。軽度の注射部位反応が観察されました。(Alnylam Pharmaceuticals; ClinicalTrials.gov番号、NCT03934307; Eudract Number、2019-000129-39による資金提供)。 第一人者の医師による解説 RNAiのメカニズムを活用した革新的降圧薬 服薬アドヒアランス不良患者のコントロール改善を期待 苅尾 七臣 自治医科大学循環器内科学教授 MMJ.April 2024;20(1):6 RNA干渉(RNAi)薬が世界で注目を集めている。ベースとなっているのは、1998年にFire、Melloらの研究チームがNature誌に発表した、線虫への二本鎖 RNA導入により観察されたRNAiの機構である(1)。RNAiは、もともと生体に備わっている遺伝子発現抑制のプロセスであり、これを応用したRNAi薬は、疾患をもたらすタンパク質の産生をコードするメッセンジャー RNA(mRNA)を分解する。つまり、より上流のプロセスにおいて疾患を阻止できる可能性がある。この発見によって両博士は2006年にノーベル生理学・医学賞を受賞した。世界初のRNAi薬は、2018年に米国で承認されたトランスサイレチン型アミロイドーシス治療薬オンパットロである。日本では2023年12月時点でオンパットロを含む4剤のRNAi薬が承認されており、うち1剤は高コレステロール血症治療薬である。 本稿で紹介するのは、アンジオテンシノーゲンを治療標的とし肝臓のアンジオテンシノーゲンmRNAを特異的に減少させるように設計されたRNAi治療薬ジレベシラン(zilebesiran)の第 I相試験の結果である。本試験のPartAでは英国の4施設で登録した高血圧患者(収縮期血圧130 ~ 165mmHg)84人をプラセボ群(28人)と10、25、50、100、200、400、800 mg群(各群8人)にランダムに割り付け、試験薬を単回皮下投与して24週追跡した。治療を要する低血圧、高カリウム血症、または腎機能悪化の報告はなく、ジレベシラン 100 mgまたはそれ以上の用量を投与した群では、3週~12週目にかけて血中アンジオテンシノーゲン濃度が90%以上抑制されていた。また、ジレベシラン 200 mgまたはそれ以上の用量を投与した群は投与8週後の収縮期血圧がベースラインに比較して10 mmHg超低下し、投与24週後も降圧効果が持続した。この降圧効果は昼間から夜間・早朝にわたり24時間持続していた。 1回の注射で半年近い安定した降圧が得られる革新的治療薬の登場で、服薬アドヒアランス不良患者の血圧コントロール改善が期待される。2021年に国際共同疫学研究グループ NCD-RisCから発表された高血圧治療管理状況の長期推移によれば、降圧薬で治療しても、コントロールできているのはそのうちの半数に満たないという(2)。日本においても、既存降圧薬2剤以上で治療中の高血圧患者における早朝、夜間血圧のコントロール不良の割合はそれぞれ55%、45%にも及ぶ(3)。現在進行中の第 II相試験 KARDIA-1とKARDIA-2の結果が待たれる。 1. Fire A, et al. Nature. 1998;391(6669):806-811. 2. NCD Risk Factor Collaboration (NCD-RisC). Lancet. 2021;398(10304):957-980.(MMJ2022 年 4 月号で紹介) 3. Kario K, et al. Hypertens Res. 2023;46(2):357-367.
テストステロン補充療法はプラセボに対して心血管系有害事象の発生率で非劣性
テストステロン補充療法はプラセボに対して心血管系有害事象の発生率で非劣性
Cardiovascular Safety of Testosterone-Replacement Therapy N Engl J Med. 2023 Jul 13;389(2):107-117. doi: 10.1056/NEJMoa2215025. Epub 2023 Jun 16. 上記論文のアブストラクト日本語訳 ※ヒポクラ 論文検索(Bibgraph)による機械翻訳です。 【背景】性腺機能低下症の中高年男性におけるテストステロン補充療法の心臓血管への安全性はまだ確認されていない。 【方法】多施設共同無作為化二重盲検プラセボ対照非劣性試験において、心血管疾患の既往またはリスクが高く、性腺機能低下症の症状を報告し、空腹時テストステロンが2つある45~80歳の男性5,246人を登録した。 1デシリットルあたり300ng未満のレベル。患者は、1.62%テストステロンゲル(テストステロンレベルを1デシリットルあたり350~750ngに維持するように用量を調整した)を毎日経皮投与するか、プラセボゲルを投与するかに無作為に割り当てられた。心血管安全性の主要評価項目は、発生までの時間分析で評価された、心血管死、非致死性心筋梗塞、または非致死性脳卒中による死亡の複合要素の最初の発生であった。二次心血管エンドポイントは、イベント発生までの時間分析で評価された、心血管死、非致死性心筋梗塞、非致死性脳卒中、または冠動脈血行再建術の複合要素の最初の発生であった。非劣性には、テストステロンまたはプラセボを少なくとも 1 回投与された患者のハザード比の 95% 信頼区間の上限が 1.5 未満である必要がありました。 【結果】平均(±SD)治療期間は21.7±14.1ヶ月、平均追跡期間は33.0±12.1ヶ月でした。主要な心血管エンドポイントイベントは、テストステロン群の患者 182 人(7.0%)、プラセボ群の患者 190 人(7.3%)で発生しました(ハザード比、0.96、95% 信頼区間、0.78 ~ 1.17、p<0.001)。非劣性)。同様の所見は、テストステロンまたはプラセボの中止後のさまざまな時点でイベントに関するデータが打ち切られた感度分析でも観察されました。二次エンドポイント事象の発生率、または複合一次心血管エンドポイントの各事象の発生率は、2 つのグループで同様であるように見えました。テストステロン群では、心房細動、急性腎障害、肺塞栓症の発生率が高いことが観察されました。 【結論】性腺機能低下症を患い、心血管疾患の既往またはリスクが高い男性において、テストステロン補充療法は、重大な心臓有害事象の発生率に関してプラセボよりも劣りませんでした。 (AbbVie およびその他によって資金提供されています。TRAVERSE ClinicalTrials.gov 番号、NCT03518034。)。 第一人者の医師による解説 非致死的不整脈、心房細動、急性腎障害、肺塞栓症の既往がある場合は注意 佐々木 春明 昭和大学藤が丘病院泌尿器科教授 MMJ.April 2024;20(1):20 性腺機能低下症の中高年男性におけるテストステロン補充療法の心血管系への影響は確定されていない(1)。これまでの報告では、心血管リスクの上昇を示す研究もあれば、リスクの低下を示す研究もあり、相反する結果が示されている(1)。 本論文 は、米国 の316施設で実施された第4相、無作為化、二重盲検、プラセボ対照、非劣性試験(TRAVERSE試験)の報告である。45~80歳、性腺機能低下症状を有し、心血管疾患の既往があるか心血管リスクが高く、かつ午前11時までの採血による空腹時血清テストステロン値が300 ng/dL(10.4 nmol/L)未満が対象とされた。患者は、1.62%のテストステロンゲルを連日経皮投与する群(T群)またはプラセボ群(P群)に1:1で割り付けられた。安全性の主要評価項目は主要心血管イベント、あるいは心血管疾患・非致死的心筋梗塞・非致死的脳卒中による死亡までの期間とした。最大の解析対象集団(FAS)は5,204人(T群2,601人、P群2,603人)で、安全性解析対象は5,198人(T群2,596人、P群2,602人)であった。 12カ月時点の血清テストステロン値のベースラインからの上昇中央値はT群148 ng/dL、P群14ng/dLであった。平均(± SD)治療期間は21.7±14.1カ月、平均追跡期間は33.0±12.1カ月であった。主要心血管イベントは、T群で182人(7.0%)、P群で190人(7.3%)に発生した(ハザード比 ,0.96;95%信頼区間[CI];0.78 ~ 1.17;非劣性に関してP<0.001)。前立腺特異抗原(PSA)値はT群で有意に上昇したが(P<0.001)、前立腺がんの発生率は同程度であった(0.5% 対 0.4%;P=0.87)。T群では治療介入が必要な非致死的不整脈(5.2% 対 3.3%;P=0.001)、心房細動(3.5%対 2.4%;P=0.02)、急性腎障害(2.3% 対 1.5%;P=0.04)、肺塞栓症(0.9% 対 0.5%)が多かった。 本論文では、心血管疾患の既往があるか心血管リスクが高い男性性腺機能低下症において、テストステロン補充療法はプラセボに対して心血管系の有害事象の発生率に関して非劣性であったと結論している。また、前立腺がんの発生率も有意に上昇しなかったことが確認された。ただし、治療介入が必要な非致死的不整脈、心房細動、急性腎障害、肺塞栓症の既往がある場合は注意を要する。 日本でも男性性腺機能低下症が広く認知されるようになり、対象となる患者が増加しているので、安全に投与できることを再確認できた。 1. Bhasin S, et al. J Clin Endocrinol Metab. 2018;103(5):1715-1744.